You are on page 1of 176

BONES & JOINTS OF THE HEAD & NECK

I. THE SKULL (Fig—1,2,3 & 4)


 The skull has 22 bones, excluding the ossicles of the ear. The bones of the skull are attached to
each other by sutures
 Skull is composed of CRANIUM & MANDIBLE.
 The cranium can be subdivided into:
AN UPPER PART (calvaria), which surrounds the cranial cavity containing the brain.
A LOWER ANTERIOR PART (facial skeleton or viscerocranium)

Fig-1: Bones of Anterior aspect of the Skull

HEAD & NECK Page 1


II. ANTERIOR ASPECT OF THE SKULL
 METOPIC SUTURE: The original two halves of the frontal bone sometimes fail to fuse, leaving a
midline metopic suture.
 GLABELLA: Between these superciliary arches is a small depression (the glabella).
 NASION: The nasion is the intersection of the frontal bone and two nasal bones. It is a depressed
area between the eyes, just superior to the bridge of the nose & inferior to the glabella.
 SUPERCILIARY ARCH: Just superior to the rim of the orbit on each side are the raised superciliary
arches. These are more pronounced in men than in women.
 PIRIFORM APERTURE: The anterior nasal aperture or piriform aperture is a pear-shaped
opening in the skull. It is bounded above by the inferior borders of the nasal bones; laterally by the
thin, sharp margins which separate anterior from the nasal surfaces of the maxilla and below by
anterior nasal spine.
 NASAL BONES: The nasal bones are two small oblong bones, varying in size and form. They
contribute in the formation of "the bridge" of the nose.
 ANTERIOR NASAL SPINE:
 MAXILLA:
 ZYGOMATIC BONE:
 FORAMINA:
― Supraorbital
― Infraorbital
― Zygomaticofacial
― Zygomaticotemporal

III. SUPERIOR ASPECT OF THE SKULL


 CORONAL SUTURE:
 SAGITTAL SUTURE:
 BREGMA: The junction of coronal and sagittal suture is called the bregma.
 ANTERIOR FONTANELLE: Bregma is the site of anterior fontanelle.
 LAMBDOID SUTURE:
 LAMBDA: The junction of lamdoid and sagittal suture is called the lambda.
 POSTERIOR FONTANELLE: Lambda is the site of posterior fontanelle.

Fig-2: Bones of Superior aspect of the Skull

HEAD & NECK Page 2


IV. POSTERIOR ASPECT OF THE SKULL
 EXTERNAL OCCIPITAL PROTUBERANCE: It is an external pronounced lump on back of the
head.
 SUPERIOR NUCHAL LINES: These are curved lines extending laterally from external occipital
protuberance.

 EXTERNAL OCCIPITAL CREST: Extending downward from the external occipital protuberance is
the external occipital crest.
 INFERIOR NUCHAL LINE: About 1 inch below the superior nuchal lines, two additional lines (the
inferior nuchal lines) curve laterally.
 INION: The most prominent point of the external occipital protuberance is the inion.

Fig-3: Bones of Posterior aspect of the Skull

V. LATERAL ASPECT OF THE SKULL


 SUPERIOR & INFERIOR TEMPORAL LINES: The superior and inferior temporal lines begin as a
single line from posterior margin of the zygomatic process of the frontal bone and diverge as arch
backwards.
 PTERION: The junction where the frontal, parietal, sphenoid, and temporal bones are in close
proximity is the pterion. The bone in this area is particularly thin and overlies the anterior
division of the middle meningeal artery, which can be torn by a skull fracture in this area,
resulting in an extradural hematoma.
 ZYGOMATIC ARCH: It is formed by articulation of temporal process of zygomatic bone with the
zygomatic process of the temporal bone.
 EXTERNAL ACOUSTIC MEATUS: It is the opening into the temporal bone.
 STYLOID PROCESS: It is an elongated, narrow projection of bone which passes downwards from
the base of the tympanic part of the temporal bone.
 MASTOID PROCESS: A large prominence located immediately behind the external acoustic
meatus.

HEAD & NECK Page 3


Fig-4: Bones of Lateral aspect of the Skull

VI. CRANIAL CAVITY (Fig—5)


1. Anterior Cranial Fossa
 The greater part of the fossa is formed by the orbital plate of the frontal bone.
 At the base of the frontal crest is foramen cecum.
 The Crista galli is a thick median ridge posterior to the foramen cecum.
 On each side of Crista galli is the cribriform plate of the ethmoid.
2. Middle Cranial Fossa

 The middle cranial fossa has a central part composed of the sella turcica in the body of the
sphenoid and the large depressed parts on each side.
 The sella turcica is the saddle-like bony formation that is composed of:
a. The tuberculum sallae
b. The hypophysial fossa (pituitary fossa)
c. The dorsum sellae
 The optic canal is located anteromedial to the superior orbital fissure.
 On each side of the body of the sphenoid are:

HEAD & NECK Page 4










 1. SUPERIOR ORBITAL FISSURE: It is located between the greater and lesser wings of
sphenoid.
 2. FORAMEN ROTUNDUM: It is located posterolateral to the the superior orbital fissure.
3. FORAMEN OVALE: It is located posterolateral to the foramen rotundum.
4. FORAMEN SPINOUSM: It is located posterolateral to the foramen ovale.

 The foramen lecerum is ragged foramen, that lies posterolateral to the hypophysial fossa..

3. Posterior cranial fossa

 The internal occipital crest ends in the internal occipital protuberance.


 At the base of the petrous ridge of the temporal bone is the jugular foramen.
 Anterorsuperior to the jugular foramen is the internal acoustic meatus.
 The hypoglossal canal is superior to foramen magnum.

Fig-5: Superior view of the internal surface of the cranial base

HEAD & NECK Page 5


Summary of the More Important Openings in the Base of the
Skull and the Structures That Pass Through Them
Opening in Skull Bone of Skull Structures Transmitted
Anterior Cranial Fossa
Perforations in Ethmoid Olfactory nerves
cribriform plate

Middle Cranial Fossa


Optic canal Lesser wing of sphenoid Optic nerve,
Ophthalmic artery
Superior orbital fissure Between lesser and greater wings of Lacrimal,
sphenoid Frontal,
Trochlear,
Oculomotor,
Nasociliary, and
Abducent nerves;
Superior ophthalmic vein
Foramen rotundum Greater wing of sphenoid Maxillary division of the
trigeminal nerve
Foramen ovale Greater wing of sphenoid Mandibular division of the
trigeminal nerve,
Lesser petrosal nerve
Foramen spinosum Greater wing of sphenoid Middle meningeal artery
Foramen lacerum Between petrous part of temporal and Internal carotid artery
sphenoid

Posterior Cranial Fossa


Foramen magnum Occipital Medulla oblongata,
Spinal part of accessory nerve
Right and left vertebral arteries
Hypoglossal canal Occipital Hypoglossal nerve
Jugular foramen Between petrous part of temporal and Glossopharyngeal,
condylar part of occipital Vagus,
Accessory nerves;
Sigmoid sinus becomes internal
jugular vein
Internal acoustic Petrous part of temporal Vestibulocochlear and facial
meatus nerves

HEAD & NECK Page 6


VII. INFERIOR VIEW OF THE SKULL (Fig—6)
1. Anterior Part
 Palatal processes of the maxillae
 Horizontal plates of the palatine bones
 Incisive fossa and foramen.
 Greater palatine foramina.
 Lesser palatine foramina.
 Choanae (posterior nasal apertures).
 Vomer
2. Middle Part
 Medial pterygoid plates
 Pterygoid hamulus.
 Lateral pterygoid plate
 Greater wing of the sphenoid
― Foramen ovale
― Foramen spinosum
― Spine of the sphenoid.
 Foramen lacerum
 Carotid canal
 Temporal bone
― Squamous part of the temporal bone Petrous part of the temporal bone
― Tympanic plate of temporal bone
― Styloid process
― Mandibular fossa
― Articular tubercle
― Petrotympanic fissure
― Squamotympanic fissure
― Sylomastoid foramen
3. Posterior Part
 Foramen magnum
 Pharyngeal tubercle
 Occipital condyles
 Hypoglossal canal

HEAD & NECK Page 7


Fig-6: Inferior surface of the base of the skull.

HEAD & NECK Page 8


VIII. ORBIT (Fig—7)

Fig-7: Bones forming the walls of the right orbit.

HEAD & NECK Page 9


a. Roof
 It is formed by the orbital plate of frontal bone — It houses Superior orbital fissure & Optic canal.
b. Floor:
 It is formed by the orbital plate of maxilla — It has the infraorbital groove and canal which opens
at the face by infraorbital foramen — Posteriorly, on the floor, is the inferior orbital fissure.
c. Lateral Wall:
 It is formed by the zygomatic bones and the greater wing of sphenoid bone — It houses the
zygomaticofacial and zygomaticotemporal foramina.
d. Medial Wall:
 It is formed, from before backwards, by frontal process of maxilla, lacrimal bone, orbital plate of
ethmoid bone — It houses the anterior and posterior ethmoidal foramina.

IX. MANDIBLE (Fig—8)

Fig-8: Mandible.

1. Ramus
 Condylod process
 Coronoid process
2. Body
 Mandibular notch  Angle
 Mandibular foramen  Mental foramen
 Mandibular canal  Oblique line
 Lingula  Mylohyoid line
 Submandibular fossa
 Sublingual fossa
 Superior genial spine
 Inferior genial spine
 Digastric fossa
 Genu

HEAD & NECK Page 10


X. TEMPORAL FOSSA (Fig—9)
 It is depressed fossa on each side of the skull in the temporal region located between the temporal lines and
the zygomatic arch.
1. Boundaries
 Superiorly: Superior temporal line — Laterally: Zygomatic arch and Temporal fascia — Medially: Outer
surfaces of frontal, parietal, temporal and sphenoid bone — Anteriorly: frontal process of zygomatic bone &
zygomatic process of frontal bone — Inferiorly: Infratemporal crest.
2. Contents
a. Arteries: Deep temporal arteries and Superficail temporal artery
b. Veins: Superior temporal vein.
c. Nerves: Zygomaticotemporal nerve (branch of maxillary nerve), Deep temporal nerves (Branches of
mandibular nerve) and Temporal branch of facial nerve.
d. Muscles: Temporalis muscle.
XI. INFRATEMPORAL FOSSA (Fig—9)
1. Boundaries:
a. Floor: It is continuous with the spaces of the neck.
b. Roof: Greater wing of sphenoid (that has foramen ovale and foramen spinosum communicating
with middle cranial fossa) and the squamous and petrous parts of the temporal bone (that has
pterotympanic fissure communicating with middle ear cavity).
c. Lateral wall: Ramus of mandible (that has mandibular foramen communicating with mandibular
canal and mandibular notch communicating with the face).
d. Medial wall: Lateral pterygoid plate (that shares the wall of pterygomaxillary fissure
communicating with pterygopalatine fossa).
e. Anterior wall: Posterior surface of the maxilla (that shares the wall of inferior orbital
fissure communicating with orbit and alveolar foramina communicating with maxillary sinus).
f. Posterior wall: Styloid process.
2. Contents:
a. Arteries: 1ST and 2ND parts of the maxillary artery and middle maningeal artery.
b. Veins: Pterygoid plexus of veins.
c. Nerves: Mandibular division of trigeminal nerve and its branches (e.g: Ligual nerve,
Auriculotemporal nerve), Chorda tympani, Lesser petrosal nerve and Otic ganglion.
d. Muscles: Medial pterygoid, Lateral pterygoid and tendon of insertion of temporalis.
e. Miscellaneous: Deep part of the parotid gland.

XII. PTERYGOPALATINE FOSSA


1. Boundaries
a. Floor: Bottom of pterygomaxillary fissure (that has Greater palatine foramen
communicating with oral cavity and Lesser palatine foramen communicating with the
oropharynx).
b. Roof: Body of sphenoid with spenoidal air sinuses.
c. Medial wall: Perpendicular plate of palatine bone (that shares the wall of sphenopalatine
foramen communicating with nasal cavity)
d. Lateral wall: Pterygomaxillary fissure (communicating with infratemporal fossa)
e. Anterior wall: Posterior wall of the maxilla (that shares the wall of inferior orbital fissure
communicating with orbit).
f. Posterior wall: Pterygoid process of the sphenoid bone (that has foramen rotundum
communicating with middle cranial fossa, Pterygoid canal communicating with the foramen
lacerum and palatovaginal canal communicating with nasopharynx).

Head & Neck (4TH semester—Unit-I PRACTICAL MANUAL) Page 11


2. Contents
a. Arteries: 3RD part of the maxillary artery and its branches
b. Veins: Veins corresponding to the branches of maxillary artery.
c. Nerves: Maxillary nerve, Nerve of pterygoid canal and Pterygopalatine ganglion.

Fig-9: Infratemporal fossa.

Head & Neck (4TH semester—Unit-I PRACTICAL MANUAL) Page 12


XIII. TEMPOROMANDIBULAR JOINT (Fig—10 & 11)
1. Type
 Atypical synovial — Ball and socket.
2. Articulation
 Between the articular tubercle and the anterior portion of the mandibular fossa of the temporal
bone above & the head (condyloid process) of the mandible below — The articular surfaces are
covered with fibro-cartilage — The articular disc divides the joint into upper and lower cavities.
3. Capsule
 It surrounds the joint.
4. Ligaments
a. Lateral ligament:
 It strengthens the lateral aspect of the capsule. This ligament limits the movements of the
mandible in a posterior direction and thus protects the external auditory meatus.
b. Sphenomandibular ligament:
c. Stylomandibular ligament:.
5. Synovial membrane
 The synovial membrane lines all nonarticular surfaces of the upper and lower compartments of
the joint.
6. Nerve supply
 Auriculotemporal nerve
 Masseteric nerve
7. Movements
a. Depression of the mandible
 Depression of the mandible is brought about by lateral pterygoid assisted by the digastrics,
the geniohyoids and the mylohyoids.
b. Elevation of the mandible
 Elevation of the mandible is brought about by the temporalis, masseter and medial pterygoids.
c. Protrusion of the mandible
d. Retraction of the mandible
e. Lateral chewing movements
 These are accomplished by alternately protruding and retracting the mandible on each side. For
this to take place, the muscles responsible on both sides work alternately and not in unison.

Head & Neck (4TH semester—Unit-I PRACTICAL MANUAL) Page 13


Fig-10: Temporomandibular joint as seen from the lateral aspects.

Fig-11: Temporomandibular joint as seen from the medial) aspects.

Head & Neck (4TH semester—Unit-I PRACTICAL MANUAL) Page 14


XIV. CERVICAL VERTEBRAE (Fig—12)
 Seven cervical vertebrae form the cervical region of the vertebral column, which encloses the
spinal cord and meninges.
 There are FOUR typical cervical vertebrae (3RD to 6TH).
 There are THREE atypical cervical vertebra (1ST, 2ND & 7TH).

Fig-12: Cervical Vertebrae

Head & Neck (4TH semester—Unit-I PRACTICAL MANUAL) Page 15


1. First cervical vertebra
 A ring-like bone.
 It lacks a spinous process.
 It also lacks a body.
 It consists of two lateral masses connected by anterior and posterior arches.
 Its concave superior articular facets receive the occipital condyles to form the lateral atlanto-
occipital joints.
2. Second cervical Vertebra
 The C2 vertebrae or AXIS has a peg-like dens (odontoid process) projects superiorly from its body.
3. Typical cervical Vertebra
 The vertebral body is small and longer from side to side than anteroposteriorly.
 The superior surface of the body is concave, and the inferior surface is convex.
 The vertebral foramen is large and triangular.
 The transverse processes of all cervical vertebrae (typical or atypical) include transverse
foramina for the vertebral vessels.
 Their spinous processes are short and bifid.
4. Seventh cervical Vertebra
 The C7 vertebra or VERTEBRA PROMINENS is so named because of its long spinous process,
which is not bifid.
 Its transverse processes are large, but its transverse foramina are small.

XV. JOINTS OF CERVICAL VERTEBRAL COLUMN


(Fig—13)
1. Atlanto-occipital joint
a. Type
 Synovial joint.
b. Articulation
 Between the occipital condyles above and facets on the superior surfaces of the lateral masses
of the atlas below.
c. Capsule
 It surrounds the joint.
d. Ligaments
i. Anterior Atlanto-Occipital Membrane
 It connects the anterior arch of the atlas to the anterior margin of the foramen magnum.
ii. Posterior atlanto-occipital membrane
 It connects the posterior arch of the atlas to the posterior margin of the foramen magnum.
e. Movements
 Flexion, extension, and lateral flexion — No rotation is possible.

2. Atlanto-axial joint
a. Type & articulation
 These are three synovial joints—one median atlanto-axial joint: between the odontoid process of C2
and the anterior arch of the atlas and the other two lateral atlanto-axial joints: between the lateral
masses of the C1 & the lateral masses of the C2.
b. Capsule
 The joints are enclosed by capsules.

Head & Neck (4TH semester—Unit-I PRACTICAL MANUAL) Page 16


c. Ligaments:
i. Apical ligament
 It connects the apex of the odontoid process to the anterior margin of the foramen magnum.
ii. Alar ligaments
 They connect the odontoid process to the medial sides of the occipital condyles.
iii. Cruciate ligament
iv. Membrana tectoria
 It is attached above to the occipital bone just within the foramen magnum. It covers the posterior
surface of the odontoid process and the apical, alar, and cruciate ligaments
d. Movements
 There can be extensive rotation of the atlas and thus of the head on the axis.

Fig-13: Anterior view (A) and posterior view (B) of the atlanto-occipital joints. Sagittal section (C)
and posterior view (D) of the atlantoaxial joints.

XVI. PARANASAL SINUSES (Fig—14)


Head & Neck (4TH semester—Unit-I PRACTICAL MANUAL) Page 17
 The paranasal sinuses are air-filled extensions of the respiratory part of the nasal cavity.
 They are named according to the bones in which they are located.

Fig-14: Paranasal sinuses

1. Frontal sinus
 Frontal sinuses are usually detectable in children by 7 years of age.

Head & Neck (4TH semester—Unit-I PRACTICAL MANUAL) Page 18


2. Ethmoidal sinuses
 The ethmoidal cells usually are not visible in plain radiographs before 2 years of age but are
recognizable in CT scans
3. Sphenoidal sinuses
 The sphenoidal sinuses are located in the body of the sphenoid.
 They are related: Superiorly: to the pituitary gland and to the optic chiasma & Laterally: to the
cavernous sinuses & Inferiorly: to the nasal cavities.
4. Maxillary sinuses
 They occupy the bodies of the maxillae.
a. Boundaries:
 The apex: extends into the zygomatic bone — The base: forms lateral wall of the nasal cavity —
The roof: forms the floor of the orbit — The floor: is formed by the alveolar part of the maxilla.
b. Drainage:
 Into the middle nasal meatus of the nasal cavity by way of the semilunar hiatus.
XVII.CLINICAL CORRELATIONS
1. Fractures Of The Skull
 Fractures of the skull are common in the adult but much less so in the young child.
 In infant, bones are more resilient than in adult skull, and they are separated by fibrous sutural
ligaments.
 Moreover, the sutural ligaments begin to ossify during middle age.
 The petrous parts of the temporal bones and the occipital crests strongly reinforce the base of
the skull and tend to deflect linear fractures.
 In the young child, as the skull may be like a table-tennis ball, blow produces a depression without
splintering, the lesion usually referred to as a pond fracture.
2. Fractures Of Facial Bones
Bone Injuries and Skeletal Development
 In adults, the presence of well-developed, air-filled sinuses and the mucoperiosteal surfaces of the
alveolar parts of the upper and lower jaws means that most facial fractures should be considered
to be open fractures, susceptible to infection, and requiring antibiotic therapy.
Anatomy of Common Facial Fractures
 Signs of fractures of the facial bones include deformity, ocular displacement, or abnormal
movement accompanied by crepitation and malocclusion of the teeth.
 Anesthesia or paresthesia of the facial skin will follow fracture of bones through which branches of
the trigeminal nerve pass to the skin.
 The muscles of the face are thin and weak and cause little displacement of the bone fragments.
Once a fracture of the maxilla has been reduced, for example, prolonged fixation is not needed.
 However, in the case of the mandible, the strong muscles of mastication can create considerable
displacement, requiring long periods of fixation.
Nasal Fractures
 Fractures of the nasal bones, because of the prominence of the nose, are the most common facial
fractures.
 Because the bones are lined with mucoperiosteum, the fracture is considered open; the
overlying skin may also be lacerated.
Maxillofacial Fractures
 There is extensive facial swelling, midface mobility of the underlying bone on palpation,
malocclusion of the teeth with anterior open bite, and possibly leakage of cerebrospinal fluid
(cerebrospinal rhinorrhea) secondary to fracture of the cribriform plate of the ethmoid bone.
 Double vision (diplopia) may be present, owing to orbital wall damage.
 Involvement of the infraorbital nerve with anesthesia or paresthesia of the skin of the cheek and
upper gum may occur in fractures of the body of the maxilla.
 Nose bleeding may also occur in maxillary fractures. Blood enters the maxillary air sinus and
then leaks into the nasal cavity.
Blowout Fractures of the Maxilla

Head & Neck (4TH semester—Unit-I PRACTICAL MANUAL) Page 19


 A severe blow to the orbit (as from a baseball) may cause the contents of the orbital cavity to
explode downward through the floor of the orbit into the maxillary sinus.
 Damage to the infraorbital nerve, resulting in altered sensation to the skin of the cheek, upper lip,
and gum, may occur.
3. Fontanelles
 Palpation of the fontanelles enables the physician to determine:
o the progress of growth in the surrounding bones,
o the degree of hydration of the baby (e.g., if the fontanelles are depressed below the
surface, the baby is dehydrated), and
o the state of the intracranial pressure (a bulging fontanelle indicates raised intracranial
pressure).
 Samples of cerebrospinal fluid can be obtained by passing a long needle obliquely through the
anterior fontanelle into the subarachnoid space or even into the lateral ventricle.
 Clinically, it is usually not possible to palpate the anterior fontanelle after 18 months, because
the frontal and parietal bones have enlarged to close the gap.
4. Fractures of the anterior cranial fossa
 In fractures of the anterior cranial fossa, the cribriform plate of the ethmoid bone may be
damaged.
 This usually results in tearing of the overlying meninges, resulting into epistaxis and underlying
mucoperiosteum resulting into CSF rhinorrhea.
 Fractures involving the orbital plate of the frontal bone result in hemorrhage beneath the
conjunctiva and into the orbital cavity, causing exophthalmos.
 The frontal air sinus may be involved, with hemorrhage into the nose.
5. Fractures of the middle cranial fossa
 Fractures of the middle cranial fossa are common, because this is the weakest part of the base of
the skull.
 Anatomically, this weakness is caused by the presence of numerous foramina and canals in this
region; the cavities of the middle ear and the sphenoidal air sinuses are particularly vulnerable.
 The leakage of cerebrospinal fluid and blood from the external auditory meatus is common. The
7TH and 8TH cranial nerves may be involved as they pass through the petrous part of the temporal
bone.
 The 3RD, 4TH, and 6TH cranial nerves may be damaged if the lateral wall of the cavernous sinus is
torn.
6. Fractures of the posterior cranial fossa
 In fractures of the posterior cranial fossa, blood may escape into the nape of the neck deep to the
postvertebral muscles.
 Some days later, it tracks between the muscles and appears in the posterior triangle, close to the
mastoid process.
 The mucous membrane of the roof of the nasopharynx may be torn, and blood may escape
there.
 In fractures involving the jugular foramen, the 9TH, 10TH, and 11TH cranial nerves may be damaged.
 The strong bony walls of the hypoglossal canal usually protect the hypoglossal nerve from injury.
7. Blowout Fractures Of The Orbital Floor
 Blowout fractures of the orbital floor involving the maxillary sinus commonly occur as a result of
blunt force to the face.
 If the force is applied to the eye, the orbital fat explodes inferiorly into the maxillary sinus, fracturing
the orbital floor.
 Blowout fractures cause:
o Displacement of the eyeball, with resulting symptoms of double vision (diplopia).
o Injury to the infraorbital nerve, producing loss of sensation of the skin of the cheek
and the gum on that side.
o Entrapment of the inferior rectus muscle in the limiting upward gaze.
8. Fractures of the mandible
 The mandible is horseshoe shaped and forms part of a bony ring with the two TMJs and the base of
the skull.

Head & Neck (4TH semester—Unit-I PRACTICAL MANUAL) Page 20


 Traumatic impact is transmitted around the ring, causing a single fracture or multiple fractures of
the mandible, often far removed from the point of impact.
9. Clinical significance of the temporomandibular joint
 The temporomandibular joint lies immediately in front of the external auditory meatus.
 The great strength of the lateral temporomandibular ligament prevents the head of the
mandible from passing backward and fracturing the tympanic plate when a severe blow falls on
the chin.
 The articular disc of the temporomandibular joint may become partially detached from the
capsule, and this results in its movement becoming noisy and producing an audible click during
movements at the joint.
10. Dislocation of the temporomandibular joint
 Dislocation sometimes occurs when the mandible is depressed.
 In this movement, the head of the mandible and the articular disc both move forward until they
reach the summit of the articular tubercle.
 In this position, the joint is unstable, and a minor blow on the chin or a sudden contraction of the
lateral pterygoid muscles, as in yawning, may be sufficient to pull the disc forward beyond the
summit.
 In bilateral cases the mouth is fixed in an open position, and both heads of the mandible lie in
front of the articular tubercles.
 Reduction of the dislocation is easily achieved by pressing the gloved thumbs downward on the
lower molar teeth and pushing the jaw backward.
 The downward pressure overcomes the tension of the temporalis and masseter muscles, and
the backward pressure overcomes the spasm of the lateral pterygoid muscles.
11. Hangman’s fracture
 A hangman's fracture is the name given to a fracture of both pedicles of the axis vertebra (C2).
 The mechanism of the injury is forcible hyperextension of the head.
 The vertebral body of C2 is severely subluxed from C3, causing the spinal cord being crushed.
 The causes are:

― Judicial hanging, when the noose was placed below the condemned subject's chin.
When the subject was dropped, the head would be forced into hyperextension by the full
weight of the body.
― Falls, usually in older adults
― Car crash, especially with no seat belt, when a person slams his chin against the
steering wheel, dashboard, or windshield, causing the hyperextension to occur.
― Diving injuries
― Collisions between players in contact sports.

12. Sinusitis and the examination of the paranasal


sinuses
 Infection of the paranasal sinuses is a common complication of nasal infections.
 Rarely, the cause of maxillary sinusitis is extension from an apical dental abscess.
The frontal sinus
 The frontal sinus can be examined by pressing the finger upward beneath the medial end of the superior orbital
margin. Here the floor of the frontal sinus is closest to the surface.
 The frontal sinus is innervated by the supraorbital nerve, which also supplies the skin of the forehead and
scalp as far back as the vertex. It is, therefore, not surprising that patients with frontal sinusitis have pain referred
over this area.

The ethmoidal sinuses


 The ethmoidal sinuses can be palpated by pressing the finger medially against the medial wall of the
orbit.

Head & Neck (4TH semester—Unit-I PRACTICAL MANUAL) Page 21


The maxillary sinus
 The maxillary sinus can be examined for tenderness by pressing the finger against the anterior wall of
the maxilla below the inferior orbital margin; pressure over the infraorbital nerve may reveal increased
sensitivity.
 Radiologic examination of the sinuses is also most helpful in making a diagnosis. One should
always compare the clinical findings of each sinus on the two sides of the body.
 The maxillary sinus is innervated by the infraorbital nerve and, in this case, pain is referred to the
upper jaw, including the teeth.
 The maxillary sinus is particularly prone to infection because its drainage orifice through the hiatus
semilunaris is badly placed near the roof of the sinus.

XVIII.REVIEW QUESTIONS:
Q.1: EARLY CLOSURE OF THE FONTANELLES OF THE INFANT SKULL CAN RESULT IN
COMPRESSION OF THE BRAIN, RESTRICTING BRAIN GROWTH. WHICH OF THE FOLLOWING
FONTANELLES IS LOCATED AT THE JUNCTION OF SAGITTAL AND CORONAL SUTURES AND
AT WHAT AGE DOES THIS FONTANELLE TYPICALLY CLOSE?
A. Posterior fontanelle, which closes at about 2 years
B. Mastoid fontanelle, which closes at about 16 months
C. Lambdoid fontanelle, which closes at 8 months to 1 year
D. Sphenoidal fontanelle, which closes at 3 years
E. Anterior fontanelle, which closes at 18 months
ANSWER: E.
 The anterior fontanelle is located at the junction of the sagittal and coronal sutures and closes at
around 18 months of age.
Q.2: A 40-YEAR-OLD UNCONSCIOUS MAN IS ADMITTED TO THE EMERGENCY DEPARTMENT
AFTER BEING HIT IN THE HEAD WITH A BASEBALL. A CT SCAN EXAMINATION REVEALS A
FRACTURED PTERION AND AN EPIDURAL HEMATOMA. BRANCHES OF WHICH OF THE
FOLLOWING ARTERIES ARE MOST LIKELY TO BE INJURED?
A. External carotid
B. Superficial temporal
C. Maxillary
D. Deep temporal
E. Middle meningeal
ANSWER: E.
 The middle meningeal artery is a branch of the maxillary artery and courses between the dura mater
and skull close to the area of the pterion. Any fracture or impact trauma to this location typically
results in a laceration of the middle meningeal artery resulting in an epidural hematoma

Q.3: A 54-YEAR-OLD MAN WAS ADMITTED TO THE EMERGENCY DEPARTMENT AFTER HE WAS
STRUCK BY AN AUTOMOBILE. RADIOGRAPHIC EXAMINATION REVEALED A FRACTURE
THROUGH THE CRISTA GALLI OF THE ANTERIOR CRANIAL FOSSA, RESULTING IN SLOW,
LOCAL BLEEDING. WHICH OF THE FOLLOWING IS THE MOST LIKELY SOURCE OF BLEEDING?
A. Middle meningeal artery
B. The great cerebral vein of Galen
C. Superior sagittal sinus
D. Straight venous dural sinus
E. Superior ophthalmic vein
ANSWER: C.
 The superior sagittal sinus would most likely be the source of the bleeding because it attaches
anteriorly to the crista galli and because of the slow nature of the bleed.

Q.4: TRAUMA TO WHICH OF THE FOLLOWING STRUCTURES COULD RESULT IN LEAKAGE OF


CEREBROSPINAL FLUID INTO THE NASAL CAVITY:

A. Foramen spinosum

Head & Neck (4TH semester—Unit-I PRACTICAL MANUAL) Page 22


B. Jugular foramen
C. Foramen magnum
D. Foramen ovale
E. Cribriform plate of ethmoid bone
ANSWER: E.

Q.5: A 25-YEAR OLD MAN IS INVOLVED IN AN AUTOMOBILE ACCIDENT AND SLAMS HIS HEAD
INTO A CONCRETE WALL OF A BRIDGE. HIS ct SCAN REVEALS THAT THE MIDDLE MENINGEAL
ARTERY HAS RUPTURED BUT MENINGES REMAIN INTACT. BLOOD FROM THIS ARTERY
ENTERS WHICH OF THE FOLLOWING SPACES:

A. Subarachnoid space
B. Subdural space
C. Epidural space
D. Subpial space
E. Cranial dural sinuses
ANSWER: C.

Q.6: A 14-YEAR OLD BOY HITS HIS HEAD ON THE ROAD AFTER FALLING OFF HIS
MOTORCYCLE. HIS RADIOGRAPH REVEALS DAMAGE TO THE SELLA TURCICA. THIS IS
PROBABLY DUE TO FRACTURE OF WHICH OF THE FOLLOWING BONES:

A. Frontal bone
B. Ethmoid bone
C. Temporal bone
D. Basioccipital bone
E. Sphenoid bone.
ANSWER: E.

Q.7: DURING A GAME, A 26-YEAR OLD BASEBALL PLAYER RECEIVES A SEVERE BLOW TO THE
HEAD THAT FRACTURES THE OPTIC CANAL. WHICH OF THE FOLLOWING PAIRS OF
STRUCTURES IS MOST LIKELY DAMAGED:

A. Optic nerve and ophthalmic vein


B. Ophthalmic vein and ophthalmic nerve
C. Ophthalmic artery and optic nerve
D. Ophthalmic nerve and optic nerve
E. Ophthalmic artery and ophthalmic vein
ANSWER: C.

Q.8: A 37-YEAR OLD MAN FEELS A LITTLE DISCOMFORT WHEN MOVING HIS TONGUE,
PHARYNX, AND LARYNX. PHYSICAL EXAMINATION INDICATES THAT THE MUSCLES ATTACHED
TO STYLOID PROCESS ARE PARALYZED. WHICH OF THE FOLLOWING GROUP OF CRANIAL
NERVES ARE DAMAGED:

A. Facial, glossopharyngeal and hypoglossal nerves


B. Hypoglossal, vagus and facial nerves
C. Glossopharyngeal, trigeminal and vagus nerves
D. Vagus, spinal accessory and hypoglossal nerves
E. Facial, glossopharyngeal and vagus nerves.
ANSWER: A.

XIX.CASE STUDY
Case—1

Head & Neck (4TH semester—Unit-I PRACTICAL MANUAL) Page 23


A 26-YEAR-OLD BASEBALL PLAYER WAS STRUCK ON THE RIGHT SIDE OF THE HEAD WITH A
BALL. THE PLAYER FELL TO THE GROUND BUT DID NOT LOSE CONSCIOUSNESS. AFTER
RESTING FOR 1 HOUR AND THEN GETTING UP, HE WAS SEEN TO BE CONFUSED AND
IRRITABLE. LATER, HE STAGGERED AND FELL TO THE FLOOR. ON QUESTIONING, HE WAS
SEEN TO BE DROWSY, AND TWITCHING OF THE LOWER LEFT HALF OF HIS FACE AND LEFT
ARM WAS NOTED. A DIAGNOSIS OF EPIDURAL HEMORRHAGE WAS MADE:

1. What is the source of bleeding


ANS: The anterior branch of the middle meningeal artery may be sectioned due to fracture at the site
of pterion, that results into epidural hematoma.

2. Why is there twitching of the lower left half of his face and left arm.
ANS: Epidural hematoma at the site of right pterion can exert pressure on the lower end of the
precentral gyrus of the right cerebral hemisphere. The lower end of the right precentral gyrus (motor
area) supplies the lower left half facial muscles and the muscles of the left upper limb.

3. What is lucid period


ANS: A lucid interval is a temporary improvement in a patient's condition after a traumatic brain injury,
after which the condition worsens. A lucid interval is especially indicative of an epidural hematoma.

Case—2
AN 80-YEAR OLD LADY PRESENTED TO THE EMERGENCY DEPARTMENT (ED) WITH NECK PAIN
3 DAYS AFTER A FALL. SHE DENIED ANY HEAD INJURY, LOSS OF CONSCIOUSNESS OR ANY
OTHER INJURIES. THERE WERE NO PRECEDING HEADACHES, CHEST PAIN OR PALPITATIONS.
THE LADY WOKE UP THE FOLLOWING DAY WITH RESTRICTED NECK MOVEMENTS DUE TO
CONSTANT PAIN. EXAMINATION REVEALED REDUCED RANGE OF NECK MOVEMENTS, LIMITED
BY PAIN. THE LADY WAS ABLE TO FLEX HER HEAD TO TOUCH HER CHEST BUT HAD REDUCED
LATERAL ROTATION, LATERAL FLEXION AND EXTENSION TO 10 DEGREES. THERE WERE NO
ABNORMAL NEUROLOGICAL SIGNS AND SYSTEMIC EXAMINATION REVEALED NO OTHER
ABNORMALITIES.

1. Which cervical vertebra is more commonnly fractured in this type of injury?

ANS: Cervical spine fractures are also common following trauma with C2 being fractured most
frequently, 55% affect the odontoid.

2. Why the patient was able to flex her head but had reduced rotation?

ANS: The patient was able to flex her head because flexion takes place at the joint between C1 and
occipital condyles (atlanto-occipital joint) but he was unable to rotate her head because rotation takes
place at the joint between C1 and C2 (atlanto-axial joint).

SKIN & FASCIAE OF THE HEAD & NECK

Head & Neck (4TH semester—Unit-I PRACTICAL MANUAL) Page 24


I. THE SCALP (Fig—15)
 The scalp is a multilayered structure with layers that can be defined by the word itself.

1. S=skin
 Thickest in the body and is thickest of all is in the occipital region — Hairiest —Contains high
concentration of sebaceous glands.

2. C=connective tissue (dense)


 It is composed of many fibrous septa which unite the overlying skin (layer 1) with the
underlying epicranial aponeurosis (layer 3) — It contains: Superficial arteries, Superficial
veins and Cutaneous nerves.

3. A=aponeurotic layer
 It consists of occipitofrontalis muscle, which has a frontal belly anteriorly, an occipital belly
posteriorly, and an aponeurotic tendon-the epicranial aponeurosis -connecting the two.

4. L=loose connective tissue


 It lies beneath the epicranial aponeurosis, hence called subaponeurotic space — It provides
a plan upon which the first three layers of the scalp move as a single unit — It extends down
beneath the orbicularis oculi into the eyelids — It contains some important emissary veins
(valveless veins that connect the superficial veins of scalp with the diploic veins of the skull).

5. P=pericranium
 Periosteum on the outer surface of the calvaria.

Head & Neck (4TH semester—Unit-I PRACTICAL MANUAL) Page 25


Fig-15: Coronal section of the upper part of the head showing the layers of the scalp.

Fig-16: Cross section of the neck at the level of the sixth cervical vertebra.

II. THE DEEP CERVICAL FASCIA (Fig—16)


 It consists of four parts—investing layer, pretracheal layer, prevertebral layer and carotid sheath.
1. Investing layer
 INVESTS STERNOCLEIDOMASTOID & TRAPEZIUS: It splits to enclose the
sternocleidomastoid muscle and trapezius.
 PAROTID FASCIA: It splits to invest the parotid gland and is called parotid fascia.
 TEMPORAL FASCIA: From the zygomatic arch, it extends upward to become continuous with
temporal fascia.
 EPICRANIAL APONEUROSIS: From here, it extends further upward and sweeps over the scalp
to be called epicranial aponeurosis (3rd layer of the scalp).
 STYLOMANDIBULAR LIGAMENT: From the styloid process to the angle of the mandible it
thickens to form the stylomandibular ligament.
 SUPRASTERNAL SPACE: Just a short distance above the jugular notch, the it splits to form
the suprasternal space, that contains jugular venous arch.
2. Pretracheal layer
 FALSE CAPSULE OF THYROID GLAND: It covers front of the trachea — It splits to enclose
the thyroid gland, to which is not adherent except between the isthmus of gland and 2nd 3rd
and 4th tracheal rings.

Head & Neck (4TH semester—Unit-I PRACTICAL MANUAL) Page 26


 EXTENDS INFERIORLY TO BE CONTINUOUS WITH FIBROUS PERICARDIUM: Inferiorly it
extends to pass behind the manubrium sterni to enter the thorax and attach with the outer
layer of the heart.
3. Prevertebral fascia
 AXILLARY SHEATH: It lies in front of the prevertebral muscles — It extends downward in the
axilla to form the axillary sheath.
4. Carotid sheath:
 CONTENTS: It surrounds the common and internal carotid arteries (not the external carotid
artery), internal jugular vein, vagus nerve and some deep cervical lymph nodes —
 INFERIOR & SUPERIOR EXTENSIONS: Superiorly it extends up to base of skull & Inferiorly, it
extends into the thorax to blend with the outer layer of the great vessels.

III. MENINGES
 The cranial meninges are coverings of the brain that lie immediately internal to the cranium
— The meninges are Dura mater, Arachnoid mater & Pia mater.
1. Dura mater
 The two layers of the cranial duramater are: 1. An external endosteal layer: It is formed by the
endosteum lining the internal surface of the bone. 2. An internal meningeal layer: It is
continuous with the dura mater covering the spinal cord. Except where the dural sinuses
occur, the meningeal layer is intimately fused with the endosteal layer.
2. Arachnoid mater
 It is separated from pia by the subarachnoid space, which contains cerebrospinal fluid (CSF).
3. Pia mater
 It is highly vascularized layer — It adheres to the surface of the brain and follows all its
contours.

IV.CLINICAL CORRELATIONS
1. Clinical significance of the scalp structure
 The skin of the scalp possesses numerous sebaceous glands, the ducts of which are prone to
infection and damage by combs. For this reason, sebaceous cysts of the scalp are common.
2. Lacerations of the scalp
 The scalp has a profuse blood supply to nourish the hair follicles.
 Even a small laceration of the scalp can cause severe blood loss.
 It is often difficult to stop the bleeding of a scalp wound because the arterial walls are attached
to fibrous septa in the subcutaneous tissue and are unable to contract or retract to allow blood
clotting to take place.
 The tension of the epicranial aponeurosis, produced by the tone of the occipitofrontalis muscles,
is important in all deep wounds of the scalp.For satisfactory healing to take place, the opening in
the aponeurosis must be closed with sutures.
3. Scalp infections
 Occasionally, an infection of the scalp spreads by the emissary veins, which are valveless, to
the skull bones, causing osteomyelitis.

Head & Neck (4TH semester—Unit-I PRACTICAL MANUAL) Page 27


 On the other hand, subperiosteal blood or pus is limited to one bone because of the
attachment of the periosteum to the sutural ligaments.
4. Fascial spaces
 Between the dense layers of deep fascia in the neck is loose connective tissue that forms
potential spaces that are clinically important.
 More important spaces are visceral, retropharyngeal, submandibular, & masticatory spaces.
 It is possible for blood, pus, or air in the retropharyngeal space to spread downward into
the superior mediastinum of the thorax.
5. Acute infections of the fascial spaces of the neck
 Dental infections most commonly involve the lower molar teeth.
 The infection spreads medially from the mandible into the submandibular and masticatory
spaces and pushes the tongue forward and upward.
 Further spread downward may involve the visceral space and lead to edema of the vocal
cords and airway obstruction.
 Ludwig's angina is an acute infection of the submandibular fascial space and is commonly
secondary to dental infection.
6. Chronic infection of the fascial spaces of the neck
 Tuberculous infection of the deep cervical lymph nodes can result in liquefaction and
destruction of one or more of the nodes.
 The pus is at first limited by the investing layer of the deep fascia.
 Later, this becomes eroded at one point, and the pus passes into the less restricted
superficial fascia.
 A dumbbell or collar-stud abscess is now present.
 The clinician is aware of the superficial abscess but must not forget the existence of the
deeply placed abscess.
7. Intracranial hemorrhages
 Intracranial hemorrhage may result from trauma or cerebral vascular lesions.
 Four varieties are considered here:
a. Extradural hemorrhage
 Extradural hemorrhage results from injuries to the meningeal arteries or veins.
 The most common artery to be damaged is the anterior division of the middle meningeal
artery.
 A comparatively minor blow to the side of the head, resulting in fracture of the skull in the
region of the pterion, may sever the artery.
 Bleeding strips up the meningeal layer of dura from the internal surface of the skull. The
intracranial pressure rises, and the enlarging blood clot exerts local pressure on the
underlying motor area in the precentral gyrus.
 Blood may also pass outward through the fracture line to form a soft swelling under the
temporalis muscle.
 To stop the hemorrhage, the torn artery must be ligated or plugged.
b. Subdural hemorrhage
 Subdural hemorrhage results from tearing of the superior cerebral veins at their point of
entrance into the superior sagittal sinus.
 The cause is usually a blow on the front or the back of the head, causing excessive
anteroposterior displacement of the brain within the skull.
 Once the vein is torn, blood under low pressure begins to accumulate in the potential space
between the dura and the arachnoid.
 Acute and chronic forms of the clinical condition occur, depending on the speed of accumulation
of fluid in the subdural space.
― If the patient starts to vomit, the venous pressure will rise as a result of a rise in
the intrathoracic pressure. Under these circumstances, the subdural blood clot will
increase rapidly in size and produce acute symptoms.
― In the chronic form, over a course of several months, the small blood clot will
attract fluid by osmosis so that a hemorrhagic cyst is formed, which gradually
expands and produces pressure symptoms.

Head & Neck (4TH semester—Unit-I PRACTICAL MANUAL) Page 28


 In both forms the blood clot must be removed through burr holes in the skull.
c. Subarachnoid hemorrhage
 Subarachnoid hemorrhage results from rupture of a congenital aneurysm on the circle of
Willis.
 The symptoms, which are sudden in onset, include:
― severe headache,
― stiffness of the neck, and
― loss of consciousness.
 The diagnosis is established by withdrawing heavily blood-stained cerebrospinal fluid through
a lumbar puncture.
d. Cerebral hemorrhage
 Cerebral hemorrhage is generally caused by rupture of the thin-walled lenticulostriate artery,
a branch of the middle cerebral artery.
 The hemorrhage involves the vital corticobulbar and corticospinal fibers in the internal
capsule and produces hemiplegia on the opposite side of the body.
 The patient immediately loses consciousness, and the paralysis is evident when
consciousness is regained.
V. REVIEW QUESTIONS
Q.1: A 55-YEAR-OLD MAN IS ADMITTED TO THE HOSPITAL AFTER AN INJURY SUSTAINED
AT WORK IN A FACTORY. HE PRESENTS WITH SEVERE SCALP LACERATIONS, WHICH
WERE SUTURED. AFTER THREE DAYS THE WOUND IS INFLAMED, SWOLLEN, AND PAINFUL.
BETWEEN WHICH TISSUE LAYERS IS THE INFECTION MOST LIKELY LOCATED:
A. The periosteum and bone
B. The aponeurosis and the periosteum
C. The dense connective tissue and the aponeurosis
D. The dense connective tissue and the skin
E. The dermis and the epidermis
ANSWER: B.
 The scalp is divided into five layers: skin, dense connective tissue, aponeurosis, loose connective
tissue, and periosteum. Typically, infections will be located in the loose connective tissue because
of the ease with which infectious agents spread via the many veins located in this region. This
area is usually referred to as the “danger zone” of the scalp mainly because scalp infections here
can be transmitted into the skull via emissary veins, then via diploic veins of the bone to the
cranial cavity.

Q.2: THE ARACHNOID VILLI ALLOW CEREBROSPINAL FLUID TO PASS BETWEEN WHICH
TWO OF THE FOLLOWING SPACES:
A. Choroid plexus and subdural space
B. Subarachnoid space and superior sagittal sinus
C. Subdural space and cavernous sinus
D. Superior sagittal sinus and jugular vein
E. Epidural and subdural space
ANSWER: B.
 The arachnoid villi are extensions of the arachnoid mater into the superior sagittal sinus. The villi
allow for proper drainage of the CSF into the venous bloodstream from the subarachnoid space in
which the CSF circulates. The villi are a crucial element in maintaining proper intracranial
pressure and circulation of the CSF.
Q.3: A 55-YEAR-OLD MAN IS ADMITTED TO THE EMERGENCY DEPARTMENT AFTER
SLIPPING ON WET PAVEMENT AND FALLING. PHYSICAL EXAMINATION REVEALS THAT THE
PATIENT HAS A HEMATOMA THAT FORMED IN THE DANGER ZONE OF THE SCALP,
SPREADING TO THE AREA OF THE EYELIDS. WHICH OF THE FOLLOWING LAYERS IS
REGARDED AS THE “DANGER ZONE”?
A. Loose areolar layer
B. Skin
C. Galea aponeurotica

Head & Neck (4TH semester—Unit-I PRACTICAL MANUAL) Page 29


D. Pericranium
E. Subcutaneous layer
ANSWER: A.
 The loose areolar connective tissue layer is known as the “danger zone” because hematoma can
spread easily from this layer into the skull by means of emissary veins that pass into and through
the bones of the skull. None of the other scalp layers listed is referred to as the “danger zone.”
Q.4: A NEWBORN INFANT IS FI NALLY DELIVERED WITH FORCEPS AFTER A DIFFI CULT DELIVERY.
UPON PHYSICAL EXAMINATION OF THE NEWBORN A CEPHALOHEMATOMA IS NOTED FROM
RUPTURE OF SMALL PERIOSTEAL ARTERIES. BETWEEN WHICH OF THE FOLLOWING LAYERS OF
TISSUE DOES THE BLOOD ACCUMULATE?
A. Between skin and dense connective tissue layer
B. Between loose connective tissue layer and galea aponeurotica
C. Between galea aponeurotica and pericranium
D. Between pericranium and calvaria
E. At the subcutaneous layer
ANSWER: D.
 Rupture of the periosteal arteries resulting in a cephalohematoma is defi ned as a collection of blood
underneath the periosteum. On the head, it is located between the pericranium (periosteum of the skull) and
the calvaria (skull).
Q.5: A PATIENT WITH AN INJURY TO LATERAL ASPECT OF SKULL MAY HAVE SUFFERED AN
INTRACRANIAL HEMORRHAGE. HE SUSTAINED A BRIEF LOSS OF CONSCIOUSNESS, FOLLOWED BY
A LUCID PERIOD OF SOME HOURS, SUCCEEDED BY DROWSINESS OR COMA. THIS PRESENTATION IS
MOST CONSISTENT WITH:

A. Epidural (extradural) hematoma


B. Increased pressure in the ventricles
C. Intraparenchymal hemorrhage
D. Subarachnoid hematoma
E. Subdural hematoma
ANSWER: A.

Q.6: A MAN PRESENTED WITH THE COMPLAIN OF DEAFNESS. ON EVALUATION, IT WAS FOUND THAT
THE BLOOD SUPPLY TO MASTOID AIR CELLS & MASTOID ANTRUM HAS BEEN SEVERED. WHAT
BLOOD VESSEL DO YOU THINK IS DAMAGED:

A. Posterior auricular artery


B. Caroticotympanic artery
C. Ascending pharyngeal artery
D. Maxillary artery
E. Middle meningeal artery
ANSWER: A.

Q.7: A MOTORCYCLIST WAS ADMITTED IN EMERGENCY AFTER HAVING BLOW ON LATERAL


SIDE OF SKULL. IF THE INTRACRANIAL BLEEDING OCCURS FROM MIDDLE MENINGEAL
ARTERY, THE BLOOD WILL ACCUMULATE:

A. Between endocranium & dura mater


B. Between two laryers of dura mater
C. Between arachnoid & dura mater
D. Between pia & arachnoid mater
E. Between brain & pia mater
ANSWER: A.

Q.8: AN OLD MAN IS SUFFERING FROM NECK INFECTION. THE SOURCE OF INFECTION
LIES ANTERIOR TO PRETRACHEL FASCIA. INFECTION CAN SPREAD UP TO WHICH SPACE:

A. Anterior mediastinum
B. Inferior mediastinum
C. Middle mediastinum
D. Posterior mediastinum
E. Superior mediastinum

Head & Neck (4TH semester—Unit-I PRACTICAL MANUAL) Page 30


ANSWER: A.

Q.9: A 46 YEARS OLD MAN SUSTAINS A SPIDER BITE ON HIS UPPER EYELID AND AN
INFECTION DEVELOPED. THE PHYSICIAN IS VERY CONCERNED ABOUT SPREAD OF THE
INFECTION TO THE DURAL VENOUS SINUSES OF THE BRAIN VIA EMISSARY VEINS. WITH
WHICH OF THE FOLLOWING DURAL VENOUS SINUSES DOES THE SUPERIOR OPHTHALMIC
VEIN DIRECTLY COMMUNICATE:

A. Cavernous sinus
B. Occipital sinus
C. Sigmoid sinus
D. Superior petrosal sinus
E. Straight sinus

ANSWER: A.

VI. CASE STUDY


Case—1
A 46-YEAR-OLD MAN WAS SEEN IN THE EMERGENCY DEPARTMENT AFTER BEING
KNOCKED DOWN IN A STREET BRAWL. HE HAD RECEIVED A BLOW ON THE HEAD WITH AN
EMPTY BOTTLE. ON EXAMINATION, THE PATIENT WAS CONSCIOUS AND HAD A LARGE
DOUGHLIKE SWELLING OVER THE BACK OF THE HEAD THAT WAS RESTRICTED TO THE
AREA OVER THE OCCIPITAL BONE. THE SKIN WAS INTACT, AND THE SWELLING
FLUCTUATED ON PALPATION. THE HEMATOMA WAS LOCATED DEEP TO THE PERIOSTEUM
OF THE OCCIPITAL BONE.

1. What is the site of the hematoma?


ANS: The hematoma is is situated beneath the periosteum.

2. Why the hematoma is restricted to the occipital bone only and did not extend forward
to the orbital margins and laterally to the temporal lines?
ANS: The hematoma is is situated beneath the periosteum of the occipital bone. The edge of the
swelling is limited by the attachment of the periosteum to the sutural ligaments, hence the
hematoma is restricted to one skull bone.

3. How caput succedaneum can be clinically differentiated from subperiosteal


cephalohematoma?
ANS: Caput succedaneum is swelling of the scalp superficial soft tissue, which is a normal
response of the fetal head to the birth process. In this situation, the blood will cross over suture
lines. When a soft tissue mass seems contained by suture lines, then subperiosteal
cephalohematoma is suspected.

Case—2
A MALE INFANT WHO WEIGHS 3500-G APPEARS ICTERIC ON EXAMINATION. THE PREVIOUS
DAY, THE INFANT WAS DELIVERED VAGINALLY BY VACUUM-ASSISTED EXTRACTION
BECAUSE THERE WERE SEVERE FETAL HEART RATE DECELERATIONS. THE INFANT’S
SCALP HAS A 5-CM DISCOLORED SOFT TISSUE SWELLING THAT SEEMS TO BE CONTAINED
BY AND DOES NOT CROSS THE SAGITTAL OR LAMBDOIDAL SUTURES. THE MOTHER HAD
NO PRENATAL OR MEDICAL PROBLEMS. THERE IS NO FAMILY HISTORY OF BLEEDING
DISORDERS. MOST LIKELY DIAGNOSIS IS CAPUT SUCCEDENEUM.

1. What is capus succedeneum?

Head & Neck (4TH semester—Unit-I PRACTICAL MANUAL) Page 31


ANS: It is swelling of the scalp superficial soft tissue, which is a normal response of the fetal
head to the birth process. In this situation, the blood will cross over suture lines. When a soft tissue
mass seems contained by suture lines, then subperiosteal cephalohematoma is suspected.

2. What is the anatomical mechanism for the condition?


ANS: Injury to the branches of arteries supplying the lateral skull

Case—3
YOU HAVE BEEN ASKED TO ASSESS THE POST-OPERATIVE CONDITION OF A 65-YEAR-OLD
MAN WHO HAS BEEN SURGICALLY TREATED FOR CARCINOMA OF THE TONGUE. SINCE
THE TUMOR WAS IN ITS EARLY STAGES, THE SURGEON HAS PERFORMED A LEFT-SIDED
HEMIGLOSSECTOMY WITH BLOCK DISSECTION OF THE LEFT NECK. IN THIS OPERATION
ALL POSTERIOR TRIANGLE LYMPH NODES WERE REMOVED, ALONG WITH OTHER
STRUCTURES. THE PATIENT WAS RECOVERING WELL AND WAS ABLE TO MOVE.

1. Which important nerve is likely to be injured in posterior neck triangle operations? how
would you test for its integrity post-operatively?

ANS: The accessory nerve (XI) is likely to be injured in such an operation. Its injury in the posterior
triangle will cause paralysis of the trapezius & sternocleidomastoid muscles. The usual clinical tests
for assessing the integrity of the nerve is by asking the patient to shrug his shoulder (action of
trapezius) and to rotate the face to the opposite side (action of sternocleidomastoid) against
resistance.

2. What groups of lymph nodes might the surgeon remove from the posterior triangle?

ANS: Supraclavicular lymph nodes are relatively important, (they are part of the deep cervical
lymph nodes); other less important nodes are the occipital situated at the apex of the triangle.

MUSCLES & SPACES OF HEAD & NECK


I. MUSCLES OF THE NECK
1. Superficial posterolateral muscles (Fig—17)
i. Sternocleidomastoid
 ORIGIN: Sternal head arises from the anterior surface of the manubrium sterni, Clavicular
head arises from the medial 1/3 of the clavicle.
 INSERTION: Lateral half of the superior nuchal line & mastoid process.
 NERVE SUPPLY: The spinal part of the accessory nerve (CN XI)
 ACTIONS: It tilts the head towards the same shoulder. It rotates the head and face to the
opposite side. With the chest fixed, both the muscles act together from below to draw the head
forwards. With the head fixed, both the muscle assist in raising the thorax in forced
inspiration.

Head & Neck (4TH semester—Unit-I PRACTICAL MANUAL) Page 32


ii. Trapezius
 ORIGIN: Medial half of superior nuchal line, External occipital protuberance, Ligamentum
nuchae & Spinous processes of C7 to C12 vertebrae.
INSERTION: Lateral 1/3 of clavicle…Acromion….Spine of scapula.
NERVE SUPPLY: The spinal part of the accessory nerve (CN XI).
ACTIONS: Assists in rotating the scapula during abduction of humerus above horizontal.
2. Suprahyopid Muscles (Fig—17)
i. Mylohyoid
 ORIGIN: From Mylohyoid line of mandible.
 INSERTION: Into Mylohyoid raphe and body of hyoid.
 NERVE SUPPLY: Nerve to mylohyoid (from mandibular nerve, CN V3).
 ACTIONS: Elevates hyoid, floor of mouth, and tongue during swallowing and speaking.
ii. Geniohyoid
 ORIGIN: From Inferior genial spines of mandible.
 INSERTION: Hyoid bone.
 NERVE SUPPLY: C1 via hypoglossal nerve (CN XII).
 ACTIONS: Pulls hyoid anterosuperiorly; shortens floor of mouth & Widens the pharynx.
iii. Stylohyoid
 ORIGIN: Styloid process.
 INSERTION: Hyoid bone.
 NERVE SUPPLY: Stylohyoid branch of (preparotid) facial nerve (CN VII).
 ACTIONS: Elevates and retract hyoid, thus elongating floor of mouth
iv. Diagastric
 ORIGIN & INSERTION: Anterior belly: from digastric fossa of the mandible, Intermediate
tendon: each digastric muscle has two bellies, joined by an intermediate tendon, Posterior
belly: from mastoid notch of temporal bone.
 NERVE SUPPLY: Anterior belly: nerve to mylohyoid, a branch of inferior alveolar nerve &
Posterior belly: digastric branch (preparotid) of facial nerve (CN VII).
 ACTIONS: Working with infrahyoid muscles, it depresses mandible against resistance —
Elevates and steadies hyoid during swallowing and speaking.
3. Infrahyopid Muscles
i. Sternothyroid
 ORIGIN: From Posterior surface of the manubrium sterni.
 INSERTION: Into oblique line of the lamina of thyroid cartilage.
 NERVE SUPPLY: By ansa cervicalis.
 ACTIONS: Elevates hyoid, floor of mouth, and tongue during swallowing and
speaking.

ii. Sternohyoid
 ORIGIN: From posterior surface of the manubrium sterni.
 INSERTION: Into Lower border of the body of hyoid.
 NERVE SUPPLY: By ansa cervicalis.
 ACTIONS: Pulls hyoid anterosuperiorly; shortens floor of mouth & Widens the pharynx.
iii. Thyrohyoid
 ORIGIN: From the greater horn of the hyoid bone.
 INSERTION: Into Oblique line of the lamina of thyroid cartilage.
 NERVE SUPPLY: By branch from the hypoglossal nerve which contains C1 fibres.
 ACTIONS: Elevates and retrac hyoid, thus elongating floor of mouth.
iv. Omohyoid
 ORIGIN & INSERTION: Inferior belly: from upper border of scapula, Intermediate tendon: it
passes deep to the sternomastoid muscle & Superior belly: is inserted to the lateral part of
the hyoid bone.
 NERVE SUPPLY: Superior belly: by a branch of the superior root of ansa cervicalis &
Inferior belly: by a branch of the ansa cervicalis.

Head & Neck (4TH semester—Unit-I PRACTICAL MANUAL) Page 33


 ACTIONS: All infrahyoid muscles are the depressors of the larynx. The sternohyoid acts
directly on the thyroid cartilage and other three indirectly via the hyoid bone. By depressing
the larynx, the infrahyoid muscle increase the volume of the resonating chambers during
phonation, thus enhancing the quality of the voice.The infrahyoid muscle opposse the
elevators of the larynx enabling them to act progressively and gradually. The infrahoid
muscle prevent the ascent of the hyoid bone when digastric and gniohyoid lower the
mandible.

Fig-17: Sternocleidomastoid, Trapezius, Supra-and-infrahyoid muscles

4. Prevertebral muscles (Fig—18)


a. Anterior group
1. Longus colli 2. Longus capitis 3. Rectus capitis anterior 4. Scalenus anterior

i. Scalenus anterior:
 ORIGIN: Transverse processes of C4 to C6 vertebrae.
 INSERTION: Scalene tubercle of the 1st rib.
 NERVE SUPPLY: Cervical spinal nerves C4 to C6.
b. Lateral group
1. Rectus capitis lateralis 2. Splenius capitis 3. Levator scapulae 4. Scalenus medius 5. Scalenus posterior

i. Scalenus medius

Head & Neck (4TH semester—Unit-I PRACTICAL MANUAL) Page 34


 ORIGIN: Posterior tubercles of transverse processes of C4 to C6 vertebrae.
 INSERTION: Superior surface of 1st rib.
 NERVE SUPPLY: Anterior rami of cervical spinal nerves.
 ACTIONS: Flexes neck laterally; elevates 1st rib during forced inspiration.
ii. Scalenus posterior
 ORIGIN: Posterior tubercles of transverse processes of C4 to C6 vertebrae.
 INSERTION: External border of 2nd rib.
 NERVE SUPPLY: Anterior rami of cervical spinal nerves C7 and C8.
 ACTIONS: Flexes neck laterally; elevates 2nd rib during forced inspiration.

Fig-18: Prevertebral region and the root of the neck.

II. MUSCLES OF THE FACE (Fig—19)


 They are supplied by the facial nerve — The primary function of the facial muscles is to control
the organs guarding the orifices on the face — The varying expressions so produced on
the face are their side effects.
1.Muscles of the eyelids
a. Openers Of The Eyelids
i. Occipitofrontalis
 ORIGIN: Occiptalis Part: from the superior nuchal line & Frontalis Part: from the front of the
epicranial aponeurosis.

Head & Neck (4TH semester—Unit-I PRACTICAL MANUAL) Page 35


 INSERTION: Occipitalis Part: passes forward to be inserted into the epicranial aponeurosis &
Frontalis Part: passes forward to be attached to the upper part of the orbicularis oculi and the
skin overlying the eyebrows.
 NERVE SUPPLY: Temporal branch of the facial nerve to the frontalis, and Posterior auricular
branch to the occipitalis.
 ACTIONS: Frontalis elevates the eyebrows and produces wrinkles in the skin of forehead. It
gives an Expression Of Surprise And Horror – Occipitalis merely anchors epicranial
aponeurosis.
ii. Levator palpabrae superioris
b. Closers Of The Eyelids
i. Orbicularis oculi
 ORIGIN: Palpaberal Part: from the medial palpaberal ligament & Orbital Part: from the nasal
part of the fontal bone & the anterior lacrimal crest.
INSERTION: Palpaberal Part: It passes across eyelids, and interdigitate laterally to form the
lateral palpaberal raphe & Orbital Part: Circumscribe the orbital margin in a series of
concentric loops.
NERVE SUPPLY: Temporal and zygomatic (mainly) branches of the facial nerve.
ACTIONS: Contraction of the palpaberal fibers closes the eyelids gently without burying
the eyelashes, Orbital and palpaberal fibres contracting together close the eyelids forcibly
so that the eyelashes are buried — It helps to spread lacrimal secretion from the lacrimal
gland.
2. Muscles of the nostrils
a. Openers Of The Nostrils
1. Dilator naris (alar part of nasalis) 2. Depressor septi 3. Leavator labii superioris alaeque nasi
b. Closers Of The Nostrils

1. Comprressor naris (transverse part of nasalis)


3. Muscles of the lips
a. Openers Of The Lips
i. Acting on upper lip
1. Levator labii superioris alaeque nasi 2. Levator labii superioris 3. Zygomaticus minor & major
ii. Acting on angle of mouth
1. Levator anguli oris 2. Risorius 3. Depressor anguli oris
iii. Acting on lower lip
1. Depressor labii inferioris 2. Mentalis

Head & Neck (4TH semester—Unit-I PRACTICAL MANUAL) Page 36


Fig-19: Muscles of facial expression.

b. Closers Of The Lips


i. Orbicularis oris
 ORIGIN: Intrinsic Fibres are deepest of all the orbicularis fibres and are attached to maxillae
and the mandible near the midline and well away from the alveolar margin & Extrinsic Fibres
form bulk of the orbicularis oris, and most of them come from the buccinator muscle.
 INSERTION: Intrinsic Fibres are thin sheets that curve around the angle of the mouth in a
loop on either side & Extrinsic Fibres 1. The uppermost fibres coming from buccinator
pass straight on into the upper lip, 2. The lowermost fibers coming from buccinator pass
straight on into the lower lip & 3.The middle fibers coming from buccinator decussate in
such a way that the upper fibres pass on into the lower lip and the lower fibres pass on into the
upper lip.
 NERVE SUPPLY: Buccal and mandibular branches of the facial nerve.

Head & Neck (4TH semester—Unit-I PRACTICAL MANUAL) Page 37


 ACTIONS: Contraction of the muscle causes the narrowing of the mouth, and the lips
becoming pursed up into a smallest possible circle — It gives an EXPRESSION OF
WHISTLING.
ii. Buccinator
 ORIGIN: From Maxilla: opposite the molar teeth…From Mandible: from the oblique line on
the outer surface of the body of the mandible…From Pterygomaxillary Ligament: between
the tuberosity of maxilla and the pterygoid hamulus…From Ptreygomandibular Raphe:
between the pterygoid hamulus and the posterior end of the mylohyoid line.
INSERTION: Maxillary fibres: fibres pass straight on into the upper lip….Mandibular fibres:
fibres pass straight on into the lower lip…Raphe fibres: fibres decussate in such a way that
the upper fibres pass on into the lower lip and the lower fibers pass on into the upper lip…….all
these muscle fibres converge on the modiolous.
NERVE SUPPLY: Buccal branch of the facial nerve.
ACTIONS: It is essentially an accessory muscle of mastication, however, it is classified as muscle
of facial expression because of being supplied by the facial nerve — It is essential for returning the
bolus from the cheek pouch to the molar — When cheeks are inflated, the muscle is relaxed, but
when the air is forcibly expelled out of the mouth, the muscle contracts.
4. Muscles of the auricles
 1. Auricularis anterior 2. Auricularis posterior 3. Auricularis superioris
5. Other Muscles of the face
1. Corrugator supercilii 2. Procerus
III. MUSCLES OF THE MASTICATION (Fig—20)
i. Masseter
 ORIGIN: It arises from lower border of zygomatic arch.
 INSERTION: It is inserted in whole lateral surface of ramus of mandible.
 NERVE SUPPLY: Masseteric branch from anterior division of the mandibular nerve.
 ACTIONS: It elevates the mandible when open mouth is closed.
ii. Temporalis
 ORIGIN: It is a large fan-shaped muscle that fills much of temporal fossa…It arises from
temporal fossa over whole area between inferior temporal line and infratemporal crest —
 INSERTION: Fan shaped muscle converges to become a narrow tendon, that passes deep
to zygomatic arch, and is inserted on medial surface of coronoid process of mandible —
 NERVE SUPPLY: Two deep temporal branches from ant. division of mandibular nerve
 ACTIONS: Temporalis is a powerful elevator of mandible…..It retracts (specially its posterior
fibres) protruded mandible.
iii. Medial pterygoid
 ORIGIN: Larger Deep Head: arises from medial (deep) surface of lateral pterygoid plate &
Smaller Superficial Head: arises from post. surface of maxilla, at maxillary tuberosity.
 INSERTION: The two heads unite to be inserted into medial surface of the angle of mandible.
 NERVE SUPPLY: Nerve to medial pterygoid, a branch from main trunk of mandibular nerve
 ACTIONS: It elevates mandible to close mouth — It moves mandible towards opposite side in
chewing.
iv. Lateral pterygoid
 ORIGIN: Superior Head: arises from the infratemporal surface of the greater wing of
sphenoid & Inferior Head: arises from the lateral surface of the lateral pterygoid plate.
 INSERTION: The two heads fuse to form a common tendon which is inserted into the
pterygoid fovea at the anterior surface of the neck of the mandible — Upper fibres are attached
to the capsule and the articular disc of TMJ.
 NERVE SUPPLY: Nerve to lateral pterygoid, a branch from the anterior division of
mandibular nerve.
 ACTIONS: It pull the neck of mandible and the disc of TMJ forward from the mandibular
fossa down the slop of articular eminence, which is essential for active opening of mouth.

Head & Neck (4TH semester—Unit-I PRACTICAL MANUAL) Page 38


Fig-20: Muscles of mastication
IV. TRIANGLES OF THE NECK
1. Posterior triangle of neck (Fig—21)
 It is a depressed area enclosed between the sternocleidomastoid anteriorly and the
trapezius posteriorly.
a. Boundaries
 ANTERIORLY: Posterior border of the sternocleidomastoid muscle — POSTERIORLY:
Anterior border of the trapezius muscle — BASE: Middle 1/3 of clavicle — APEX: Mid-point
of the superior nuchal line, where the anterior and posterior borders meet — ROOF: Investing
layer of deep cervical fascia, supeficial fascia and skin (from deep to superficial) — FLOOR:
Prevertebral muscles.
b. Contents
 ARTERIES: 3RD part of subclavian artery, Superficial cervical and Suprascapular arteries
(branches of the thyrocervical trunk) & Occipital artery, one of the branches of the external
carotid artery — VEINS: The terminal part of the External jugular vein — LYMPH NODES:
Superficial cervical lymph nodes — NERVES: Spinal part of Accesory nerve (CN XI) that
passes across the triangle, being superficial to the prevertebral fascia, Cutaneous nerve from
Cervical plexus (Lesser occipital nerve, Great auricular nerve, Transverse
cervical nerve and Supraclavicular nerves) & Trunks of Brachial plexus (Superior, Middle
and Inferior trunks)
2. Suboccipital triangle of neck (Fig—22)
 It is a region in the upper part of the back of the neck.
a. Boundaries
 The floor is formed by the posterior occipito-atlantal membrane, and the posterior arch of the
atlas vertebra.
b. Contents
 In the deep groove on the upper surface of the posterior arch of the atlas are the 3RD part of the
vertebral artery.

Head & Neck (4TH semester—Unit-I PRACTICAL MANUAL) Page 39


Fig-21: Posterior triangle of the neck

Fig-22: Suboccipital triangle of the neck

Head & Neck (4TH semester—Unit-I PRACTICAL MANUAL) Page 40


3. Anterior triangle of the neck (Fig—23)
 It is a depressed area enclosed between the sternocleidomastoid posteriorly and the
midline of the neck anteriorly.
 It is:
o Bounded anteriorly by the midline of the neck from chin to the jugular notch
o Bounded posteriorly by the anterior border of the sternocleidomastoid muscle
o Based by inferior border of the body of mandible.
 The anterior triangle is divided into four smaller triangles:

1.Digastric Triangle
a. Boundaries
 ANTERIORLY: anterior belly of digastric — POSTERIORLY: posterior belly of digastric —
BASE: Lower boder of body of mandible.
b. Contents
 ARTERIES: Facial artery, that passes deep to the submandibular gland — VEINS: Facial vein,
that passes superficial to the submandibular gland — LYMPH NODES: Submandibular lymph
nodes — NERVES: Marginal mandibular and cervical branches of the facial nerve —
GLANDS: Superficial part of the submandibular gland & Lower pole of the parotid gland.
2.Submental Triangle
a. Boundaries
 ANTERIORLY: Midline — POSTERIORLY: anterior belly of digastric — BASE: Hyoid bone.
b. Contents
 VEINS: Submental vein, that continues into the anterior jugular vein — LYMPH NODES:
Submental group of superficial cervical lymph nodes.
3.Carotid Triangle
a. Boundaries
 SUPERIORLY: anterior belly of digastric — INFERIORLY: superior belly of omohyoid — BASE:
anterior border of sternocleidomastoid muscle.
b. Contents
 ARTERIES: Common carotid artery enclosed within the carotid sheath & External and
internal carotid arteries — VEINS: Internal jugular vein enclosed within the carotid sheath —
LYMPH NODES: Deep cervical lymph nodes enclosed within the carotid sheath — NERVES:
Ansa cervicalis embedded in the anterior facial wall of the carotid sheath.
4. Muscular Triangle
a. Boundaries
 ANTERIORLY: midline — POSTERIORLY: superior belly of omohyoid — BASE: anterior
border of sternocleidomastoid muscle.
b. Contents
 Anterior jugular vein.

Head & Neck (4TH semester—Unit-I PRACTICAL MANUAL) Page 41


Fig-23: Anterior triangle of the neck
V.CLINICAL CORRELATIONS
1. Sternocleidomastoid muscle and protection from
trauma
 Suicide attempts by cutting one's throat often fail because the individual first extends the neck
before making several horizontal cuts with a knife.
 Extension of the cervical part of the vertebral column and extension of the head at the atlanto-
occipital joint cause the carotid sheath with its contained large blood vessels to slide
posteriorly beneath the sternocleidomastoid muscle.
 To achieve the desired result with the head and neck fully extended, some individuals have to
make several attempts and only succeed when the larynx and the greater part of the
sternocleidomastoid muscles have been severed.
2. Congenital torticollis
 Most cases of congenital torticollis are a result of excessive stretching of the
sternocleidomastoid muscle during a difficult labor.

Head & Neck (4TH semester—Unit-I PRACTICAL MANUAL) Page 42


 Hemorrhage occurs into the muscle and may be detected as a small, rounded tumor during the
early weeks after birth. Later, this becomes invaded by fibrous tissue, which contracts and
shortens the muscle.
 The mastoid process is thus pulled down toward the sternoclavicular joint of the same side,
the cervical spine is flexed, and the face looks upward to the opposite side.
 If left untreated, asymmetrical growth changes occur in the face, and the cervical vertebrae
may become wedge shaped.
3. Spasmodic torticollis
 Spasmodic torticollis, which results from repeated chronic contractions of the
sternocleidomastoid and trapezius muscles, is usually psychogenic in origin. Section of the
spinal part of the accessory nerve may be necessary in severe cases.
4. Facial muscle paralysis
 The facial muscles are innervated by the facial nerve.
 Damage to the facial nerve in the internal acoustic meatus (by a tumor), in the middle ear
(by infection or operation), in the facial canal (Bell's palsy), or in the parotid gland (by a tumor)
or caused by lacerations of the face will cause distortion of the face, with drooping of the lower
eyelid, and the angle of the mouth will sag on the affected side.
 This is essentially a lower motor neuron lesion.
 An upper motor neuron lesion (involvement of the pyramidal tracts) will leave the upper part of
the face normal because the neurons supplying this part of the face receive corticobulbar fibers
from both cerebral cortices.
5. Clinical identification of the platysma
 The platysma can be seen as a thin sheet of muscle just beneath the skin by having the
patient clench his or her jaws firmly.
 The muscle extends from the body of the mandible downward over the clavicle onto the
anterior chest wall.
6. Platysma tone and neck incisions
 In lacerations or surgical incisions in the neck it is very important that the subcutaneous layer with
the platysma be carefully sutured, since the tone of the platysma can pull on the scar tissue,
resulting in broad, unsightly scars.
7. Platysma innervation, mouth distortion, and neck
incisions
 The platysma muscle is innervated by the cervical branch of the facial nerve.
 This nerve emerges from the lower end of the parotid gland and travels forward to the
platysma; it then sometimes crosses the lower border of the mandible to supply the depressor
anguli oris muscle.
 Skin lacerations over the mandible or upper part of the neck may distort the shape of the mouth.

VI.REVIEW QUESTIONS
Q.1: A 32-YEAR-OLD WOMAN IS ADMITTED TO THE HOSPITAL WITH HEADACHES AND
DIZZINESS. DURING PHYSICAL EXAMINATION IT IS NOTED THAT THE PATIENT HAS
PARTIAL PTOSIS (DROOPING EYELID). WHICH OF THE FOLLOWING MUSCLES IS MOST
LIKELY PARALYZED?
A. Orbicularis oculi, lacrimal part
B. Orbicularis oculi, palpebral part
C. Levator palpebrae superioris
D. Superior oblique
E. Superior tarsal (of Müller)
ANSWER: E.
 The superior tarsal muscle (of Müller), innervated by sympathetics, assists in elevating the eyelids
and holding them up. Damage would result in partial ptosis of the eyelid

Head & Neck (4TH semester—Unit-I PRACTICAL MANUAL) Page 43


Q.2: AN 8-YEAR-OLD BOY HAD AN EXTENSIVE MASTOIDECTOMY DUE TO AN INFECTION
THAT DID NOT RESPOND TO ANTIBIOTICS. POSTOPERATIVELY HE HAD BELL’S PALSY
(FACIAL PARALYSIS), AND ONE OF THE FEATURES WAS THAT SALIVA TENDED TO
ACCUMULATE IN THE VESTIBULE OF HIS ORAL CAVITY AND DRIBBLE FROM THE CORNER
OF HIS MOUTH. WHICH OF THE FOLLOWING MUSCLES WAS PARALYZED TO ALLOW THIS
CONDITION TO OCCUR?
A. Zygomoaticus major
B. Orbicularis oculi
C. Buccinator
D. Levator palpebrae superioris
E. Orbicularis oris
ANSWER: C.
 Normally the tonus of the buccinator muscle prevents the accumulation of saliva and foodstuffs in
the oral vestibule. Although a lesion of the facial nerve would paralyze the other muscles listed,
the buccinator is the most important muscle of the cheek.
Q.3: A 65-YEAR-OLD MAN IS ADMITTED TO THE EMERGENCY DEPARTMENT AFTER HIS
HEAD HIT THE DASHBOARD IN AN AUTOMOBILE COLLISION. RADIOGRAPHIC AND
PHYSICAL EXAMINATIONS REVEAL THAT THE INFERIOR ALVEOLAR NERVE IS INJURED AT
ITS ORIGIN. WHICH OF THE FOLLOWING MUSCLES WOULD MOST LIKELY BE PARALYZED
AS A RESULT?
A. Geniohyoid
B. Hyoglossus
C. Mylohyoid
D. Stylohyoid
E. Palatoglossus
ANSWER: C.
 Just before it passes into the mandible to supply the lower teeth and chin, the inferior alveolar
nerve gives rise to the mylohyoid nerve, a motor nerve supplying the mylohyoid and anterior belly
of the digastric.
Q.4: A 27-YEAR-OLD MAN IS ADMITTED TO THE HOSPITAL AFTER A MIDDLEWEIGHT BOXING MATCH.
DURING PHYSICAL EXAMINATION THE STRENGTH AND SYMMETRY OF STRENGTH IN OPENING THE
JAWS ARE TESTED. WHICH OF THE FOLLOWING MUSCLES IS THE MOST IMPORTANT IN JAW
PROTRUSION AND DEPRESSING THE MANDIBLE?
A. A nterior portion of temporalis
B. L ateral pterygoid
C. M edial pterygoid
D. Masseter
E. Platysma
ANSWER: B.
 The lateral pterygoid muscle is a muscle of mastication innervated by the lateral pterygoid nerve of the
mandibular division of the trigeminal nerve. The lateral pterygoid acts to protrude the mandible and open the
jaw.
Q.5: A MEDICAL STUDENT IS STUDYING THE POSTERIOR TRIANGLE OF NECK IN A CADEVER
SPECIMEN. SHE IS LIKELY TO FOUND ALL OF THE FOLLOWING EXCEPT:

A. Spinal accessory nerve


B. Trunks of brachial plexus
C. Superior belly of omohyoid
D. Supra-clavicular nerves
E. Transverse cervical artery

ANSWER: C.

Q.6: A 36-YEAR OLD WOMAN IS DIAGNOSED AS CARCINOMA OF PAROTID GLAND. DURING


PAROTIDECTOMY THE FACIAL NERVE IS SACRIFICED. FOLLOWING STRUCTURE WILL LOSE ITS
INNERVATION:

A. Buccinator
B. Mylohyoid
C. Parotid gland
D. Anterior belly of digastric
E. Cornea

ANSWER: A.

Head & Neck (4TH semester—Unit-I PRACTICAL MANUAL) Page 44


Q.7: A CT SCAN OF A 17-YEAR-OLD GIRL SHOWS A TUMOR COMPRESSING THE
MANDIBULAR BRANCH OF TRIGEMINAL NERVE. ALL OF THE FOLLOWING MUSCLES MAY
BE PARALYSED EXCEPT:

A. Masseter
B. Buccinator
C. Tensor tympani
D. Mylohyoid
E. Anterior belly of digastrics

ANSWER: B.

Q.8: A PERSON IS UNABLE TO SUCK LIQUIDS THROUGH A STRAW. THIS MAY BE DUE TO
LESION OF:

A. Mandibular nerve
B. Lingual nerve
C. Gloss pharyngeal nerve
D. Facial nerve
E. None of above
ANSWER: D.

Q.9: A 28-YEAR-OLD WOMAN IS INVOLVED IN AN AUTOMOBILE ACCIDENT. SHE IS NOT


WEARING A SEAT BELT, AND FALLS THROUGH THE WINDSHIELD WHEN HER CAR HITS A
TREE. AFTER THE ACCIDENT, SHE HAS DIFFICULTY OPENING HER MOUTH, BUT HAS NO
DIFFICULTY CLOSING IT. WHICH OF THE FOLLOWING MUSCLES WAS MOST LIKELY
INJURED:

A. Lateral pterygoid
B. Masseter
C. Medial pterygoid
D. Orbicularis oris
E. Temporalis
ANSWER: A.

Q.10: AN ANGIOGRAPHIC STUDY OF THE DISTAL PART OF THE CIRCLE OF WILLIS


REQUIRES ACCESS TO THE VERTEBRAL ARTERY. ACCESS TO THIS ARTERY CAN BE
OBTAINED VIA THE:

A. Anterior triangle of the neck


B. Muscular triangle of the neck
C. Posterior triangle of the neck
D. Submental triangle
E. Subccipital triangle
ANSWER: E.

Q.11: CONTRACTION OF WHICH OF THE FOLLOWING MUSCLES CONTRIBUTES MOST TO


THE BACKWARD MOVEMENT OF THE LOWER JAW DURING THE PROCESS OF
MASTICATION:

A. Digastrics
B. Lateral pterygoid
C. Medial pterygoid
D. Mylohyoid
E. Temporalis
ANSWER: E.

Q.12: IF A PATIENT HAS NO CUTANEOUS SENSATION ON THE ANTERIOR CERVICAL


TRIANGLE, IT IS LIKELY THAT DAMAGE HAS OCCURRED TO WHICH OF THE FOLLOWING
NERVES:

Head & Neck (4TH semester—Unit-I PRACTICAL MANUAL) Page 45


A. Phrenic nerve
B. Greater auricular nerve
C. Transverse cervical nerve
D. Superclavicular nerve
E. Lesser occipital nerve.
ANSWER: C.

VII.CASE STUDY
Case—1
A 7-YEAR-OLD BOY WITH RIGHT-SIDED OTITIS MEDIA WAS TREATED WITH ANTIBIOTICS.
THE ORGANISMS DID NOT RESPOND TO THE TREATMENT, AND THE INFECTION SPREAD
TO THE MASTOID ANTRUM AND THE MASTOID AIR CELLS. THE SURGEON DECIDED TO
PERFORM A RADICAL MASTOID OPERATION. AFTER THE OPERATION, IT WAS NOTICED
THAT HIS MOUTH WAS DRAWN UPWARD TO THE LEFT, HE WAS UNABLE TO CLOSE HIS
RIGHT EYE & SALIVA TENDED TO ACCUMULATE IN HIS RIGHT CHEEK. THE DIAGNOSIS
WAS RIGHT FACIAL NERVE DAMAGE.

1. Why the mouth was drawn upward to the left?


ANS: The facial muscles on the left side of the mouth on contraction pull the mouth upward and to
the left because the muscles on the right side were paralyzed.

2. Why he was unable to close his right eye?


ANS: Due to paralysis of right orbicularis oculi.

3. Why saliva tended to accumulate in his right cheek?


ANS: Due to paralysis of right buccinator muscle.

Case—2
A 3-MONTH-OLD GIRL IS NOTED BY THE PEDIATRICIAN TO HAVE A STIFF NECK FOR A 2-
MONTH DURATION. THE MOTHER STATES THAT THE NECK SEEMS TO BE PULLED TO THE
RIGHT. ON EXAMINATION, THE BABY’S RIGHT EAR IS TILTED TOWARD HER RIGHT SIDE,
BUT THE BABY’S FACE IS TURNED TOWARD THE LEFT. PALPATION OF THE NECK
REVEALS A NONTENDER MASS OF THE RIGHT ANTERIOR NECK REGION. THE MOST
LIKELY DIAGNOSIS IS TORTICOLIS.

1. Which anatomical structure is affected? what is its nerve supply?


ANS: Sternocleidomastoid — Spinal part of accessory nerve
2. What is torticolis?
ANS:
3. What is the most likely cause of torticolis?
ANS: The etiology is unclear, although it may be associated with breech babies or difficult deliveries.
The result is that the baby’s head is flexed laterally toward the affected side and rotated
contralaterally. Facial asymmetry may be noted.

Head & Neck (4TH semester—Unit-I PRACTICAL MANUAL) Page 46


GLANDS OF THE HEAD & NECK
I.PITUITARY GLAND (Fig—24)
1. Location
 The gland is located in the sella turcica in the body of the sphenoid bone.
2. Lobes
a. Anterior lobe or adenohypophysis
 It is subdivided into the: I. PARS ANTERIOR (SOMETIMES CALLED THE PARS DISTALIS),
II. PARS INTERMEDIA: It may be separated by a cleft that is remnant of embryonic pouch &
III. PARS TUBERALIS: It extends up along anterior and lateral surfaces of the pituitary stalk.
b. Posterior lobe, or neurohypophysis
3. Relations
 ANTERIORLY: Sphenoid sinus — POSTERIORLY: Dorsum sellae, the basilar artery, and the
pons — SUPERIORLY: Diaphragma sellae that separates the anterior lobe from the optic
chiasma — INFERIORLY: Body of the sphenoid, with its sphenoid air sinuse — LATERALLY:
Cavernous sinus and its contents.

Fig-24: Pitutary gland

Head & Neck (4TH semester—Unit-I PRACTICAL MANUAL) Page 47


II. THYROID GLAND (Fig—25)
 It consists primarily of right and left lobes & isthmus.
1.Lateral lobes
a. Location
 Each lateral lobe has an apex extending upto the lamina of the thyroid cartilage, and base
extending down to the level of 5TH or 6TH tracheal ring.
b. Relations
 They lie against the lateral sides of the larynx and upper trachea and with the pharynx and
upper esophagus — In the groove between trachea and esophagus, lie the recurrent
laryngeal nerve & Inferior thyroid artery.
2.Isthmus
a. Location
 The isthmus joins the two lateral lobes — The posterior surface of the isthmus is firmly
adherent to the 2ND, 3RD & 4TH tracheal rings, and the pretracheal fascia is here fixed between
them.
b. Movement with swallowing
 The fixation of the pretracheal fascia is responsible for the gland moving up and down with the
larynx during swallowing.
3.Pyramidal lobe
a. Location
 A small portion of gland substance often projects upwards from the isthmus, generally to the left
of midline, as the pyramidal lobe.
b. Levator glandulae throidae
 Pyramidal lobe may be attached to the inferior border of the hyoid bone by a fibrous tissue.
Sometimes a muscle develops in this fibrous tissue, called the Levator glandulae thyroidae.

Head & Neck (4TH semester—Unit-I PRACTICAL MANUAL) Page 48


Fig-25: The blood supply and venous drainage of the thyroid gland.

Head & Neck (4TH semester—Unit-I PRACTICAL MANUAL) Page 49


III. SUBMANDIBULAR GLAND (Fig—26)
 It is both serous and mucous type— It consists of large superficial part and a small deep part
which are continuous with each other by rounding posterior free margin of mylohyoid muscle.
1.Superficial part
a. Relations of lateral surface
 It is deeply grooved by facial artery
b. Relations of inferior surface
 It is covered by skin, superficial fascia embedded with platysma and investing layer of deep
cervical fascia — It is crossed by facial vein and cervical branch of the facial nerve and
sometimes marginal mandibular branch of facial nerve — Submandibular lymph nodes either
lie in contact with this surface r within its substance.

2.Deep part
Relations
 Lingual nerve (attached with the submandibular ganglion) lies above it and hypoglossal nerve
below.
3.Submandibular duct
Site of opening
 The submandibular duct opens in the floor of oral cavity beside the base of frenulum of the
tongue.

IV. SUBLINGUAL GLAND (Fig—26)


 The sublingual glands are the smallest and most deeply situated of the salivary glands— Each
gland lies in the floor of the mouth — Numerous small sublingual ducts open into the floor of
the mouth along the sublingual folds.

Head & Neck (4TH semester—Unit-I PRACTICAL MANUAL) Page 50


Fig-26: Submandibular and sublingual salivary glands (lateral view). B. Coronal section
through the superficial and deep parts of the submandibular salivary glands. C. Coronal
section (anterior to B) through the sublingual salivary glands and the ducts of the
submandibular salivary glands.

Head & Neck (4TH semester—Unit-I PRACTICAL MANUAL) Page 51


V. PAROTID GLAND (Fig—27)
 It is the largest of all salivary glands. It is mainly a serous gland, with few mucous acini.
 1. Lobes & processes:
 The facial nerve and its branches passes forward within the parotid gland and divide it into
superficial and deep parts.
 2. Capsule:
 In addition, the gland is enclosed in a dense fibrous capsule derived from the Investing layer of
deep cervical fascia.
 3. Parotid duct:
 It turns sharply medially and pierces the buccal pad of fat and buccinator muscle — It finally
opens into the vestibule of the mouth opposite the upper 2ND molar tooth.

 4. Structures within the parotid gland:


 1. Facial nerve 2. Retromandibular vein 3. External carotid artery 4. Parotid lymph
nodes
VI.CLINICAL CORRELATIONS
1. Swellings of thyroid gland and movement on
swallowing
 The thyroid gland is invested in a sheath derived from the pretracheal fascia. This tethers the
gland to the larynx and the trachea and explains why the thyroid gland follows the movements of
the larynx in swallowing.
2. Retrosternal goiter
 The attachment of the sternothyroid muscles to the thyroid cartilage limits upward expansion
of the gland.
 There being no limitation to downward expansion, it is not uncommon for a pathologically
enlarged thyroid gland to extend downward behind the sternum.
 A retrosternal goiter (any abnormal enlargement of the thyroid gland) can compress the trachea
and cause dangerous dyspnea; it can also cause severe venous compression.
3. Thyroid arteries and important nerves
 The superior thyroid artery on each side is related to the external laryngeal nerve, which
supplies the cricothyroid muscle.
 The terminal branches of the inferior thyroid artery on each side are related to the recurrent
laryngeal nerve.
 Damage to the external laryngeal nerve results in an inability to tense the vocal folds and in
hoarseness.
4. Thyroidectomy And The Parathyroid Glands
 In partial thyroidectomy, the posterior part of the thyroid gland is left undisturbed so that the
parathyroid glands are not damaged.
 The development of the inferior parathyroid glands is closely associated with the thymus.
 For this reason it is not uncommon for the surgeon to find the inferior parathyroid glands in the
superior mediastinum because they have been pulled down into the thorax
5. Calculus formation in submandibular
 The submandibular salivary gland is a common site of calculus formation.
 This condition is rare in the other salivary glands.
 The presence of a tense swelling below the body of the mandible, which is greatest before or
during a meal and is reduced in size or absent between meals, is diagnostic of the condition.
 Examination of the floor of the mouth will reveal absence of ejection of saliva from the orifice of
the duct of the affected gland. Frequently, the stone can be palpated in the duct, which lies below
the mucous membrane of the floor of the mouth.

Head & Neck (4TH semester—Unit-I PRACTICAL MANUAL) Page 52


6. Enlargement of the submandibular lymph nodes and
swelling of the submandibular salivary gland
 The submandibular lymph nodes are commonly enlarged as a result of a pathologic condition of the
scalp, face, maxillary sinus, or mouth cavity.
 One of the most common causes of painful enlargement of these nodes is acute infection of the
teeth.
 Enlargement of these nodes should not be confused with pathologic swelling of the
submandibular salivary gland.
7. Sublingual salivary gland and cyst formation
 The sublingual salivary gland, which lies beneath the sublingual fold of the floor of the mouth,
opens into the mouth by numerous small ducts.
 Blockage of one of these ducts is believed to be the cause of cysts under the tongue.
8. Parotid duct injury
 The parotid duct, which is a comparatively superficial structure on the face, may be
damaged in injuries to the face or may be inadvertently cut during surgical operations on the
face.
9. Parotid salivary gland and lesions of the facial nerve
 The parotid salivary gland consists essentially of superficial and deep parts, and the important
facial nerve lies in the interval between these parts.
 A benign parotid neoplasm rarely, if ever, causes facial palsy.
 A malignant tumor of the parotid is usually highly invasive and quickly involves the facial
nerve, causing unilateral facial paralysis.
10. Parotid gland infections
 The parotid gland may become acutely inflamed as a result of retrograde bacterial infection from
the mouth via the parotid duct.
 The gland may also become infected via the bloodstream, as in mumps.
 In both cases the gland is swollen; it is painful because the fascial capsule derived from the
investing layer of deep cervical fascia is strong and limits the swelling of the gland.
 The swollen glenoid process, which extends medially behind the temporomandibular joint, is
responsible for the pain experienced in acute parotitis when eating.
11. Frey's syndrome
 Frey's syndrome is an interesting complication that sometimes develops after penetrating wounds
of the parotid gland.
 When the patient eats, beads of perspiration appear on the skin covering the parotid.
 This condition is caused by damage to the auriculotemporal and great auricular nerves.
 During the process of healing, the parasympathetic secretomotor fibers in the
auriculotemporal nerve grow out and join the distal end of the great auricular nerve.
 Eventually, these fibers reach the sweat glands in the facial skin. By this means, a stimulus
intended for saliva production produces sweat secretion instead.

Head & Neck (4TH semester—Unit-I PRACTICAL MANUAL) Page 53


Fig-27: Parotid gland and its relations. A. Lateral surface of the gland and the course of the
parotid duct. B. Horizontal section of the parotid gland.

Head & Neck (4TH semester—Unit-I PRACTICAL MANUAL) Page 54


VII.REVIEW QUESTIONS
Q.1: A 15-YEAR-OLD MALE IS ADMITTED TO THE EMERGENCY DEPARTMENT WITH SEVERE
HEADACHE AND HYDROCEPHALUS. RADIOGRAPHIC EXAMINATION REVEALS A
CRANIOPHARYNGIOMA OCCUPYING THE SELLA TURCICA, PRIMARILY INVOLVING THE
SUPRASELLAR SPACE. WHICH OF THE FOLLOWING IS THE MOST LIKELY CAUSE OF THIS
TUMOR:
A. Persistence of a small portion of the Rathke pouch
B. Abnormal development of pars tuberalis
C. Abnormal development of foramina of Monro
D. Abnormal development of the alar plates that form the lateral wall of diencephalon
E. Abnormal development of diencephalon
ANSWER A.
 During embryologic development of the pituitary gland, an outgrowth from the roof of the pharynx
(Rathke’s pouch) grows cephalad and comprises the anterior lobe (pars distalis) of the pituitary
gland. Since this gland normally occupies the sella turcica, it is most likely a tumor derived from
the Rathke’s pouch that is extending up into the sella turcica and the space just above it, the
suprasellar space.
Q.2: A 32-YEAR-OLD FEMALE PATIENT ASKS YOU WHAT IS THE SOFT, THIN RIDGE OF
TISSUE THAT SHE CAN FEEL RUNNING FORWARD ACROSS THE MASSETER MUSCLE
TOWARD HER UPPER LIP. YOU REASSURE HER THAT IS PERFECTLY NORMAL. WHICH OF
THE FOLLOWING IS THE MOST LIKELY STRUCTURE SHE IS FEELING?
A. Facial artery
B. Maxillary artery
C. Parotid duct
D. Marginal mandibular branch of facial nerve
E. Facial vein
ANSWER: C.
 The parotid duct, also known as the Stensen’s duct, crosses the masseter muscle transversely
and extends to the oral cavity.
Q.3: A 35-YEAR-OLD WOMAN IS ADMITTED TO THE EMERGENCY DEPARTMENT AFTER A
VIOLENT AUTOMOBILE CRASH. THE PATIENT’S UPPER AIRWAY IS OBSTRUCTED WITH
BLOOD AND MUCUS, AND A MIDLINE TRACHEOTOMY INFERIOR TO THE THYROID ISTHMUS
IS PERFORMED. WHICH OF THE FOLLOWING VESSELS ARE MOST LIKELY TO BE PRESENT
AT THE SITE OF INCISION AND WILL NEED TO BE CAUTERIZED?
A. Middle thyroid vein and inferior thyroid artery
B. Inferior thyroid artery and inferior thyroid vein
C. Inferior thyroid vein and thyroidea ima artery
D. ricothyroid artery and inferior thyroid vein
E. Left brachiocephalic vein and inferior thyroid artery
ANSWER: C.
 The most likely structures one would encounter while performing a midline incision below the
isthmus of the thyroid gland would be the inferior thyroid vein and the thyroidea ima artery. The
inferior thyroid vein drains typically to the left brachiocephalic vein, which crosses superficially,
just inferior to the isthmus. The thyroidea ima artery arises from the aortic arch, vertebral artery,
or other source but is not a constant structure.
Q.4: RECURRENT LARYNGEAL NERVES ARE LIKELY TO BE DAMAGED DURING AN
OPERATION ON THYROID GLAND. FOLLOWING IS A TRUE FACT ABOUT THIS NERVE:

A. Arises from tenth cranial nerve in the jugular foramen


B. Are so named because they curve around larynx
C. Supply all muscles of the vocal cords
D. If damaged on one side the vocal fords are in cadaveric position
E. Remain safe if inferior thyroid arteries are ligated.
ANSWER B.

Head & Neck (4TH semester—Unit-I PRACTICAL MANUAL) Page 55


Q.5: A 29-YEAR-OLD FEMALE UNDERGOES THYROIDECTOMY TO REMOVE A TUMOR.
DURING THE SURGERY, AN ADJACENT NERVE IS INJURED. WHICH OF THE FOLLOWING
SYMPTOMS WOULD MOST LIKELY RESULT FROM SUCH DAMAGE:

A. Difficulty opening the mouth


B. Difficulty swallowing
C. Hoarseness of the voice
D. Inability to turn the head
E. Increased heart rate.
ANSWER C.

Q.6: A MAN PRESENTED WITH SWELLING ON NECK, SWELLING WAS PRESENT IN FRONT
OF THE PRETRACHEAL FASCIA. THE MOST LIKELY SITE FOR SPREAD OF INFECTION IS:

A. Anterior mediastinum
B. Posterior mediastinum
C. Superior mediastinum
D. Middle mediastimum
E. Prevertebral space
ANSWER A.

Q.7: A 32-YEAR-OLD MALE WITH OLIGOSPERMIA HAS A CHILDHOOD HISTORY OF FEVER


ACCOMPANYING PAINFUL SWELLING OF HE PAROTID GLAND & RIGHT TESTICLE. THE
MOST LIKELY ETIOLOGY OF THIS CONDITION IS:

A. Mumps
B. Herpes simplex
C. Varicella zoster
D. Cytomegalovirus
E. Influenza
ANSWER A.

Q.8: A MAN IS INVOLVED IN AN AUTOMOBILE ACCIDENT AND SUSTAINS TRAUMA TO THE


MIDDLE EAR, DAMAGING THE FACIAL NERVE FIBERS THAT TRAVEL IN THE CHORDA
TYMPANI. SALIVATION FROM WHICH OF THE FOLLOWING WOULD BE IMPAIRED:

A. Parotid and sublingual glands


B. Parotid and submandibular glands
C. Parotid gland only
D. Sublingual gland only
E. Submandibular and sublingual glands
ANSWER E.

Q.9: WHILE PERFORMING A SUBTOTAL THYROIDECTOMY, A SURGEON INADVERTENTLY


SECTIONS THE RECURRENT LARYONGEAL NERVE. WHICH OF THE FOLLOWING MUSCLES
WOULD RETAIN ITS INNERVATION SUBSEQUENT TO THIS INJURY:

A. Cricothyroid
B. Lateral cricoarytenoid
C. Posterior cricoarytenoid
D. Thyroarytenoid
E. Vocalis
ANSWER A.

Q.10: IN EVALUATING THE THYROID GLAND ON PHYSICAL EXAMINATION, A PHYSICIAN


STANDS BEHIND THE PATIENT AND PALPATES THE ANTERIOR NECK WITH THE PADS OF
HIS FINGERS. AFTER PALPATION, THE PHYSICIAN THEN PALPATES THE GLAND AS THE
PATIENT SWALLOWS. A THYROID MASS USUALLY MOVES WITH SWALLOWING BECAUSE
THE THYROID GLAND IS ENCLOSED BY WHICH OF THE FOLLOWING FASCIA:

Head & Neck (4TH semester—Unit-I PRACTICAL MANUAL) Page 56


A. Carotid sheath
B. Investing layer of the deep cervical fascia
C. Pretracheal fascia
D. Prevertebral fascia
E. Superficial fascia
ANSWER C.

Q.11: DAMAGE TO THE EXTERNAL LARYNGEAL NERVE DURING THYROID SURGERY COULD RESULT
IN THE INABILITY TO:

A. Relax the vocal cords.


B. Rotate the arytenoids cartilages.
C. Tense the vocal cords.
D. Widen the rima glottidis.
E. Abduct the vocal cords.
ANSWER C.

Q.12: ON LIGATION OF THE SUPERIOR LARYNGEAL ARTERY, CARE MUST BE TAKEN TO AVOID
INJURY TO WHICH OF THE FOLLOWING NERVES:

A. External laryngeal nerve.


B. Internal laryngeal nerve.
C. Superior laryngeal nerve.
D. Hypoglossal nerve.
E. Vagus nerve.
ANSWER A.

Q.13: A PATIENT WITH A PITUITARY TUMOR WOULD EXHIBIT WHICH OF THE FOLLOWING
DISORDERS:

A. Blindness
B. Bitemporal (heteronymous) hemianopia
C. Right nasal hemianopia
D. Left homonymous hemianopia
E. Binasal hemianopia.
ANSWER B.

Q.14: WHEN A LOW TRACHEOSTOMY IS PERFORMED BELOW THE ISTHMUS OF THE


THYROID WHICH OF THE FOLLOWING VESSELS MAY BE ENCOUNTERED:

A. Inferior thyroid artery


B. Inferior thyroid vein
C. Costocervical trunks
D. Superior thyroid artery
E. Right brachiocephalic vein.
ANSWER B.

Q.15: THE MOST COMMONLY INJURED NERVE DURING THYROIDECTOMY IS:

A. Autonomic nerve arising from cervical ganglion


B. External laryngeal nerve
C. Internal laryngeal nerve
D. Recurrent laryngeal nerve
E. Transverse cervical nerve

ANSWER D.

Head & Neck (4TH semester—Unit-I PRACTICAL MANUAL) Page 57


Q.16: WHILE MAKING THE SKIN INCISION FOR SURGICAL REMOVAL OF SUBMANDIBULAR
SALIVARY GLAND, WHICH OF THE NERVE IS AT RISK:

A. Cervical branch of facial nerve


B. Great auricular nerve
C. Hypoglossal nerve
D. Lingual nerve
E. Marginal mandibular branch of facial nerve

ANSWER A.

Q.17: A SURGEON TELLS A HOUSE OFFICER TO TAP THE SIDE OF THE FACE OF PATIENT
WHO JUST HAD THYROID SURGERY. THE SURGEON IS MOST WORRIED ABOUT DAMAGE
TO WHICH OF THE FOLLOWING:

A. Common carotid artery


B. External carotid artery
C. Recurrent laryngeal nerve
D. Superior thyroid artery
E. Parathyroid gland

ANSWER E.

Q.18: A 22-YEAR-OLD MAN REPRESENTED WITH SWELLING OF PAROTID REGION. HE HAD


CONTRACTED MUMPS FIVE WEEKS BACK BUT THE SWELLING DID NOT RESOLVE. HE WAS
SUFFERING FROM AGONIZING PAIN IN THE REGION OF PAROTID AND EAR, WHICH
INCREASED DURING CHEWING. THE NERVE MOST LIKELY TO BE MEDIATING THE PAIN IS:

A. Anterior superior alveolar


B. Auriculotemporal
C. Glossopharyngeal
D. Infra-orbital
E. Middle superior alveolar

ANSWER B.

Q.19: A PATIENT DEVELOPED DYSPNEA WHEN HE LIES FLAT AND HAS AN OBVIOUS
SWELLING IN FRONT OF NECK. THE MOST LIKELY DIAGNOSIS IS:

A. Retrosternal goiter
B. Thyroid nodule
C. Thyrotoxicosis
D. Myxedema
E. None of the above

ANSWER A.

VIII.CASE STUDY
Case—1
A 36-YEAR-OLD WOMAN COMPLAINS OF PAIN AND SWELLING BENEATH THE LEFT
MANDIBLE, PARTICULARLY AFTER EATING A MEAL. ON EXAMINATION, SHE IS NOTED TO
HAVE EDEMA AND TENDERNESS OF THE LEFT SUBMANDIBULAR REGION. PALPATION OF
HER MOUTH REVEALS A 4-mm, IRREGULAR, NONMOBILE, HARD MASS IN THE MUCOSA OF
HER MOUTH.. MOST LIKELY DIAGNOSIS IS STONE IN THE SUBMANDIBULAR DUCT
(SIALOLITHIASIS).

1. Why there is complain of pain after a meal?


ANS: Pain after a meal is from the accumulation of saliva proximal to the occluded duct, which
stretches the duct or the capsule of the gland.

Head & Neck (4TH semester—Unit-I PRACTICAL MANUAL) Page 58


Case—2
A 30-YEAR-OLD TEACHER CONSULTS HER PHYSICIAN, COMPLAINING OF A SWOLLEN
NECK. SHE HAD FIRST NOTICED THE SWELLING 3 MONTHS AGO. OVER THE LAST THREE
MONTHS, IT HAD BEEN INCREASING IN SIZE. SHE ALSO HAD SOME BREATHLESSNESS. ON
EXAMINATION, A SOLITARY SWELLING OF FIRM CONSISENCY WAS FOUND ON THE RIGHT
SIDE OF THE LARYNX AND TRACHEA. THE SMALL MASS WAS NOT ATTACHED TO THE SKIN
AND THERE WERE NO CHANGES TO THE OVERLYING SKIN. THE SWELLING MOVED
UPWARD WITH SWALLOWING. FURTHER TESTS SHOWED THAT THE MASS WAS A
CARCINOMA OF THE THYROID GLAND. THE MASS WAS SURGICALLY REMOVED.

1. Why did the tumor move upward when the patient swallowed?

ANS: The thyroid gland is invested in a sheath derived from the pretracheal fascia. This holds the
gland onto the larynx and the trachea, so the thyroid follows the movements of the larynx during
swallowing. Any pathological swelling of the thyroid will move upwards with swallowing, distinguishing
it from a mass in some other part of the neck.

2. How did the tumor cause breathlessness?

ANS: Since the thyroid is anterior to the trachea, an abnormal mass can push on the trachea and
partially occlude the lumen, causing breathlessness.

3. Which lymph nodes should the physician examine for metastases if a malignant tumor
is suspected?

ANS: The thyroid gland is drained primarily by the deep cervical lymph nodes.

4. What structures can be damaged during thyroidectomy if the surgeon is not careful?

ANS: The two main arteries supplying the gland are accompanied by nerves that can be damaged
druing thyroidectomy. The superior thyroid artery is related to the external laryngeal nerve. This nerve
supplies the cricothyroid and cricopharyngeus muscles. The inferior thyroid arteries are related to the
recurrent laryngeal nerve. This nerve supplies remaining all laryngeal muscles.

Head & Neck (4TH semester—Unit-I PRACTICAL MANUAL) Page 59


VISCERA OF THE HEAD & NECK
I. THE NOSE
 It includes the external nose and nasal cavity.
1. External nose
 The external nose is visible portion projecing from the face, having cartilaginous skeleton —
The dorsum of the nose extends from the root of the nose to the apex (tip) of the nose — The
skin extends into the vestibule of the nose, where it has a variable number of stiff hairs
(vibrissae).
a. The bony part of the external nose
 It consists of nasal bones, frontal processes of the maxillae, the nasal processes of the frontal
bone, anterior nasal spine, and the bony parts of the nasal septum.
b. The cartilaginous part of the external nose
 It consists of two lateral cartilages, two alar cartilages, and a septal cartilage.
2. Nasal septum (Fig—28)
 It is composed of: 1. PERPENDICULAR PLATE OF ETHMOID BONE: It forms the superior part
of the nasal septum — 2. THE VOMER: It forms the posteroinferior part of the nasal septum —
3. SEPTAL CARTILAGE: It articulates with the edges of the bony septum.

Fig-28: External nose and nasal septum.

Head & Neck (4TH semester—Unit-I PRACTICAL MANUAL) Page 60


3. Nasal cavity (Fig—29)
 It is entered anteriorly through the nares — It opens posteriorly into the nasopharynx through the
posterior nasal apertures or choanae.
a. Nasal mucosa
 Nasal mucosa lines the nasal cavity except for the nasal vestibule, which is lined with skin —
The nasal mucosa is firmly bound to the periosteum and perichondrium of the supporting bones
and cartilages of the nose — The nasal mucosa is continuous with the lining of the nasopharynx
posteriorly, paranasal sinuses superiorly and laterally and lacrimal sac and conjunctiva
superiorly— THE INFERIOR 2/3 OF THE NASAL MUCOSA IS THE RESPIRATORY AREA:
which warms and moistens the air before it passes through the rest of the upper respiratory
tract to the lungs — THE SUPERIOR 1/3 OF THE NASAL MUCOSA IS THE OLFACTORY
AREA: which contains the peripheral organ of smell.
b. Boundaries of the nasal cavity
 ROOF: is formed, from posterior to anterior by, body of sphenoid, cribriform plate of ethmoid,
frontal bone and nasal bone — FLOOR: by the hard palate — MEDIAL WALL: by the nasal
septum —LATERAL WALL: is irregular due to three conchae.
4. Features of lateral wall of the nasal cavity
 The conchae or turbinates offer a greater surface area for heat exchange — A recess or meatus
underlies each of the conchae — The inferior concha is the longest and broadest and is
formed by an independent bone.
a. Sphenoethmoidal recess
 It lies superoposterior to the superior concha.
b. Superior meatus
 It is a narrow passage between the superior and the middle nasal conchae.
c. Middle meatus
 The anterosuperior part of middle nasal meatus leads into a funnel-shaped opening, the ethmoidal
infundibulum, through which it communicates with the frontal sinus via frontonasal duct — The
hiatus semilunaris is a semicircular groove into which the maxillary sinus opens — The bulla
ethmoidalis, a rounded elevation located superior to the hiatus semilunaris — The bulla
ethmoidalis is formed by middle ethmoidal sinus.
d. Inferior meatus
 The nasolacrimal duct, which drains tears from the lacrimal sac, opens into the anterior part of
inferior meatus.

Head & Neck (4TH semester—Unit-I PRACTICAL MANUAL) Page 61


Fig-29: Lateral wall of the right nasal cavity.

Head & Neck (4TH semester—Unit-I PRACTICAL MANUAL) Page 62


II.CLINICAL CORRELATIONS
1. Examination Of The Nasal Cavity
 Examination of the nasal cavity may be carried out by inserting a speculum through the
external nares or by means of a mirror in the pharynx. In the latter case, the choanae and the
posterior border of the septum can be visualized.
 It should be remembered that the nasal septum is rarely situated in the midline.
 A severely deviated septum may interfere with drainage of the nose and the paranasal
sinuses.
2. Infection Of The Nasal Cavity
 Infection of the nasal cavity can spread in a variety of directions.
 The paranasal sinuses are especially prone to infection.
 Organisms may spread via the nasal part of the pharynx and the auditory tube to the middle
ear.
 It is possible for organisms to ascend to the meninges of the anterior cranial fossa, along the
sheaths of the olfactory nerves through the cribriform plate, and produce meningitis.
3. Nose Bleeding
 Epistaxis, or bleeding from the nose, is a frequent condition.
 The most common cause is nose picking.
 The bleeding may be arterial or venous, and most episodes occur on the anteroinferior
portion of the septum and involve the septal branches of the sphenopalatine and facial
vessel.
4. Infections And Otitis Media
 Pathogenic organisms can gain entrance to the middle ear by ascending through the auditory
tube from the nasal part of the pharynx. Acute infection of the middle ear (otitis media)
produces bulging and redness of the tympanic membrane.
III.REVIEW QUESTIONS
Q.1: A 54-YEAR-OLD MAN IS ADMITTED TO THE HOSPITAL WITH SEVERE PAIN IN HIS
NASAL CAVITY. RADIOGRAPHIC EXAMINATION REVEALS A CARCINOMA IN HIS NASAL
CAVITY. IN WHICH OF THE FOLLOWING LOCATIONS WOULD THE CARCINOMA BLOCK THE
HIATUS OF THE MAXILLARY SINUS?
A. Inferior meatus
B. Middle meatus
C. Superior meatus
D. Nasopharynx
E. Sphenoethmoidal recess
ANSWER: B.
 The maxillary sinus drains via the middle meatus, specifically via the semilunar hiatus. The middle
meatus and semilunar hiatus are located under the middle nasal concha.
Q.2: A 54-YEAR-OLD MALE IS DIAGNOSED WITH AN ANEURYSM OF THE BASILAR ARTERY
CLOSE TO THE CAVERNOUS SINUS. AN ANTERIOR APPROACH TO THE SELLA TURCICA
THROUGH THE NASAL CAVITY IS PERFORMED. THROUGH WHICH OF THE FOLLOWING
ROUTES IS THE SURGEON MOST LIKELY TO ENTER THE CRANIAL CAVITY?
A. Cribriform plate
B. Cavernous sinus
C. Frontal sinus
D. Maxillary sinus
E. Sphenoidal sinus
ANSWER: E.
 The sphenoidal sinus provides the most direct access to the pituitary gland, which is situated
directly above this sinus. Neither the frontal sinus nor maxillary sinus has any direct
communication with the interior of the cranial vault and would therefore not allow the surgeon a
potential access point to the pituitary gland

Head & Neck (4TH semester—Unit-I PRACTICAL MANUAL) Page 63


Q.3: AN UNCONSCIOUS 57-YEAR-OLD MAN IS TRANSPORTED TO THE EMERGENCY
DEPARTMENT AFTER FALLING FROM A TREE. A CT SCAN EVALUATION REVEALS A
FRACTURE OF THE CRIBRIFORM PLATE. WHICH OF THE FOLLOWING CONDITIONS WILL
MOST LIKELY BE PRESENT DURING THE PHYSICAL EXAMINATION:
A. Entrapment of the eyeball
B. Anosmia
C. Hyperacusis
D. Tinnitus
E. Deafness
ANSWER: B.
 The axons of olfactory nerves run directly through the cribriform plate to synapse in the olfactory
bulb. Damage to this plate can damage the nerve axons, causing anosmia (loss of the sense of
smell).
Q.4: A 1-YEAR-OLD INFANT IS ADMITTED TO THE HOSPITAL WITH FEVER. RADIOGRAPHIC
EXAMINATION REVEALS A SINUS INFECTION. WHICH OF THE FOLLOWING SINUSES IS
PRESENT AT THIS AGE?
A. Frontal sinus
B. Maxillary sinus
C. Sphenoid sinus
D. Middle ethmoidal air cells
E. Posterior ethmoidal air cells
ANSWER B.
 The maxillary sinus arises late in fetal development and is the only sinus present at birth.The
frontal and sphenoid sinuses often develop at approximately 2 years of age from the anterior
ethmoid air cells and the posterior ethmoid air cells, respectively.
Q.5: IN A PATIENT WITH SWELLING OF THE MUCOUS MEMBRANES OF THE SUPERIOR
NASAL MEATUS, WHICH OF THE OPENINGS OF THE PARANASAL SINUSES ARE PLUGGED:

A. Middle ethmoidal sinus


B. Maxillary sinus
C. Posterior ethmoidal sinus
D. Anterior ethmoidal sinus
E. Frontal sinus.
ANSWER: C.
Q.6: DURING GANG FIGHTING, THE NASAL SEPTUM OF A 17-YEAR-OLD BOY INJURED BY A
FIST. WHICH OF THE FOLLOWING STRUCTURES WOULD BE DAMAGED:

A. Septal cartilage and nasal bone.


B. Inferior concha and vomer.
C. Vomer and perpendicular plate of ethmold.
D. Septal cartilage and middle concha.
E. Cribriform plate and incisive foramen.
ANSWER: C.
Q.7: DURING EXAMINATION OF A PATIENT WITH SEVERE FACIAL ACHE,
OTOLARYNGOLOGIST NOTES DRAINAGE OF PURULENT DISCHARGE INTO MIDDLE
MEATUS WHEN THE PATIENT IS LYING ON THE RIGHT SIDE. THE MOST LIKELY SOURCE OF
THE INFECTION IS THE:

A. Anterior ethmoidal sinuses


B. Frontal sinus
C. Maxillary sinus
D. Nasolacrimal duct
E. Sphenoidal sinus

ANSWER: C.

Head & Neck (4TH semester—Unit-I PRACTICAL MANUAL) Page 64


IV.CASE STUDY
Case—1
A 22-YEAR-OLD MALE PRESENTS TO THE EMERGENCY DEPARTMENT COMPLAINING OF
SEVERE NASAL BLEEDING FOR THE PAST 30 MINUTES THAT HE CANNOT STOP. HE DENIES
ANY TRAUMA, BLEEDING DISORDERS, OR USE OF MEDICATIONS SUCH AS ASPIRIN OR
IBUPROFEN. THE PATIENT INDICATES THAT THIS NOSE BLEED IS UNIQUE BECAUSE HE IS
BLEEDING FROM BOTH NOSTRILS AND BLOOD IS DRAINING INTO HIS THROAT AND
CHOKING HIM. HE FEELS AS THOUGH THE BLOOD WERE COLLECTING IN THE BACK OF HIS
THROAT. HE HAS TRIED “PINCHING” HIS NOSE, BUT THE BLEEDING CONTINUES.
1. What is the most likely anatomical explanation for this condition?
ANS: Posterior epistaxis
2. What is Kiesselbach plexus?
ANS: Kiesselbach plexus is an area on the anterior portion of the nasal septum that is very vascular
because of the anastomosis of blood vessels; this is the most common site for epistaxis.
3. What is the most common site for epistaxis?
ANS: The most common site for epistaxis is where the several arteries that supply the nasal cavity
anastomose on the anterior nasal septum (Kiesselbach area).
Case—2
A 55-YEAR-OLD MAN HAS BECOME ANEMIC AND HYPOTENSIVE DUE TO SEVERE
ANTERIOR EPISTAXIS. AN ENT SURGEON HAS BEEN CALLED TO ADDRESS THE BLEEDING.
HE STATES THAT HE MAY NEED TO OCCLUDE THE MAJOR ARTERIAL SUPPLY.

1. Which of the following arteries is most likely to be responsible?


ANS: The major blood supply to the anterior septum is the sphenopalatine artery, a branch of which
supplies the nasal septum. The sphenopalatine artery arises from the maxillary artery, which is a
terminal branch of the external carotid artery.
Case—3
AN 18-YEAR-OLD WOMAN ARRIVES IN THE EMERGENCY DEPARTMENT COMPLAINING OF
PERSISTENT EPISTAXIS. ON EXAMINATION, THERE IS BLEEDING FROM THE RIGHT
NOSTRIL.

1. Which of the following locations is the most likely source of the bleeding?
ANS: The most common location of epistaxis is the region of the anterior septum known as
Kiesselbach plexus, which has a rich anastomosis of arteries.
Case—4
AN 18-YEAR-OLD WOMAN IS THROWN FROM HER CAR DURING A MOTOR VEHICLE
ACCIDENT AND HITS HER HEAD AGAINST THE PAVEMENT. SHE HAS LOST
CONSCIOUSNESS BUT CURRENTLY IS ALERT AND HAS EQUALLY REACTIVE PUPILS. SHE
IS WELL EXCEPT FOR CLEAR NASAL LEAKAGE FROM THE RIGHT NOSTRIL THAT HAS NOT
ABATED OVER 24 HOURS.

1. What is the most likely etiology?


ANS: This patient likely has cerebrospinal fluid (CSF) rhinorrhea, which is not unusual after head
trauma. The cribriform plate and meninges are disrupted, which allows CSF to leak through the nose.
This predisposes to meningitis.

Head & Neck (4TH semester—Unit-I PRACTICAL MANUAL) Page 65


I.THE ORAL CAVITY
1. Hard Palate
 The palate forms the roof of the mouth and the floor of the nasal cavities — The palate
consists of two regions: the hard palate anteriorly and the soft palate posteriorly — The hard
palate is formed by: 1. palatine processes of maxillae & 2. horizontal plates of palatine
bones.
 The incisive fossa is a depression in the midline of the hard palate. It transmits: 1. nasopalatine
nerve that passes from nose to oral cavity & 2. greater palatine artery that passes from oral
cavity to nose.
 Medial to the 3rd molar tooth, there is greater palatine foramen that transmits greater palatine
vessels and nerve — The lesser palatine foramen lies posterior to the greater palatine foramen
that transmit the lesser palatine vessels and nerve.
2. Parts & Surfaces of the Tongue (Fig—30 & 31)
 It is partly in the oral cavity (anterior 2/3) and partly in the oropharynx (anterior 1/3) —
ROOT: It is usually defined as the posterior third of the tongue — BODY: The body of the tongue
is the anterior two thirds of the tongue — APEX: The apex (tip) of the tongue is the anterior
end of the body — DORSAL SURFACE (DORSUM): It is located partly in the oral cavity and
partly in the oropharynx — It is characterized by a V-shaped groove, the sulcus terminalis —
The sulcus terminalis divides the dorsum of the tongue into the anterior 2/3 (oral) part and
the posterior 1/3 (pharyngeal) part — The foramen cecum is small pit anterior to the angle of
sulcus terminalis which represents the embryonic thyroglossal duct from which the thyroid
gland developed — MARGINS — VENTRAL (INFERIOR) SURFACE: This surface is
connected to the floor of the mouth by a midline fold called the frenulum of the tongue — A
sublingual caruncle (papilla) is present on each side of the base of the lingual frenulum that
includes the opening of the submandibular duct from the submandibular salivary gland.
3. Mucous Membrane Of Tongue
 The mucous membrane on the anterior part of the tongue is rough because of the presence of
numerous small lingual papillae: 1. VALLETE PAPILLAE: They lie directly anterior to the
terminal sulcus and are arranged in a V-shaped row — There walls studded with taste buds
— 2. FLILFORM PAPILLAE: These conical projections are pinkish gray and are arranged in V-
shaped rows that are parallel to the terminal sulcus, except at the apex, where they tend to be
arranged transversely — THEY DO NOT CONTAIN TASTE BUDS — 3. FUNGIFORM
PAPILLAE: They are scattered among the filiform papillae but are most numerous at the
apex and margins of the tongue — They contain taste buds — 4. FOLIATE PAPILLAE: They
are poorly developed in humans.
4. Muscles Of the Tongue
 All the intrinsic and extrinsic muscle of the tongue, with the exception of
PALATOGLOSSUS, are innerveated by the Hypoglossal (CN XII) nerve.
a. Extrinsic muscles of the tongue
 1. HYOGLOSSUS: (Depresses tongue, especially pulling its sides inferiorly)—2.
GENIOGLOSSUS: (Bilateral activity protrudes the tip of the tongue — Unilateral
contraction deviates tongue to contralateral side) — 3. STYLOGLOSSUS: (Retrudes tongue
and curls [elevates] its sides) — 4. PALATOGLOSSUS (Capable of elevating
posterior tongue or depressing soft palate).
b. Intrinsic muscles of the tongue
 The superior and inferior longitudinal, transverse, and vertical muscles are confined to the
tongue — They have their attachments entirely within the tongue and are not attached to
bone.

Head & Neck (4TH semester—Unit-I PRACTICAL MANUAL) Page 66


Fig-30: Cavity of the mouth. Cheek on the left side of the face has been cut away to show the
buccinator muscle and the parotid duct. B. Undersurface of the tongue.

Head & Neck (4TH semester—Unit-I PRACTICAL MANUAL) Page 67


Fig-31: Dorsal surface of the tongue showing the valleculae, the epiglottis, and the entrance
into the piriform fossa on each side (arrows)

II.CLINICAL CORRELATIONS
1. Clinical significance of the examination of the mouth
 The sensory nerve supply and lymph drainage of the mouth cavity should be known.
 The close relation of the lingual nerve to the lower third molar tooth should be remembered.
 The close relation of the submandibular duct to the floor of the mouth may enable one to
palpate a calculus in cases of periodic swelling of the submandibular salivary gland.
2. Laceration of the tongue
 A wound of the tongue is often caused by the patient's teeth following a blow on the chin
when the tongue is partly protruded from the mouth.
 It can also occur when a patient accidentally bites the tongue while eating, during recovery
from an anesthetic, or during an epileptic attack.
 Bleeding is halted by grasping the tongue between the finger and thumb posterior to the
laceration, thus occluding the branches of the lingual artery.
III.REVIEW QUESTIONS
Q.1: A 45-YEAR-OLD MALE CONSTRUCTION WORKER SLIPS AND FALLS ON A NAIL PROTRUDING
FROM A BOARD. THE NAIL PENETRATES THE SKIN OVERLYING THE SUBMENTAL TRIANGLE
LATERAL TO THE MIDLINE. WHICH OF THE FOLLOWING MUSCLES WOULD BE THE LAST TO BE
PENETRATED:
A. Platysma
B. Mylohyoid
C. A nterior belly of the digastric
D. Geniohyoid
E. Genioglossus
ANSWER E.

Head & Neck (4TH semester—Unit-I PRACTICAL MANUAL) Page 68


 During a puncture wound as described in this case, passing up from below the chin, the nail would first
pierce the platysma, then the anterior belly of the digastric, then the mylohyoid, then the geniohyoid, and
finally the genioglossus.
Q.2: A 55-YEAR-OLD WOMAN IS ADMITTED TO THE EMERGENCY DEPARTMENT WITH
CHEST ANGINA. ECG EXAMINATION REVEALS AN ACUTE MYOCARDIAL INFARCTION. A
SERIES OF MEDICATIONS IS ADMINISTERED TO THE PATIENT, INCLUDING SUBLINGUAL
NITROGLYCERIN FOR REDUCING HER BLOOD PRESSURE. WHICH OF THE FOLLOWING
STRUCTURES IS MOST LIKELY TO BE THE ROUTE OF ABSORPTION OF THIS DRUG:
A. Deep lingual vein
B. Submandibular duct
C. Sublingual duct
D. Lingual vein
E. Sublingual vein
ANSWER A.
 The deep lingual vein is located most superficially on the underside of the tongue. It is therefore
the most direct route for absorption of the administered nitroglycerin.
Q.3: A 35-YEAR-OLD MAN IS ADMITTED TO THE HOSPITAL WITH SEVERE PAIN IN THE
AREA OF HIS RIGHT SUBMANDIBULAR GLAND. RADIOGRAPHIC EXAMINATION REVEALS A
TUMOR OF THE GLAND. AN INCISION IS MADE AND THE SUBMANDIBULAR GLAND AND ITS
DUCT ARE REMOVED. WHICH OF THE FOLLOWING NERVES IS MOST COMMONLY INJURED
IN THIS TYPE OF PROCEDURE?
A. Buccal
B. Lingual
C. Inferior alveolar
D. Nerve to mylohyoid
E. Glossopharyngeal
ANSWER B.
 The lingual nerve initially courses directly underneath the mucosa of the floor of the mouth and
superficial to the submandibular gland, specifically the submandibular duct. This nerve is
therefore at risk for ligation, division, or trauma during excision of the gland and duct. The lingual
nerve is part of mandibular division of the trigeminal nerve and carries fibers from chorda tympani.
 Q.4: INJURY TO THE RIGHT LINGUAL NERVE WILL CAUSE:
A. Loss of sensation in right half of posterior 2/3 of tongue
B. An area of anesthesia over the chin
C. Deviation of tongue to left side
D. Loss of taste in anterior two third of the tongue
E. Lack of sensation in the gums of upper jaw.
ANSWER: D.
Q.5: 34-YEAR OLD MALE PATIENT PRESENTED TO A SURGEON WITH A SWELLING ON TIP
OF TONGUE. ON CLINICAL EXAMINATION WHICH LYMPH NODE GROUP WILL BE
PALPABLE:

A. Preauricualr lymph node


B. Deep cervical chain
C. Submandibular lymph nodes
D. Submental lymph nodes
E. Jugulodigastric node
ANSWER: D.
Q.6: A 28-YEAR-OLD WOMAN IS INVOLVED IN AN AUTOMOBILE ACCIDENT. SHE IS NOT
WEARING A SEAT BELT, AND FALLS THROUGH THE WINDSHIELD WHEN HER CAR HITS A
TREE. AFTER THE ACCIDENT, SHE HAS DIFFICULTY OPENING HER MOUTH, BUT HAS NO
DIFFICULTY IN CLOSING IT. WHICH OF FOLLOWING MUSCLES WAS MOST LIKELY INJURED:

A. Lateral pterygoid
B. Masseter
C. Medial pterygoid
D. Orbicularis oris
E. Temporalis
ANSWER: A.

Head & Neck (4TH semester—Unit-I PRACTICAL MANUAL) Page 69


Q.7: A 68-YEAR-OLD HYPERTENSIVE MAN SUFFERS A SMALL STROKE. ON EXAMINATION,
THE MAN IS UNABLE TO TOUCH THE TIP OF TONGUE TO ROOF OF HIS MOUTH.
DYSFUNCTION OF WHICH OF THE FOLLOWING MUSCLES WOULD BE MOST LIKELY TO
PRODUCE THIS RESULT:

A. Buccinator
B. Geniohyoid
C. Palatoglossus
D. Palatopharyngeus
E. Tensor veli palatini
ANSWER: C.
Q.8: AFTER TONSILLECTOMY, A 7-YEAR-OLD BOY IS UNABLE TO DISTINGUISH THE
SENSATION OF TASTE ON THE POSTERIOR ONE-THIRD OF HIS TONGUE. WHICH OF THE
FOLLOWING NERVES MOST LIKELY HAS BEEN INJURED:

A. Internal laryngeal nerve


B. Lingual nerve
C. Lingual branch of the Glossopharyngeal nerve
D. Greater palatine nerve
E. Chorda tympani.
ANSWER: C.
Q.9: THE TONGUE OF A 45-YEAR-OLD PATIENT DEVIATES TO THE LEFT ON PROTRUSION.
WHICH OF THE FOLLOWING NERVES IS INJURED:

A. Right lingual nerve


B. Left lingual nerve
C. Right hypoglossal nerve
D. Left hypoglossal nerve
E. Left Glossopharyngeal nerve
ANSWER: D.

Q.10: A PATIENT IN THE OPD IS UNABLE TO TASTE SUGAR ON THE ANTERIOR PART OF
TONGUE. THE NERVE MOST LIKELY TO BE INVOLVED IS:

A. Accessory nerve
B. Facial nerve
C. Glossopharyngeal nerve
D. Maxillary nerve
E. Vagus nerve

ANSWER: B.
Q.11: A PATIENT HAS A MASS ON THE LATERAL PART OF ANTERIOR TWO-THIRD OF THE
TONGUE ON THE LEFT SIDE. WHICH GROUP OF LYMPH NODES WILL DRAIN THIS PART:
A. Submental group of lymph nodes
B. Inferior group of deep cervical lymph nodes on the right side
C. Inferior group of deep cervical lymph nodes on the left side
D. Superior group of deep cervical lymph nodes on the right side
E. Superior group of deep cervical lymph nodes on the left side

ANSWER: C.

IV.CASE STUDY
Head & Neck (4TH semester—Unit-I PRACTICAL MANUAL) Page 70
Case—1
A 17-YEAR-OLD BOY WAS SEEN IN THE EMERGENCY DEPARTMENT AFTER RECEIVING A
STAB WOUND AT THE FRONT OF THE NECK. THE KNIFE ENTRANCE WOUND WAS LOCATED
ON THE LEFT SIDE OF THE NECK JUST LATERAL TO THE TIP OF THE GREATER CORNU OF
THE HYOID BONE. DURING THE PHYSICAL EXAMINATION THE PATIENT WAS ASKED TO
PROTRUDE HIS TONGUE, WHICH DEVIATED TO THE LEFT.

1. Which muscles are responsible for the protrusion of the tongue?


ANS: The genioglossus muscles are responsible for protruding the tongue.

2. What is the nerve supply of of genioglossus?


ANS: The genioglossus muscle is supplied by the hypoglossal nerve.

3. Why injury to the left hypoglossal nerve results in deviation of the tongue towards left
side?
ANS: Injury to left hypoglossal nerve → Paralysis of the left genioglossus muscle → permitted the
right genioglossus to pull the tongue forward and turned the tip to the left side.

4. What is the relation of hypoglossal nerve to thr tip of greater cornu of the hyoid bone?
ANS: At about the level of the tip of the greater cornu of the hyoid bone the hypoglossal nerve turns
forward and crosses the internal and external carotid arteries and the lingual artery to enter the
tongue.

Case—2
YOU WERE ASKED TO ASSESS THE CASE OF 20-YEAR-OLD WOMAN WHOSE IMPACTED
RIGHT LOWER WISDOM TOOTH WAS SURGICALLY REMOVED. THE OPERATION LASTED
ABOUT AN HOUR, AND THE DENTAL SURGEON SUSPECTED THAT SOME NERVES MIGHT
HAVE BEEN INJURED DURING THE OPERATION. THE PATIENT PRESENTED WITH LOSS OF
SENSATION IN THE GUMS OF HER LOWER JAW, AND HER MOUTH WAS SLIGHTLY DRY.
YOU EXAMINED TASTE SENSATION IN THE TONGUE AND FOUND THAT IT WAS DIMINISHED
IN THE ANTERIOR 2/3 BUT IT WAS NORMAL IN THE POSTERIOR 1/3.

1. Which nerve is most likely to have been injured in this patient?

ANS: The lingual nerve, which is a branch of the posterior division of the mandibular nerve (CN V3).

2. Why the lingual nerve can be injured during the procedure on impacted 3RD molar
tooth?

ANS: The lingual nerve, which is a branch of the posterior division of the mandibular nerve (CN V3), is
closely related to the 3RD molar tooth and marks its way along the posterior part of the body of
mandible

3. How would you explain the patient's complaints?

ANS: The lingual nerve supplies general sensory fibers to the anterior two-thirds of the tongue,
the floor of the mouth and the gums of the mandibular teeth. It carries the chorda tympani nerve,
which carries taste fibers from the anterior two-thirds of the tongue and parasympathetic
innervation to the submandibular and sublingual salivary glands.

I. THE PHARYNX
Head & Neck (4TH semester—Unit-I PRACTICAL MANUAL) Page 71
 The pharynx is situated behind the nasal cavities, the oral cavity and the larynx and may be
divided into: Nasopharynx, Oropharynx & Laryngeal part of pharynx (laryngopharynx or
hypopharynx)
 It has a musculomembranous wall, that is deficient anteriorly, where, it is replaced by:
1.Posterior nasal apertures (choana) in case of NASOPHARYNX, 2. Oropharyngeal isthmus
in case of OROPHARYNX & 3. Laryngeal inlet in case of LARYNGOPHARYNX
 By means of auditory tube, the mucous membrane is also continuous with that of the middle
ear cavity and the mastoid air cells.

1.Muscles of the pharynx (Fig: 32 & 33)


a. Circular muscles of the pharynx:

 SUPERIOR PHARYNGEAL CONSTRICTOR: It takes origin from Pterygoid hamulus,


pterygomandibular raphe, posterior end of mylohyoid line & Pharyngeal tubercle on basilar
part of occipital bone and inserts on Pharyngeal raphe — 2. MIDDLE PHARYNGEAL
CONSTRICTOR: It takes origin from Stylohyoid ligament & Greater and lesser horns of hyoid
and Inserts on Pharyngeal raphe — 3. INFERIOR PHARYNGEAL CONSTRICTOR: It takes
origin from Oblique line on lamina of thyroid cartilage and side of cricoid cartilage and
inserts in Pharyngeal raphe. However, cricopharyngeal part encircles pharyngo-
esophageal junction without forming a raphe.

b. Longitudinal muscles of the pharynx:

 PALATOPHARYNGEUS: It takes origin from Hard palate and palatine aponeurosis &
inserts in posterior border of lamina of thyroid cartilage — 2. SALPINGOPHARYNGEUS: It
takes origin from cartilaginous part of auditory tube & blends with palatopharyngeus —
3. STYLOPHARYNGEUS: It takes origin from styloid process of temporal bone & inserts in
posterior borders of thyroid cartilage.
 The pharynx is situated behind the nasal cavities, the oral cavity and the larynx and may be
divided into nasopharynx, oropharynx and laryngeal part of pharynx.
2. Nasopharynx
 It extends from the base of the skull to the upper surface of the soft palate — It is bounded by:
ANTERIORLY: Posterior openings into nasal cavities (choana), through which nasopharynx
communicates with the nasal cavity — POSTERIORLY: Superior pharyngeal constrictor
supported by 1ST cervical vertebra — ROOF: Base of skull — In the submucosa of the roof is a
collection of lymphoid tissue, prominent only in children, called the pharyngeal tonsil. When
enlarged, the pharyngeal tonsils are commonly called adenoids — FLOOR:Superior sloping
surface of the soft palate and Oropharyngeal isthmus, an opening in the floor between the soft
palate and the posterior pharyngeal wall, through which nasopharynx joins with the oropharynx
— LATERALLY: Superior pharyngeal constrictor — On the lateral wall is the opening of the
auditory tube, the elevated ridge of which is called the tubal elevation — The inferior edge of the
auditory tube gives attachment to salpingopharyngeus muscle — The muscle is covered by fold of
mucous membrane, called the salpingopharyngeal fold — A depression in the pharyngeal wall
behind the tubal elevation is called the pharyngeal recess (or fossa of Rosenmullar) — In the
submucosa of the pharyngeal recess is a collection of lymphoid tissue called the tubal tonsils.

3.Oropharynx
 It extends from the lower surface of the soft palate to the upper border of the epiglottis — It is

Head & Neck (4TH semester—Unit-I PRACTICAL MANUAL) Page 72


bounded by: ANTERIORLY: Posterior opening into the oral cavity, called pharyngeal isthmus,
bounded by the palatoglossal arch, through which oropharynx communicates with the oral
cavity — POSTERIORLY: All three pharyngeal constrictors — FLOOR: Posterior 1/3 of the
tongue — In the submucosa of the posterior 1/3 of the tongue is a collection of lymphoid tissue
called the lingual tonsil — ROOF: Superior sloping surface of the soft palate &
Oropharyngeal isthmus — LATERALLY: Palatoglossal fold is a projecting ridge formed by the
underlying palatoglossal muscle — The palatoglossal folds of the two sides form the
PALATOGLOSSAL ARCH — Palatopharyngeal arch is a projecting ridge formed by the
underlying palatopharyngeal muscle — The palatopharyngeal folds of the two sides form the
PALATOPHARYNGEAL ARCH — The palatoglossal arch and palatopharyngeal arch together are
called the pillars of fauces — Palatine tonsil is a large collection of lymphoid tissue which projects
into the oropharynx from the tonsillar fossa between the palatoglossal fold in front and the
palatoglopharyngeal fold behind.

Fig-36: The pharynx seen from behind.

Head & Neck (4TH semester—Unit-I PRACTICAL MANUAL) Page 73


Fig-32: Three constrictor muscles of the pharynx.

4.Laryngeal Part Of Pharynx


 The laryngeal part of pharynx (laryngopharynx) extends from the upper border of the epiglottis to
the level of the lower border of cricoid cartilage where it becomes continuous with the
oesophagus — It is bounded by: ANTERIORLY: The opening into the larynx (laryngeal inlet)
— The piriform recesses lie beside the aperture of the larynx — POSTERIORLY: The posterior
wall of the laryngopharynx is formed by the three overlapping constrictors — LATERALLY:
 At each side of the epiglottis the lateral glossoepiglottic fold separates the oropharynx from the
laryngopharynx.
5. Soft Palate
 It can be raised so that the it makes contact with the posterior wall of the pharynx to close off
the nasopharynx during swallowing — It consists of an aponeurosis that is acted upon by
attached muscles to alter its shape and position, but much `of its bulk is due to mucous and
serous glands.
Muscles of the soft palate
 1. TENSOR VELI PALATINE: It tenses the palatine aponeurosis so that other muscles may
elevate and depress it without altering its shape — It pulls upon the cartilage of the auditory
tube (e.g. in swallowing and yawning), & opens the tube, to permit equalization of air pressure
between the middle ear and nose — 2. LEVATOR VELI PALATINE: It pulls the palate upwards
and backwards — 3. PALATOGLOSSUS: It raises the tongue and narrows the transverse
diameter of the isthmus — 4. PALATOPHARYNGEUS: It is an elevator of the pharynx and
larynx — 5. MUSCULUS UVULAE: It aids palatopharyngeal closure.

Head & Neck (4TH semester—Unit-I PRACTICAL MANUAL) Page 74


Fig-33: Junction of the nose with the nasal part of the pharynx and the mouth with the oral
part of the pharynx.

II.CLINICAL CORRELATIONS
1. The lymphoid tissue of the pharynx
 At the junction of the mouth with the oral part of the pharynx, and the nose with the nasal part of
the pharynx, are collections of lymphoid tissue of considerable clinical importance.
 The palatine tonsils and the nasopharyngeal tonsils are the most important.
2. Tonsils and tonsillitis
 The palatine tonsils reach their maximum normal size in early childhood.
 After puberty, together with other lymphoid tissues in the body, they gradually atrophy.

Head & Neck (4TH semester—Unit-I PRACTICAL MANUAL) Page 75


 The palatine tonsils are common site of infection, producing characteristic sore throat and
pyrexia.
 The deep cervical lymph node situated below and behind the angle of the mandible, which
drains lymph from this organ, is usually enlarged and tender.
 Recurrent attacks of tonsillitis are best treated by tonsillectomy.
 After tonsillectomy, the external palatine vein, which lies lateral to the tonsil, may be the
source of troublesome postoperative bleeding.
3. Quinsy
 A peritonsillar abscess (quinsy) is caused by spread of infection from the palatine tonsil to the
loose connective tissue outside the capsule.
4. Adenoids
 Excessive hypertrophy of the lymphoid tissue, usually associated with infection, causes the
pharyngeal tonsils to become enlarged; they are then commonly referred to as adenoids.
 Marked hypertrophy blocks the posterior nasal openings and causes the patient to snore
loudly at night and to breathe through the open mouth.
 The close relationship of the infected lymphoid tissue to the auditory tube may be the cause of
deafness and recurrent otitis media.
 Adenoidectomy is the treatment of choice for hypertrophied adenoids with infection.
5. Piriform fossa and foreign bodies
 Clinically, it is important because it is common site for the lodging of sharp ingested bodies.
 The presence of such a foreign body immediately causes the patient to gag violently. It is
difficult for the patient to remove it without a physician's assistance.
6. Pharyngeal pouch
 The inferior constrictor muscle reveals a potential gap between the upper oblique and the
lower horizontal fibers (cricopharyngeus).This area is marked by a dimple in the lining
mucous membrane.
 Should the cricopharyngeus fail to relax during swallowing, the internal pharyngeal pressure
may rise to produce a diverticulum.
 Once the diverticulum has been formed, it may gradually enlarge and fill with food with each
meal.
 Unable to expand posteriorly because of the vertebral column, it turns downward, usually on the
left side. The presence of the pouch filled with food causes difficulty in swallowing (dysphagia).
III.REVIEW QUESTIONS
Q.1: A 7-YEAR-OLD BOY WITH A HIGH FEVER IS BROUGHT TO THE PEDIATRICIAN. DURING
PHYSICAL EXAMINATION THE PATIENT COMPLAINS OF PAIN IN HIS EAR. HIS THROAT
APPEARS RED AND INFLAMED, CONFIRMING THE DIAGNOSIS OF PHARYNGITIS. WHICH OF
THE FOLLOWING STRUCTURES PROVIDED A PATHWAY FOR THE INFECTION TO SPREAD
TO THE TYMPANIC CAVITY (MIDDLE EAR)?
A. Choanae
B. Internal acoustic meatus
C. External acoustic meatus
D. Pharyngotympanic tube
E. Pharyngeal recess
ANSWER: D.
 The pharyngotympanic (eustachian) tube connects the middle ear and the nasopharynx and is the
conduit for spreading infections.
Q.2: AN 11-YEAR-OLD BOY IS EXAMINED BY AN ENT DOCTOR FOR HIS SWOLLEN
PALATINE TONSILS. THE PALATINE TONSILS ARE LOCATED BETWEEN THE ANTERIOR AND
POSTERIOR TONSILLAR PILLARS. WHICH OF THE FOLLOWING MUSCLES FORM THESE
PILLARS?
A. Levator veli palatini and tensor veli palatini
B. Palatoglossus and palatopharyngeus
C. Styloglossus and stylopharyngeus
D. Palatopharyngeus and salpingopharyngeus
E. Superior and middle pharyngeal constrictors

Head & Neck (4TH semester—Unit-I PRACTICAL MANUAL) Page 76


ANSWER: B.
 The palatine tonsils lie in tonsillar beds with muscular (covered with mucosa) anterior and
posterior pillars forming the boundaries of the bed. These pillars are formed by the palatoglossal
arch, anteriorly, and the palatopharyngeal arch, posteriorly. The anterior pillar, part of the
palatoglossal arch, contains the palatoglossus muscle; the posterior pillar, provided by the
palatopharyngeal arch, is formed by the palatopharyngeus muscle.
Q.3: A 17-YEAR-OLD WOMAN IS ADMITTED TO THE HOSPITAL WITH TONSILLITIS. A
TONSILLECTOMY IS PERFORMED AND THE PATIENT COMPLAINS POSTOPERATIVELY OF
EAR PAIN. WHICH OF THE FOLLOWING NERVES WAS MOST LIKELY INJURED DURING THE
SURGICAL PROCEDURE?
A. Auriculotemporal
B. Lesser petrosal
C. Vagus
D. Glossopharyngeal
E. Chorda tympani
ANSWER: D.
 The glossopharyngeal nerve mediates general somatic sensation from the pharynx, the auditory
tube, and from the middle ear. Painful sensations from the pharynx, including the auditory tube,
can be referred to the ear by this nerve, as in this case of tonsillectomy.
Q.4: A 5-YEAR-OLD GIRL IS ADMITTED TO THE HOSPITAL WITH AN UPPER RESPIRATORY
TRACT INFECTION. DURING PHYSICAL EXAMINATION HER SENSE OF HEARING APPEARS
TO BE POOR. HER RIGHT EAR IS PAINFUL, AND UPON OTOSCOPIC EXAMINATION A
GOLDEN BROWN FL UID CAN BE OBSERVED THROUGH THE TYMPANIC MEMBRANE.
WHICH IS THE MOST LIKELY DIRECT ROUTE FOR THE SPREAD OF AN INFECTION FROM
THE UPPER RESPIRATORY TRACT TO THE MIDDLE EAR CAVITY?
A. Pharyngotympanic tube
B. Choanae
C. Nostrils
D. Facial canal
E. Internal acoustic meatus
ANSWER: A.
 The auditory (eustachian or pharyngotympanic) tube is a mucosal-lined tube that provides a direct
connection from the nasopharynx to the middle ear cavity. A respiratory infection can travel from
the upper respiratory tract to the oropharynx or nasopharynx and then on into the middle ear via
the auditory tube.
Q.5: A 10-YEAR-OLD GIRL IS ADMITTED TO THE HOSPITAL WITH TONSILLITIS. A
TONSILLECTOMY IS PERFORMED AND THE TONSILS ARE REMOVED. ON PHYSICAL
EXAMINATION ONE WEEK LATER THE PATIENT HAS ABSENCE OF THE GAG REFLEX ON
THE LEFT WHEN THE POSTERIOR PART OF THE TONGUE IS DEPRESSED. THE SENSORY
PORTION OF WHICH OF THE FOLLOWING NERVES WAS MOST LIKELY INJURED?
A. Facial
B. Glossopharyngeal
C. Mandibular
D. Maxillary
E. Hypoglossal
ANSWER: B.
 The gag reflex is composed of both an afferent and an efferent limb. These reflexes are mediated
by the glossopharyngeal and vagus nerves, respectively. Together, the glossopharyngeal and
vagus nerves are responsible for the contraction of the muscles of the pharynx involved in the gag
reflex. In this case the glossopharyngeal nerve was injured when the tonsils were excised,
resulting in the loss of the sensory side of the reflex
Q.6: A 45-YEAR-OLD MAN WITH A COMPLAINT OF EAR PAIN AND DIFFI CULTY HEARING IS
DIAGNOSED WITH TONSILLITIS. OTOSCOPIC EXAMINATION REVEALS FLUID IN THE MIDDLE EAR
CAVITY. HYPERTROPHY OF WHICH OF THE FOLLOWING STRUCTURES WOULD BE MOST LIKELY TO
COMPROMISE THE DRAINAGE OF THE AUDITORY TUBE?
A. Lingual tonsil
B. Palatine tonsil
C. Pharyngeal tonsil
D. Superior constrictor muscle
E. Uvula
ANSWER: C.

Head & Neck (4TH semester—Unit-I PRACTICAL MANUAL) Page 77


 The pharyngeal tonsil is situated in a slitlike space, the pharyngeal recess, in the nasopharynx behind the
opening of the auditory (eustachian) tube, and a pharyngeal tonsil in this location can lead to blockage of the
drainage of the auditory tube.
Q.7: AN 11-YEAR-OLD BOY WITH SWOLLEN PALATINE TONSILS IS EXAMINED BY AN
OTOLARYNGOLOGIST. WHICH OF THE FOLLOWING ARTERIES SUPPLIES MOST OF THE BLOOD TO
THESE TONSILS AND MUST BE PROTECTED WHEN ITS TONSILLAR BRANCH IS DIVIDED?
A. Ascending pharyngeal
B. Facial
C. Lingual
D. Descending palatine
E. Superior thyroid
ANSWER: B.
 The palatine tonsils are highly vascular and are primarily supplied by the tonsillar branch of the facial artery;
therefore, care is taken to preserve this artery while performing a tonsillectomy.
Q.8: A 14-YEAR-OLD FEMALE HAS BEEN SUFFERING FROM QUINSY IN THE RIGHT SIDE OF HER
OROPHARYNX. IN SURGICAL REMOVAL OF THE PATHOLOGIC TISSUE, OR FOR INCISION AND
DRAINAGE OF THE AREA, WHICH OF THE FOLLOWING ARTERIES WILL BE AT GREATEST RISK:
A. Lingual
B. A branch of facial
C. S uperior laryngeal artery
D. Ascending pharyngeal artery
E. Descending palatine artery
ANSWER: B.
 A branch of the facial artery would be of primary concern because its branches supply the oropharynx and it
is the primary source of arterial supply to the palatine tonsil.
Q.9: A 36-YEAR-OLD WOMAN IS ADMITTED TO THE HOSPITAL WITH SEVERE HEAD INJURIES AFR A
CAR CRASH. DURING NEUROLOGIC EXAMINATION IT IS NOTED THAT HER UVULA IS DEVIATED TO
THE RIGHT. WHICH OF THE FOLLOWING MUSCLES IS PARALYZED?
A. Left levator veli palatini
B. Left tensor veli palatini
C. Right levator veli palatini
D. Right tensor veli palatini
E. Right tensor veli palatini and left levator veli palatini
ANSWER A.
 The uvula would move toward the intact right side. This is because the intact levator veli palatine would be
unopposed by the opposite, paralyzed left levator veli palatini.
Q.10: AN 11-YEAR-OLD BOY VISITS THE OUTPATIENT CLINIC WITH A HISTORY OF RECURRENT
INFECTIONS OF HIS TONSILS. WHICH OF THE FOLLOWING LYMPH NODES IS MOST LIKELY TO FI RST
BECOME VISIBLY ENLARGED DURING TONSILLITIS?
A. Submandibular
B. Parotid
C. Jugulodigastric
D. Submental
E. Preauricular
ANSWER C.
 The jugulodigastric node, also known as the tonsillar lymph node, receives drainage from the tonsils, tongue,
and pharynx. It is often enlarged during tonsillitis.
Q.11: A 5-YEARS-OLD BOY IS DIAGNOSED AS HAVING ADENOIDS. DURING EXAMINATION OF
NASOPHARYNX FOLLOWING IS LIKELY TO BE NOTED:

A. Has Eustachian tube opening in its posterior wall.


B. Has enlarged pharyngeal tonsils
C. Has enlarged palatine tonsils
D. Lies below soft palate
E. Is continuous with esophagus through pharyngeal isthmus
ANSWER: B.
Q.12: IN A PATIENT WHO DEMONSTRATES, A LACK OF GENERAL SENSATION IN THE NASOPHARYNX, A LESION
OF WHICH OF THE FOLLOWING NERVES WOULD BE EXPECTED:

A. Maxillary nerve.
B. Superior cervical ganglion.
C. External laryngeal nerve.
D. Glossopharyngeal nerve.
E. Vagus nerve.
ANSWER: A.

Head & Neck (4TH semester—Unit-I PRACTICAL MANUAL) Page 78


Q.13: A 32-YEAR-OLD WOMAN HAS HOARSENESS IN HER VOICE AND HER UVULA IS
DEVIATED TO THE LEFT UPON PHONATION. WHICH OF THE FOLLOWING NERVES IS
DAMAGED:

A. Right trigeminal nerve


B. Left trigeminal nerve
C. Right vagus nerve
D. Left vagus nerve
E. Left Glossopharyngeal nerve.
ANSWER: C.
IV.CASE STUDY
Case—1
AN 8-YEAR-OLD GIRL WAS TAKEN TO A PEDIATRICIAN BECAUSE HER MOTHER HAD
NOTICED A SMALL PAINLESS SWELLING BELOW AND BEHIND THE ANGLE OF THE JAW ON
THE RIGHT SIDE. ON EXAMINATION, THE SWELLING WAS SUPERFICIAL, COOL TO TOUCH,
AND SHOWED NO REDNESS. CAREFUL PALPATION OF THE NECK REVEALED TWO FIRM
LUMPS MATTED TOGETHER BENEATH THE ANTERIOR BORDER OF THE RIGHT
STERNOCLEIDOMASTOID MUSCLE. EXAMINATION OF THE PALATINE TONSILS SHOWED
MODERATE HYPERTROPHY ON BOTH SIDES WITH A FEW PUSTULES EXUDING FROM THE
TONSILLAR CRYPTS ON THE RIGHT SIDE. THE PATIENT DID NOT HAVE A PYREXIA. THE
PATIENT IS HAVING CHRONIC CERVICAL LYMPHADENITIS:

1. Where does the palatine tonsil drain its lymph?


ANS: The lymph drains from tonsil into the jugulodigastric member of deep cervical lymph nodes.

2. In this case, what limits the spread of infection in the neck?


ANS: The investing layer of deep cervical fascia can limit the spread of infection in the neck.

3. Why the lymph nodes become matted together?


ANS: Tuberculous infection of a lymph node commonly spreads to other nodes in the group and they
become matted together.

4. What is the cause of cold abscess?


ANS: Tuberculous infection results in the destruction of the node with the formation of pus that later
erodes through the deep fascia, producing a large cold abscess beneath the skin.

Case—2
A 52-YEAR-OLD MAN WAS EATING HIS DINNER IN A SEAFOOD RESTAURANT WHEN HE
SUDDENLY CHOKED ON A PIECE OF FISH. HE GASPED THAT HE HAD A BONE STUCK IN HIS
THROAT. THE FISH BONE WAS STUCK IN THE PIRIFORM FOSSA.

1. Where is piriform fossa is located?


ANS: The piriform fossae lie on either side of the entrance into the larynx.

2. What is nerve supply of the mucous membrane lining the piriform fossa?
ANS: The mucous membrane lining the piriform fossa is innervated by the internal laryngeal branch
of the superior laryngeal nerve from the vagus.

3. What is relation of larynx with piriform fossa?


ANS: The piriform fossa is bounded laterally by the thyroid cartilage and the thyrohyoid membrane.
The piriform fossa is bounded medially by the aryepiglottic fold.

Head & Neck (4TH semester—Unit-I PRACTICAL MANUAL) Page 79


Case—3
A 7-YEAR-OLD BOY WAS REFERRED TO AN ENT SPECIALIST AFTER HAVING RECURRENT
EPISODES OF TONSILLITIS WITH PERITONSILLAR ABSCESSES. HIS MOTHER NOTED
ABOUT SEVEN INFECTIONS OVER THE PAST 8 MONTHS, ALL TREATED WITH ANTIBIOTICS.
AFTER DISCUSSING TREATMENT OPTIONS WITH THE FAMILY, THE ENT RECOMMENDED A
TONSILLECTOMY. THE PATIENT’S TONSILLECTOMY WAS COMPLICATED BY BLEEDING
FROM THE SURGICAL BED, AND HE HAD TEMPORARY LOSS OF TASTE SENSATION FROM
HIS POSTERIOR TONGUE. HE IS CURRENTLY DOING WELL AND WITHOUT COMPLAINTS.
1. Name the blood vessel involved with intraoperative bleeding.
ANS: The tonsillar bed is extremely vascular with the most common source of intraoperative bleeding
from the external palatine vein which arises from the lateral aspect of the tonsillar bed.
2. Why there is loss of taste sensation on posterior 1/3 of the tongue?
ANS: Compression of glossopharyngeal nerve branches, as a result of edema, causes a
temporary loss in taste sensation on the posterior 1/3 of the tongue. As the swelling goes down,
so does the nerve impairment
3. During a procedure to remove a palatine tonsil, the operating field was suddenly filled
with bright red blood. Which artery was inadvertently damaged?
ANS: The tonsillar branch of the facial artery lies in the bed of the palatine tonsil and is susceptible
to damage. Although the ascending palatine artery sends branches to the tonsil, it is not likely to be
affected in a routine procedure.
4. What is Waldeyer ring?
ANS: The tonsillar ring (Waldeyer) is a discontinuous mass of lymphoid tissue located where the body
opens to the environment, exposing the immune system to pathogens. It consists of Tubal tonsils
and pharyngeal tonsils in the nasopharynx and lingual tonsils and palatine tonsils in oropharynx
Case—4
A MEDICAL STUDENT WAS CELEBRATING THE END OF MIDTERM EXAMS WITH HER
FRIENDS IN A SEAFOOD RESTAURANT WHEN SHE STARTED FEELING A PRICKLING
SENSATION IN HER NECK AFTER SWALLOWING A LARGE BITE OF SMOKED FISH. THE PAIN
WAS GETTING WORSE, AND ATTEMPTS TO CLEAR IT WITH DRINKS FAILED. AT THE
EMERGENCY ROOM A PLAIN X-RAY OF HER NECK SHOWED A TINY FISH BONE LODGED IN
THE LOWER PART OF THE PHARYNX. THE BONE WAS QUICKLY REMOVED UNDER
GENERAL ANESTHESIA, AND THE PATIENT WAS DISCHARGED A FEW HOURS LATER.

1. Which are the most usual places for swallowed foreign bodies to be lodged?

ANS: The piriform recess and the valleculae on either side of the median glosso-epiglottic fold.

2. What is the piriform recess?

ANS: This is a small, pear-shaped depression of the laryngopharyngeal cavity on each side of
the inlet of the larynx. It is separated from the inlet by the aryepiglottic fold. Laterally the piriform
recess is bounded by the medial surfaces of the thyroid cartilage and the thyrohyoid membrane.
The branches of the internal laryngeal and inferior laryngeal nerves (continuation of recurrent
laryngeal) lie deep to the mucous membrane of the piriform recess.

3. Fish bones and other foreign bodies may pierce the mucous membrane of the recess
and cause injury to the internal laryngeal nerve. What are the possible consequences
of this injury?

ANS: Injury to the internal laryngeal nerve will result in anesthesia of the laryngeal mucous
membrane as far inferiorly as the vocal folds.

Head & Neck (4TH semester—Unit-I PRACTICAL MANUAL) Page 80


Case—5
A 12-YEAR-OLD BOY WAS ADMITTED TO THE HOSPITAL COMPLAINING OF A SEVERE SORE
THROAT AND BILATERAL EARACHE. HE HAD A HISTORY OF FREQUENT INFECTIONS OF
THE PALATINE TONSILS, WHICH HAD ALL BEEN TREATED SUCCESSFULLY WITH
ANTIBIOTICS; HOWEVER, THE INFECTIONS HAD BECOME PROGRESSIVELY MORE SEVERE.
THIS BOUT OF TONSILLITIS WAS ALSO TREATED WITH ANTIBIOTICS SUCCESSFULLY, BUT
THE BOY'S PHYSICIAN SUGGESTED THAT A TONSILLECTOMY BE PERFORMED TO
ELIMINATE THE PROBLEM ONCE AND FOR ALL. THE BOY WAS READMITTED TO THE
HOSPITAL FOR SURGERY TWO WEEKS AFTER HIS MOST RECENT INFECTION HAD
CLEARED UP. THE SURGERY WAS PROCEEDING WELL WHEN SUDDENLY THERE WAS A
MASSIVE AMOUNT OF BLEEDING. AFTER A SHORT PERIOD, THE SURGEON WAS ABLE TO
LOCATE THE BLEEDER AND LIGATE IT, FOLLOWING WHICH THE WOUND WAS CLOSED
AND THE PATIENT'S RECOVERY WAS UNEVENTFUL.

1. Where is the palatine tonsil located? The lingual tonsil? The pharyngeal tonsil?

ANS: The palatine tonsil is found beneath the mucous membrane between the palatoglossal and
palatopharyngeal arches. The lingual tonsil is located on the posterior 1/3 of the tongue and the
pharyngeal tonsil, or adenoid, is found on the upper part of the posterior wall of nasopharynx.

2. What is blood supply of palatine tonsil and who is responsible for the bleeding?

ANS: The palatine tonsil is supplied by five arterial branches: 1. the ascending palatine and 2.
tonsillar branches of the facial artery, 3. the palatine branch of the ascending pharyngeal artery,
4. the dorsal lingual branch of the lingual artery, and 5. the descending palatine branch of the
maxillary artery. The PRIMARY SOURCE OF HEMORRHAGE, HOWEVER, IS USUALLY THE
EXTERNAL PALATINE VEIN. The internal carotid artery is usually safe during tonsillectomy, but may
be damaged if it is located unusually close to the lateral side of the tonsil.

3. What nerves are at risk during a tonsillectomy?

ANS: The glossopharyngeal nerve accompanies the tonsillar artery on the lateral wall of the
pharynx and is particularly vulnerable during a tonsillectomy. In addition, a careless surgeon may
damage the lingual nerve, which passes lateral to the pharyngeal wall, just anterior to the tonsil.

4. What lymphatic structures are often secondarily affected in tonsillitis?

ANS: Tonsillitis of the palatine tonsil may spread to the lingual and pharyngeal tonsils, which are
associated in a grouping designated the tonsillar (Waldeyer's) ring. The tonsillar ring is assumed
to have protective significance during ingestion. The tonsils drain through the superior deep
cervical lymph nodes, and thus these may also be affected. The JUGULODIGASTRIC
(TONSILLAR) NODE, WHERE MOST TONSILLAR LYMPHATIC VESSELS DRAIN, IS
PARTICULARLY VULNERABLE.

5. How do you explain the patient's complaint about pain in both ears?

ANS: The earache was due to spread of the infection causing swelling of the torus tubarius and
subsequent closing of the auditory tube. This forces the tympanic membrane to compensate for
pressure changes due to altitude or temperature, which may cause severe pain or even deafness.
As well as otitis media, is commonly associated with infection of the pharyngeal tonsil.

Head & Neck (4TH semester—Unit-I PRACTICAL MANUAL) Page 81


I.THE LARYNX
 The larynx is a respiratory organ, set in the respiratory tract between the pharynx and trachea
— Although phonation is important in man, the main function of the larynx is to provide a
protective sphincter for the air passages.
1. Framework of the Larynx (Fig—34)
 The fromework of the larynx consists of cartilages, ligaments and membranes — There are
three single cartilages (thyroid, cricoid and epiglottic) and three pairs of cartilages (arytenoid,
corniculate and cuneiform) —
a. Cartilages of the larynx
 1. THYROID CARTILAGE: Its two laminae are fused at a median angle, or laryngeal
prominence — A thyroid notch marks the upper end of the prominence — The posterior
borders are free and projected upwards and downwards as the superior and inferior horns —
Each inferior horn articulates with the cricoid cartilage — The outer surface of each lamina
possesses an oblique ridge, which gives attachment to inferior pharyngeal constrictor,
sternothyroid and thyrohyoid muscles…..2. CRICOID CARTILAGE: It is the foundation of
the larynx — To this signet-ring structure the thyroid and arytenoid cartilages are articulated —
It is the only complete cartilaginous ring in the whole of the air passages — The anterior part of
the ring is the arch & posterior part of the ring is the lamina — The upper part of the lamina
carry articular facets for arytenoid cartilages…..3. EPIGLOTTIS: It is a leaf-shaped
structure — attached in the midline to the back of the laryngeal prominence by thyro-
epiglottic ligament — Higher up, attached to the back of the body and greater cornu of the
hyoid bone, by hyo-epiglottic ligament — Still higher up, attached to the back of the posterior
surface of the tongue, by median glosso-epiglottic ligament and two lateral glosso-
epiglottic ligaments— Between median glosso-epiglottic and lateral glosso-epiglottic ligaments
is a loose fold of mucous membrane, called the valleculla — It leans back to overhang the
vestibule of the larynx…..4. ARYTENOID CARTILATGE: Three sided pyramid — The base
has a forward projection, the vocal process and a lateral projection, the muscular process —
The base articulates with the cricoid lamina…..5. CORNICULATE CARTILAGE: It articulates
with the apex of each arytenoid cartilage…..6. CUNEIFORM CARTILAGE: It lies in the
aryepiglottic fold.
b. Ligaments & membranes of the larynx
 1. THYRO-HYOID MEMBRANE: It connects the whole length of the upper border of the
laminae and superior horns of thyroid cartilage to the body and greater horns of the hyoid
bone — It forms the lateral wall of the piriform recess — It is perforated by the internal
laryngeal nerve and the superior laryngeal artery…..2. QUADRANGULAR MEMBRANE: It is
a fibroelastic membrane that extends between the anterolateral surface of the arytenoids
posteriorly and the side of the lower half of the epiglottis anteriorly — Its lower border is
free, constituting the vestibular ligament (‘false vocal cord’) — The mucous membrane covering
its upper border constitutes the aryepiglottic fold…..3. CRICOTHYROID MEMBRANE: It is
composed of mainly elastic tissue — connecting the upper border of the cricoid to the lower
border of the thyroid — has a free, thickened superior edge, attached in front to the thyroid
notch, and at the back to the vocal process that constitutes the vocal ligament or vocal cord
— There is a gap between the vestibular and vocal ligaments — quadrangular and
cricothyroid membranes are lined on their inner aspects by mucouos membrane, and that
part of it which covers the vestibular and vocal ligaments forms the vestibular and vocal folds
respectively.
2. Cavity of the Larynx
a. Laryngeal inlet
 The inlet (aditus) of the larynx, through which it communicates with the pharynx, faces
backwards and upwards and is bounded: in front by the upper edge of the epiglottis — at the
sides and back by the aryepiglottic folds, and in the posterior midline by the transverse
mucosal fold between the aryteroids.

Head & Neck (4TH semester—Unit-I PRACTICAL MANUAL) Page 82


b. The vestibule (1st part) of the larynx
 The space below the level of the inlet down as far as the vestibular folds is the vestibule.
c. The ventricle (2nd part) of the larynx
 In the gap between the vestibular and vocal ligaments, the mucous membrane of the larynx
bulges outwards, forming a deep horizontal groove, the ventricle or laryngeal sinus —
Opening from its anterior end is a small pouch of mucous membrane, the laryngeal saccule —
The gap between the vocal folds is the rima of the glottis (or simply ‘the glottis’)
o 1. THE ANTERIOR 60% OF THE GLOTTIS (INTERMEMBRANOUS PART) is
bounded on each side by the vocal folds itself.
o 2. THE POSTERIOR 40% (INTERCARTILAGINOUS PART) lies between the vocal
processes of the arytenoids cartilages and the medial margins of their bases.
d. The infraglottic (3rd part) of the larynx
 Below the glottis, the infraglottic part of the larynx ends down to the level of the lower
border of the cricoid cartilage where it becomes continuous with the trachea.
3. Muscles of the Larynx (Fig—35)
a. Muscles that alter the shape of laryngeal inlet
 1. ARYEPIGLOTTIC MUSCLE: Some fibres of oblique arytenoids continue from the arytenoids
apex into the aryepiglottic fold and reach the edge of the epiglottis, so forming the aryepiglottic
muscle — It closes the laryngeal inlet…..2. THYROEPIGLOTTIC MUSCLE: Many
thyroarytenoid fibres ascend up to the aryepiglottic fold and some even reach the side of the
epiglottis. They constitute the thyroepiglottic muscle — They open the laryngeal inlet by
abducting the aryepiglottic fold.
b. Muscles that cause the movements of vocal cords
 3. POSTERIOR CRICOARYTENOID: It aries fom the back of the lamina of the cricoids and
are inserted onto the back of the muscular process of the arytenoids — It is the ONLY
MUSCLE THAT ABDUCTS THE VOCAL FOLDS AND OPENS THE GLOTTIS…..4.
TRANSVERSE ARYTENOIDS: It is attached to the posterior surfaces of the aryteroid
cartilages — It draws the vocal processes nearer to each other and adducts the vocal folds,
helping to close the glottis….. 5. OBLIQUE ARYTENOIDS: It arises from the back of the
muscular process of one arytenoids and inserts to the apex of the opposite arytenoid —
Adducts the vocal cords by drawing the two arytenoids closer to each other…..6. LATERAL
CRICOARYTENOID: It arises from the upper border of the cricoid arch, passes upwards and
backwards to be attached to the front of the muscular process of the ipsilateral arytenoids — It
rotates the vocal process inwards and closes the glottis.
c. Muscles that change the tension of vocal cords
 7. CRICOTHYROID: It arises from the lateral aspect ot the cricoid arch and is inserted onto the
inferior horn and adjacent lower border of thyroid lamina — Its makes thyroid tilt slightly
downwards and forwards, thereby lengthening and tensing vocal ligament…..
8. THYROARYTENOID: It arises from the angle of the thyroid and is inseted onto the
anterolateral surface of the arytenoids — It shortens and relaxes the vocal ligament,
thereby altering the pitch of the voice…..9. VOCALIS: A part of thyroarytenoid, runs parallel
and lateral to the vocal ligament, and some of its fibres arise from the ligament rather than the
thyroid; these form the vocalis muscle — Acts on the posterior part of the vocal ligament
II.CLINICAL CORRELATIONS
1. Edema of the laryngeal mucous membrane
 The mucous membrane of the larynx is loosely attached to the underlying structures by
submucous connective tissue.
 In the region of the vocal folds, however, the mucous membrane is firmly attached to the vocal
ligaments.
 The accumulation of tissue fluid causes the mucous membrane above the rima glottidis to
swell and encroach on the airway. In severe cases, a cricothyroidotomy or tracheostomy may
be necessary.
2. Reflex activity secondary to endotracheal intubation

Head & Neck (4TH semester—Unit-I PRACTICAL MANUAL) Page 83


 Stimulation of the mucous membrane of the upper airway during the process of intubation may
produce cardiovascular changes such as bradycardia and hypotension. These changes are
largely mediated through the branches of the vagus nerves.

3. Tracheostomy
 Tracheostomy is rarely performed and is limited to patients with extensive laryngeal damage
and infants with severe airway obstruction.
 Because of the presence of major vascular structures (carotid arteries and internal jugular vein),
the thyroid gland, nerves (recurrent laryngeal branch of vagus and vagus nerve), the pleural
cavities, and the esophagus, meticulous attention to anatomic detail has to be observed.
 .

Head & Neck (4TH semester—Unit-I PRACTICAL MANUAL) Page 84


Fig-34: The larynx and its ligaments from the front (A), from the lateral aspect (B), and from
behind (C). D. The left lamina of thyroid cartilage has been removed to display the interior of
the larynx.

Head & Neck (4TH semester—Unit-I PRACTICAL MANUAL) Page 85


Fig-35: Muscles of the larynx seen from behind. B. Coronal section through the larynx. C.
Rima glottidis partially open as in quiet breathing. D. Rima glottidis wide open as in deep
breathing. E. Muscles that move vocal ligaments

Head & Neck (4TH semester—Unit-I PRACTICAL MANUAL) Page 86


III.REVIEW QUESTIONS
Q.1: A 70-YEAR-OLD MAN IS ADMITTED TO THE HOSPITAL WITH SEVERE HEADACHES.
DURING PHYSICAL EXAMINATION HE HAS DIFFI CULTY COUGHING AND SWALLOWING. A
CT SCAN SHOWS A TUMOR AFFECTING A CRANIAL NERVE. WHICH NERVE IS MOST LIKELY
AFFECTED?
A. Mandibular
B. Maxillary
C. Glossopharyngeal
D. Vagus
E. Hypoglossal
ANSWER: D.
 The vagus nerve is responsible for sensation in the mucosa of the larynx down to the level of the
vocal folds, and also motor innervation of the muscles that initiate a cough reflex and swallowing
(motor).
Q.2: A 35-YEAR-OLD FEMALE IS UNDER GENERAL ANESTHESIA. PRIOR TO LARYNGEAL
INTUBATION THE RIMA GLOTTIDIS IS OPENED BY WHICH PAIR OF MUSCLES?
A. Posterior cricoarytenoids
B. Lateral cricoarytenoids
C. Thyroarytenoids
D. Transverse arytenoids
E. Cricothyroids
ANSWER: A.
 The posterior cricoarytenoid muscles lie on the superoposterior aspect of the lamina of the
cricoids cartilage. When these muscles contract, they cause lateral rotation (abduction) of the
vocal processes of the arytenoid cartilages, thereby opening the space between the vocal folds,
the rima glottidis.
Q.3: A 38-YEAR-OLD MAN IS ADMITTED TO THE HOSPITAL WITH A LARGE MASS IN HIS
LOWER ANTERIOR NECK. ULTRASONIC EXAMINATION REVEALS A BENIGN TUMOR OF HIS
THYROID GLAND. TWENTY-FOUR HOURS FOLLOWING A PARTIAL THYROIDECTOMY, IT
WAS NOTED THAT THE PATIENT COULD NOT ABDUCT THE TRUE VOCAL CORDS DUE TO A
NERVE INJURY DURING THE OPERATION. WHICH OF THE FOLLOWING MUSCLES WAS
MOST LIKELY DENNERVATED?
A. Posterior cricoarytenoid
B. Lateral cricoarytenoid
C. Thyroarytenoid
D. Arytenoid
E. Cricothyroid
ANSWER: A.
 The posterior cricoarytenoids are the only muscle of the larynx that abducts the vocal cords. The
remaining answer choices are muscles that act in adduction of the vocal cords.
Q.4: A 54-YEAR-OLD MALE IS TO UNDERGO BILATERAL THYROIDECTOMY. DURING THIS
PROCEDURE THERE IS THE POSSIBILITY OF BILATERAL PARALYSIS OF MUSCLES THAT
CAN OPEN THE AIRWAY. IF A PARTICULAR NERVE IS INJURED BILATERALLY, THERE IS
SIGNIFI CANT RISK OF ASPHYXIATION POSTOPERATIVELY UNLESS THE PATIENT IS
INTUBATED OR THE AIRWAY IS OPENED SURGICALLY. WHICH OF THE FOLLOWING
MUSCLE PAIRS OPENS THE AIRWAY?
A. Cricothyroids
B. Posterior cricoarytenoids
C. Arytenoideus
D. Thyroarytenoids
E. Lateral cricoarytenoids
ANSWER: B.
 The posterior cricoarytenoid muscle is the only abductor of the larynx that opens the rima glottidis
and rotates the arytenoid cartilages laterally. All of the other listed muscles have adduction as part
of their function and thus are not required to maintain the airway.

Head & Neck (4TH semester—Unit-I PRACTICAL MANUAL) Page 87


Q.5: A 32-YEAR-OLD WOMAN IS UNDERGOING A THYROIDECTOMY. TWO MONTHS
POSTOPERATIVELY THE PATIENT SUFFERS FROM LOSS OF SENSATION WITHIN THE
LARYNX FROM THE VOCAL FOLDS UPWARD TO THE ENTRANCE INTO THE LARYNX,
ALLOWING FOR ASPIRATION OF LIQUIDS INTO THE AIRWAY. WHICH OF THE FOLLOWING
NERVES IS MOST LIKELY INJURED?
A. Internal laryngeal nerve
B. External laryngeal nerve
C. Glossopharyngeal nerve
D. Hypoglossal nerve
E. Recurrent laryngeal nerve
ANSWER: A.
 If there is an injury to the internal laryngeal nerve, there is a loss of sensation above the vocal
cords. In this case, for internal laryngeal injury to occur, one must conclude that the operative field
extended above the position of the thyroid gland to the level of the thyrohyoid membrane.
Q.6: A 34-YEAR-OLD FEMALE IS ADMITTED TO THE HOSPITAL BECAUSE OF HOARSENESS
FOR THE PAST 3 MONTHS. RADIOGRAPHIC EXAMINATION REVEALS A CANCEROUS
GROWTH IN HER LARYNX WITH NO EVIDENCE OF METASTASIS. IN ADDITION, THE AREA IN
WHICH THE TUMOR IS GROWING IS CHARACTERIZED BY VERY LIMITED LYMPHATIC
DRAINAGE. WHICH OF THE FOLLOWING LOCATIONS IS MOST LIKELY TO CONTAIN A
TUMOR WITH THESE CHARACTERISTICS?
A. A nterior commissure of the vocal ligaments
B. I nterarytenoid fold
C. Laryngeal ventricle
D. Cricothyroid ligament
E. Middle segment of the vocal cord
ANSWER E.
 The middle of the vocal cord would be the most likely location of the tumor because there is no
direct lymph drainage from this region.
Q.7: A 60-YEAR-OLD MAN PRESENTS WITH A SWELLING IN HIS NECK. PHYSICAL
EXAMINATION AND BIOPSY SHOW A BENIGN TUMOR IN HIS PIRIFORM RECESS. THE
MUCOSA OF THE PIRIFORM RECESS MUST BE ANESTHETIZED DURING THE REMOVAL OF
THE TUMOR. WHICH NERVE SUPPLIES GENERAL SENSATION TO THE MUCOUS MEMBRANE
OF THE LARYNGEAL VESTIBULE AND PIRIFORM RECESSES?
A. External laryngeal
B. Glossopharyngeal
C. Hypoglossal
D. Inferior laryngeal
E. Internal laryngeal
ANSWER E.
 The internal branch of the superior laryngeal nerve, often called the internal laryngeal nerve,
supplies the mucosa of the larynx above the vocal folds (which includes the vestibule of the
larynx) and the piriform recess. The external branch of the superior laryngeal nerve (external
laryngeal nerve) is motor to the cricothyroid muscle. The inferior laryngeal nerve supplies the
mucosa of the larynx below the vocal folds.
Q.8: A PATIENT WAS UNABLE TO SPEAK IN THE POSTOPERATIVE PERIOD FOLLOWING
THYROID SURGERY. INDIRECT LAYNGOSCOPY SHOWS THE VOCAL CORDS IN MEDIAL
POSITION. PARALYSIS TO WHICH OF THE FOLLOWING MUSCLES HAS OCCURRED:

A. Posterior cricoarytenoid
B. Lateral crico-arytenoid
C. Inter-arytenoid
D. Cricothyroid
E. Thyroarytenoid
ANSWER: A.
Q.9: DURING THE INDIRECT LARYNGOSCIOPY, THE PHYSICIAN NOTICES THE WEAKNESS IN
SEPARATION OF VOCAL CORDS. THE MOST LIKELY WEAKNESS IS IN:

Head & Neck (4TH semester—Unit-I PRACTICAL MANUAL) Page 88


A. Tranverse arytenoid
B. Oblique arytenoid
C. Posterior cricoarytenoid
D. Cricothyroid
E. Thyroarytenoid
ANSWER: C.
Q.10: A PATIENT WITH AN ANEURYSM OF THE AORTIC ARCH DEVELOPS HOARSENESS OF
VOICE. PARALYSIS OF WHICH OF THE FOLLOWING MUSCLES ON THE LEFT SIDE IS MOST
LIKELY:

A. Anterior belly of the digastric


B. Cricothyroid
C. Omohyoid
D. Posterior belly of the digastrics
E. Posterior cricoarytenoid
ANSWER: E.
Q.11: DURING DIRECT LARYNGOSCOPY, THERE IS ONE AREA OF THE LARYNX WHICH
CANNOT BE SEEN EVEN WITH DIRECT LARYNGOSCOPE, WHAT IS THAT AREA:

A. Piriform sinus
B. Valleculae
C. Laryngeal ventricle
D. Rima glottis
E. Vestibular fold
ANSWER: C.
Q.12: WHILE PERFORMING A SUBTOTAL THYROIDECTOMY, A SURGEON INADVERTENTLY
SECTIONS THE RECURRENT LARYNGEAL NERVE. WHICH OF THE FOLLOWING MUSCLES
WOULD RETAIN ITS INNERVATIONS SUBSEQUENT TO THIS INJURY:

A. Cricothyroid
B. Lateral cricoarytenoid
C. Posterior cricoarytenoid
D. Thyroarytenoid
E. Vocalis
ANSWER: A.
Q.13: WHILE PERFORMING A SURGICAL PROCEDURE IN THE NECK, THE SURGEON
LIGATES THE SUPERIOR LARYNGEAL ARTERY. IF THE NERVE THAT ACCOMPANIES THIS
ARTERY IS DAMAGED, WHICH OF THE FOLLOWING FUNCTIONAL LOSSES WILL ENSURE:

A. Loss of sensation in the laryngeal mucosa above the vocal folds


B. Loss of sensation in the laryngeal mucosa below the vocal folds
C. Loss of sensation in the pharyngeal mucosa
D. Paralysis of the cricothyroid muscle
E. Paralysis of the lateral cricoarytenoid muscle
ANSWER: A.
Q.14: A LESION OF THE EXTERNAL LARYNGEAL BRANCH OF THE SUPERIOR LARYNGEAL
NERVE MAY CAUSE WEAKNESS OF WHICH OF THE FOLLOWING MUSCLES:

A. Inferior pharyngeal constrictor muscle.


B. Middle pharyngeal constrictor muscle.
C. Superior pharyngeal constrictor muscle.
D. Thyroarytenoid muscle
E. Thyrohyoid muscle.
ANSWER: A.
Q.15: WHICH OF THE FOLLOWING PAIRS OF MUSCLES IS MOST INSTRUMENTAL IN
PREVENTING FOOD FROM ENTERING THE LARYNX AND TRACHEA DURING SWALLOWING:

Head & Neck (4TH semester—Unit-I PRACTICAL MANUAL) Page 89


A. Sternohyoid and sternothyroid muscles
B. Oblique arytenoids and aryepiglottic muscles
C. Inferior pharyngeal constrictor and Thyrohyoid muscles
D. Levator veli palatini and tensor veli palatini muscles
E. Musculus uvulae and geniohyoid muscles.
ANSWER: B.
Q.16: A HORIZONTAL CUT THROUGH THE CRICOTHYROID LIGAMENT IN THE NECK WOULD
SEVER WHICH OF THE FOLLOWING STRUCTURES:

A. Inferior laryngeal nerves


B. External carotid arteries
C. Inferior thyroid veins
D. Thyrocervical trunks
E. Internal laryngeal nerves.
ANSWER: A.
Q.17: A PATIENT WAS UNABLE TO SPEAK IN THE POSTOPERATIVE PERIOD FOLLOWING
THYROIDECTOMY. INDIRECT LARYNGOSCOPY SHOWS THE VOCAL CORDS IN MEDIAL POSITION.
PARALYSIS TO WHICH OF THE FOLLOWING MUSCLES HAS OCCURRED:

A. Posterior cricoarytenoid
B. Lateral crico-arytenoid
C. Inter-arytenoids
D. Cricothyroid
E. Thyrorarytenoid

ANSWER: A.
Q.18: A PATIENT CAME IN EMERGENCY ROOM WITH SEVERE LARYNGEAL OBSTRUCTION.
A YOUNG RESIDENT DOCTOR GAVE INCISION IN TRACHEA AT THE LEVEL OF:

A. Cricoids cartilage & 1st tracheal ring


B. 1st & 2nd tracheal rings
C. 2nd and 3rd tracheal rings
D. 3rd and 4th tracheal rings
E. 4th and 5th tracheal rings

ANSWER: C.
Q.19: THE CHANCES OF INJURY TO THE LEFT RECURRENT LARYNGEAL NERVE DURING AN
OPERATION OF THE THYROID GLAND ARE HIGH, BECAUSE THE LEFT RECURRENT NERVE:

A. Hooks around the arch of aorta


B. Is buried in the thyroid tissue
C. Lies more superficial in the groove between esophagus and trachea
D. Lies within pretracheal fascia
E. Passes between the branches of the inferior thyroid artery

ANSWER: C.
Q.20: A SURGEON HAS OPERATED UPON A PATIENT FOR THYROID ANOMALY.
IMMEDIATELY AFTER THE OPERATION THE PATIENT DEVELOPS DYSPNEA TO AN EXTENT
THAT AN EMERGENCY TRACHEOSTOMY HAD TO BE DONE TO SAVE HIS LIFE. WHICH OF
THE FOLLOWING NERVES WAS INJURED:

A. Right recurrent laryngeal


B. Left recurrent laryngeal
C. Both recurrent laryngeal
D. Right superior laryngeal
E. Left external laryngeal

ANSWER: C.
IV.CASE STUDY
Case—1
Head & Neck (4TH semester—Unit-I PRACTICAL MANUAL) Page 90
A 35-YEAR-OLD WOMAN HAD A PARTIAL THYROIDECTOMY FOR THE TREATMENT OF
THYROTOXICOSIS. DURING THE OPERATION A LIGATURE SLIPPED OFF THE RIGHT
SUPERIOR THYROID ARTERY. TO STOP THE HEMORRHAGE, THE SURGEON BLINDLY
GRABBED FOR THE ARTERY WITH ARTERY FORCEPS. THE OPERATION WAS COMPLETED
WITHOUT FURTHER INCIDENT. THE FOLLOWING MORNING THE PATIENT SPOKE WITH A
HUSKY VOICE.

1. What is the cause of the huskiness of the voice?


ANS: Laryngoscopic examination revealed that the right vocal cord was slack, causing the huskiness
of the voice.

2. What is the action of cricothyroid muscle?


ANS: The vocal cord is tensed by the contraction of the cricothyroid muscle.

3. How the cricothyroid functions to tense the vocal cords?

ANS: The cricothyroid muscle tilts back the cricoid cartilage and pulls forward the thyroid cartilage.

4. What is the nerve supply of cricothyroid?


ANS: The cricothyroid muscle is innervated by the external laryngeal nerve.

Case—2
A 35-YEAR-OLD WOMAN COMPLAINS OF A 2-MONTH HISTORY OF HOARSENESS OF HER
VOICE AND SOME CHOKING WHILE DRINKING LIQUIDS. SHE DENIES VIRAL ILLNESSES.
SHE UNDERWENT SURGERY FOR A COLD NODULE OF THE THYROID GLAND 9 WEEKS
AGO. HER ONLY MEDICATION IS ACETAMINOPHEN WITH CODEINE.
1. What is most likely diagnosis?
ANS: Injury to the recurrent laryngeal nerve causing the vocal cord paralysis.
2. Why is the hoarseness of voice?
ANS: The recurrent laryngeal nerve provides motor innervation to the larynx and sensory innervation
to the laryngeal mucosa. A traction injury or inadvertent severing of the nerve leads to vocal cord
paralysis. With injury to just one nerve, the vocal cord on the same side bows into a paramedian
position instead of closing straight to the midline, leading to hoarseness of voice.
3. Give brief explanation, in this case, for chocking during drinking liquids ?
ANS: Injury to the recurrent laryngeal nerve may affect the protective function of the rima glottidis,
increasing the opportunity for a choking response.
4. What is the cause of hypocalcemia?
ANS: There are four small parathyroid glands within the thyroid tissue. These tiny parathyroid glands
secrete parathyroid hormone to maintain calcium balance. Inadvertent injury due to excision of the
parathyroid glands can lead to hypocalcemia, manifested by fatigue, dyspnea (shortness of breath),
brittle skin and nails, tetanic muscle contractions, seizures, or difficulty swallowing.
5. Which of the following muscles is most important to allow air movement through the
larynx?
ANS: The posterior cricoarytenoid muscles are the only muscles that abduct the vocal folds and are
necessary to widen the rima glottidis for breathing.
6. Why the right recurrent laryngeal nerve carries less chances to be injured as compared
to the left?
ANS: The right recurrent nerve is located more laterally than the left recurrent nerve because of the
course of the right subclavian artery.
7. Name the muscles which abduct the vocal folds and are necessary to widen the rima
glottidis for breathing.
ANS: The posterior cricoarytenoids are the only muscles to abduct the vocal folds and are necessary
to widen the rima glottidis for breathing.
8. What is the outcome of Bilateral and unilateral injury to the recurrent laryngeal nerves?
ANS: Bilateral injury to the recurrent laryngeal nerves may lead to respiratory distress, whereas
unilateral injury results in hoarseness.

Head & Neck (4TH semester—Unit-I PRACTICAL MANUAL) Page 91


I.THE EYEBALL
1. Fascial sheath of the eyeball
 It envelops the eyeball from the optic nerve nearly to the corneoscleral junction — It is pierced
by the tendons of the extraocular muscles and is reflected onto each of them as a tubular
muscle sheath — The muscle sheaths of the levator and superior rectus muscles are fused;
thus, when the gaze is directed superiorly, the superior eyelid is further elevated — Triangular
expansions from the sheaths of the medial and lateral rectus muscles, called the medial and
lateral check ligaments, are attached to the lacrimal and zygomatic bones, respectively — The
medial and lateral check ligaments limit abduction and adduction — A blending of the check
ligaments with the fascia of the inferior rectus and inferior oblique muscles forms the suspensory
ligament of the eyeball — A potential episcleral space between the eyeball and the fascial
sheath allows the eyeball to move inside the cup-like sheath — A similar check ligament from
the fascial sheath of the inferior rectus retracts the inferior eyelid when the gaze is directed
downward.
2.Eyelids (Fig—37)
 Their fibrous framework is the orbital septum, thickened at the margins of the lids to form the
tarsal plates
 MEDIAL AND LATERAL CANTHI: The eyelids meet at the medial and lateral angles (or
canthi) — The lateral canthus is in direct contact with the eyeball — The medial canthus is
separated by the lacus lacrimalis.
 ORBITAL SEPTUM: It is the continuation of the periosteum lining the walls of the orbit &
attached to the margins of the orbit — It is greatly thickened above and below to form
superior and inferior tarsal plates.
 TARSAL PLATES: The levator palpebrae superioris is attached to the superior tarsal plate.
GLANDS OF TARSAL PLATES:
o 1. Tarsal (Meibomian) glands are modified sebaceous glands embedded within
the substance of the tarsal plates — Their ducts discharge an oily secretion at
the eyelid margin, this delays evaporation of tears and discourages spilling of
excess tears — The eyelashes are located here, they do not possess arrector pili
muscles.
o 2. Sebaceous glands (Glands of Zeis) open into each hair follicle
o 3. Ciliary glands (or glands of Moll) between the follicles, open on the eyelid
margin.
 CONJUNCTVA: Mucus-secreting goblet cells are scattered in the conjunctival epithelium
and small accessory lacrimal glands are scattered in the subconjunctival connective tissue —
MUSCLES OF THE EYELIDS:
o 1. Levator palpabrae superioris: Origin: Back of orbital cavity — Insertion:
Anterior surface and upper margin of superior tarsal plate — Nerve supply: Striated
part by oculomotor nerve & smooth part by sympathetic nerves — Actions: Raises
upper lid.
o 2.Orbicularis oculi
o 3. Frontal bellies of occipito-frontalis
3.Lacrimal gland and apparatus (Fig—37)
 LACRIMAL GLAND: A serous gland with a large orbital part (in the lacrimal fossa on the
lateral part of the roof of the orbit) and a small palpebral part, both separated by aponeurotic
tendon of levator palpebrae superioris — The gland drains by a dozen ducts into the lateral
extent of the superior fornix — Closure of the eyelids spreads tears across the eye — Under
normal conditions the lacrimal gland secretes just enough tears to replace those lost by
evaporation.
 LACRIMAL CANALICULI: At the medial end of each lid margin is a low elevation, the lacrimal
papilla, surmounted by a minute lacrimal punctum — Lacrimal punctum opens into a lacrimal
canaliculus, which conveys tears to the lacrimal sac.

Head & Neck (4TH semester—Unit-I PRACTICAL MANUAL) Page 92


 LACRIMAL SAC: The lacrimal sac lies in the lacrimal groove formed by the maxilla and
lacrimal bone.
 NASOLACRIMAL DUCT: The lacrimal sac continues into nasolacrimal duct, that slopes
downwards in the nasal cavity, to open in the inferior meatus — The mucous membrane is
raised into several variable folds which act as valves to prevent air being blown up the duct
into the lacrimal sac.

Fig-36: Right eyeball exposed from in front..

Head & Neck (4TH semester—Unit-I PRACTICAL MANUAL) Page 93


Fig-37: “Right eye, with the eyelids separated to show the openings of the tarsal glands, plica
semilunaris, caruncula lacrimalis, and puncta lacrimalis. B. Left eye, showing the superior and
inferior tarsal plates and the lacrimal gland, sac, and duct. Note that a small window has been
cut in the orbital septum to show the underlying lacrimal gland and fat (yellow). C. Sagittal
section through the upper eyelid, and the superior fornix of the conjunctiva. Note the presence
of smooth muscle in the levator palpebrae superioris”

4. Structure of the eyeball: (fig: 38)


 It consists of tunica, optical and neural structures.
 The tunica form the outer and inner walls of the eyeball and consist of three layers—fibrous,
vascular and neural.

Head & Neck (4TH semester—Unit-I PRACTICAL MANUAL) Page 94


o The outer fibrous tunica consists of the opaque sclera (white of the eye) behind,
and the transparent cornea in front.
o The middle vascular tunica consists of the choroid behind, and the ciliary body
and the iris in front.
o The inner neural tunica consists of the retina.

Fig-38: Horizontal section through the eyeball and the optic nerve.

a. Outer fibrous tunica

 1. SCLERA: It is the posterior five-sixths of the outer coat — It is weakest at the entrance of
the optic nerve, whose perforating fibres give it a sieve-like appearance, the lamina cribrosa —
the lamina cribrosa bulges posteriorly (‘cupping’ of the disc) — The sheath of dura mater
around the optic nerve blends with the sclera — It receives the insertions of the ocular
muscles — It is pierced by the ciliary nerves and arteries around the entrance of the optic
nerve,and by the venae verticousae (the choroids veins) just behind the coronal equator…..
2. CORNEA: It bulges forward from the sclera at the corneoscleral junction or limbus — It
occupies the anterior one-sixth of the eye and is completely avascular…..3. CANAL OF
SCHLEMN: Just beyond the corneoscleral junction, within the sclera is a circularly running canal,
called the sinus venosus sclerae or the canal of Schlemn — Posterior to the canal is a
triangular projection, the sclera spur, pointing forwards and inwards, to which the ciliary muscle

Head & Neck (4TH semester—Unit-I PRACTICAL MANUAL) Page 95


is attached. The canal is connected with anterior scleral veins, into which it drains.

b. Intermediate vascular tunica

 1. CHOROID: Anteriorly it merges into the ciliary body — Posteriorly it is perforated by the
optic nerve, to which it is firmly attached — Its inner surface is firmly attached to the pigment
layer of the retina, and the choroid capillaries provide nutrition for the rods and cones of the
retina — The veins collect into four or five large venae vorticosae, which pass through the
sclera just behind the equator…..2. CILIARY BODY: The ciliary body is continuous with
choroid behind, and the iris in front — The scleral surface of the ciliary body contains the
ciliary muscle — The ciliary body appears smooth where it is continous with the choroid at
the ora serrata, but further forward this is projected into 70-80 small ciliary processes which lie
in reciprocal grooves on the anterior surface of the vitreous body — Ciliary Muscle: The ciliary
muscle consists of smooth muscle; its function is to focus the lens for the near vision…..
3. IRIS: It forms the angle of the anterior chamber — The iris is perforated centrally by the pupil
— The sphincter pupillae is a circular band of smooth muscle lying at the margin of the pupil
— The dilartor pupillae is a thin sheet of radial fibres of smooth muscle extending from the
ciliary body to the sphincter pupillae.

c. Inner nervous tunica (Retina)

 Its outer surface is attached to the choroid, and its inner surface is in contact with vitreous
body — The light-sensitive area ends abruptly at ora serrata — At the entrance of the optic
nerve, is a 1.5 mm circular pale area, the optic disc — The optic disc is excavated to a variable
degree, producing the physiological cup — There are no rods or cones in the optic disc, hence
it is insensitive to light, the physiologic blind spot — The disc and whole surrounding area as
seen with ophthalmoscope constitute the fundus of the eye — At the posterior pole of the eye (3
mm lateral to the optic disc) is a shallow depression, it is completely free of blood vessels and
is yellowish in colour, hence called the macula lutea — In the centre of the macula is a shallow
pit, the fovea centralis — This is the thinnest part of the retina. There are no blood vessels
and no rods here, but there is a high concentration of cones. It is the area of the most acute
vision — The outer layer of the retina consists of a single layer of pigmented epithelial cells
firmly attached to the choroids — Next to this layer lie the light receptors (rods and cones) —
The rods do not register colour, but are sensitive to dim light (Scotopic vision) — The
periphery of the retina contains rods only — The cones register colour (Photopic vision) —
Cones alone occupy the fovea centralis. Beyond this they share equally with the rods, but they
fall short of the periphery of the retina — The central artery of the retina passes through the
lamina cribrosa within the optic nerve and in the optic disc divides into an upper and lower
branches, which are END ARTERIES — They supply the neurons (bipolar and ganglion cells)
of the retina — The light receptors are supplied by diffusion from capillaries of choroid.

5. Refraction media
a. Cornea
 This is the anterior part of the fibrous tunica of the eye. It is transparent and projects anteriorly —
It bends light so that it enters the eye through the aperture of the pupil.
b. Aqueous humour
 It fills the anterior and posterior chambers of the eye and allows light to pass through it — The
aqueous humor is a clear fluid that lies between the back of the cornea and the front of the
lens — The space is divided by the iris into anterior and posterior chambers, which
communicate with each other through the pupil — Aqueous humor is produced by the ciliary
processes by diffusion from the capillaries and transported by the ciliary epithelium into the
posterior chamber, it passes through the pupil into the anterior chamber — At the margin of
the anterior chamber is the iridocorneal angle and here aqueous humour filters through
trabecular tissue into the canal of Schlemn — Obliteration of the angle therefore prevents
absorption of aqueous humor, with consequent rise of intraocular tension, leading to the condition
of glaucoma.

Head & Neck (4TH semester—Unit-I PRACTICAL MANUAL) Page 96


c. Lens
 The lens is a transparent biconvex body enclosed in a transparent elastic capsule — Its
posterior surface, resting on the vitreous, is more highly convex than the anterior — The
lens capsule is an elastic membrane that envelops the whole lens — The suspensory
ligament of the lens, or zonule, is a series of delicate fibrils attached to the ciliary processes.
d. Vitreous body
 The vitreous body is a colorless, jelly-like mass which occupies the posterior four-fifths of the
eyeball — It comprises about 99% water and a sparse cellular and fibrous content.
5. Extraocular Muscles (Fig—39)
a. Superior rectus
 ORIGIN: The tendinous ring — INSERTION: Superior surface of sclera of the eyeball —
NERVE SUPPLY: Oculomotor nerve (third cranial nerve) — ACTIONS: Raises cornea upward
and medially. For superior rectus to raise the cornea directly upward, inferior oblique muscle
must assist.
b. Inferior rectus
 ORIGIN: The tendinous ring — INSERTION: Inferior surface of sclera of the eyeball — NERVE
SUPPLY: Oculomotor nerve (third cranial nerve) — ACTIONS: Depresses cornea downward and
medially. For inferior rectus muscle to depress the cornea directly downward, superior
oblique muscle must assist.
c. Medial rectus
 ORIGIN: The tendinous ring — INSERTION: Medial surface of sclera of the eyeball — NERVE
SUPPLY: Oculomotor nerve (third cranial nerve) — ACTIONS: Rotates eyeball so that cornea
looks medially.
d. Lateral rectus
 ORIGIN: The tendinous ring — INSERTION: Lateral surface of sclera of the eyeball — NERVE
SUPPLY: Abducent nerve (sixth cranial nerve) — ACTIONS: Rotates eyeball so that cornea
looks laterally.
e. Superior oblique
 ORIGIN: From the body of the sphenoid bone — INSERTION: It comes forward to become a
tendon, and passes through a pulley (trochlea). The tendon, is inserted into the sclera —
NERVE SUPPLY: Trochlear nerve (fourth cranial nerve) — ACTIONS: It rotates the eyeball so
that cornea is depressed (down) and rotated laterally (out).
f. Inferior oblique:
 ORIGIN: From the floor of the orbit — INSERTION: It passes backward and laterally and is
inserted into the sclera — NERVE SUPPLY: It is supplied by oculomotor nerve — ACTIONS:
It rotates the eyeball so that cornea is raised (up) and rotated laterally (out).
II.CLINICAL CORRELATIONS
1. Eye trauma
 Careful examination of the eyeball relative to the orbital margins shows that it is least protected
from the lateral side.
 Blowout fractures of the orbital floor involving the maxillary sinus commonly occur as a
result of blunt force to the face.
 Not only can blowout fractures cause displacement of the eyeball, with resulting symptoms of
double vision (diplopia), but also the fracture can injure the infraorbital nerve, producing loss of
sensation of the skin of the cheek and the gum on that side.
 Entrapment of the inferior rectus muscle in the fracture may limit upward gaze.
2. Strabismus
 Many cases of strabismus are nonparalytic and are caused by an imbalance in the action of
opposing muscles.
 This type of strabismus is known as concomitant strabismus and is common in infancy.
3. Light reflexes
 The pupillary reflexes that is, the reaction of the pupils to light and accommodation depend
on the integrity of nervous pathways.

Head & Neck (4TH semester—Unit-I PRACTICAL MANUAL) Page 97


 a. In the direct light reflex the normal pupil reflexly contracts when a light is thrown into
the patient's eye…..The nervous impulses pass from the retina along the optic nerve to the
optic chiasma and then along the optic tract…..Before reaching the lateral geniculate body,
the fibers concerned with this reflex leave the tract and pass to the oculomotor nuclei on both
sides via the pretectal nuclei…..From the parasympathetic (Edinger-Wesphal) part of the
nucleus, efferent fibers leave the midbrain in the oculomotor nerve and reach the ciliary
ganglion via the nerve to the inferior oblique…..Postganglionic fibers pass to the constrictor
pupillae muscles via the short ciliary nerves.
 b. The consensual light reflex is tested by shining the light in one eye and noting the
contraction of the pupil in the opposite eye. This reflex is possible because the afferent
pathway just described travels to the parasympathetic nuclei of both oculomotor nerves.
4. Accomodation reflexes
 The accommodation reflex is the contraction of the pupil that occurs when a person
suddenly focuses on a near object after having focused on a distant object.
 The nervous impulses pass from the retina via the optic nerve, the optic chiasma, the optic
tract, the lateral geniculate body, the optic radiation, and the cerebral cortex of the occipital
lobe of the brain.
 The visual cortex is connected to the eye field of the frontal cortex…..From here, efferent
pathways pass to the parasympathetic (Edinger-Wesphal) nucleus of the oculomotor
nerve…..From there, the efferent impulses reach the constrictor pupillae via the oculomotor
nerve, the ciliary ganglion, and the short ciliary nerves.

5. Extraocular muscle testing


 For the superior and inferior recti, turning the eye outward (abduction) by approximately 25
degrees places the superior rectus in position to raise the eye and the inferior rectus to lower the
eye.
 Similarly, turning the eye inward (adduction) approximately 51 degrees places the inferior
oblique in position to raise the eye and the superior oblique to lower the eye.
 The medial and lateral recti may be checked while the eye is staring straight ahead since they
have simple planar actions.

Head & Neck (4TH semester—Unit-I PRACTICAL MANUAL) Page 98


Fig-39: Muscles and nerves of the right orbit viewed from the lateral side. The maxillary
nerve and the pterygopalatine ganglion are also shown.
III.REVIEW QUESTIONS
Q.1: A 22-YEAR-OLD WOMAN IS ADMITTED TO THE HOSPITAL WITH AN INJURY TO HER
EYE. THE CORNEAL REFLEX IS TESTED AND FOUND TO BE PRESENT. WHICH OF THE
FOLLOWING NERVES IS RESPONSIBLE FOR THE AFFERENT LIMB OF THIS REFLEX?
A. Frontal
B. Lacrimal
C. Nasociliary
D. Oculomotor
E. Optic
ANSWER: C.
 The afferent/sensory limb of the corneal (blink) reflex is carried by the nasociliary nerve. It is a
branch of the ophthalmic division of the trigeminal nerve.
Q.2: A 47-YEAR-OLD WOMAN IS ADMITTED TO THE HOSPITAL WITH SIGNS OF CAVERNOUS
SINUS THROMBOSIS. RADIOGRAPHIC EXAMINATION REVEALS A PITUITARY TUMOR
INVOLVING THE CAVERNOUS SINUS, CONFIRMING THE INITIAL DIAGNOSIS. DURING
PHYSICAL EXAMINATION IT IS SUSPECTED THAT THE RIGHT ABDUCENS NERVE OF THE
PATIENT HAS BEEN DAMAGED BY THE TUMOR. IN WHICH DIRECTION WILL THE PHYSICIAN
MOST LIKELY ASK THE PATIENT TO TURN HER RIGHT EYE TO CONFIRM THE ABDUCENS
NERVE DAMAGE, ASSUMING SHE IS UNABLE TO PERFORM THIS TASK?
A. Inward
B. Outward
C. Downward
D. Down and out
E. Down and in
ANSWER: B.
 The right abducens nerve innervates the right lateral rectus, which mediates outward movement
(abduction) of the right eye.

Head & Neck (4TH semester—Unit-I PRACTICAL MANUAL) Page 99


Q.3: A 62-YEAR-OLD MAN IS ADMITTED TO THE HOSPITAL WITH BLURRED VISION.
PHYSICAL EXAMINATION REVEALS A LONG HISTORY OF GRADUAL LOSS OF HIS VISUAL
FIELD. THE INTRAOCULAR PRESSURE IS HIGH, AND A DIAGNOSIS OF GLAUCOMA IS MADE.
WHICH OF THE FOLLOWING SPACES FIRST RECEIVES THE AQUEOUS HUMOR SECRETED
BY THE EPITHELIUM OF THE CILIARY BODY?
A. Anterior chamber
B. Posterior chamber
C. Pupil
D. Vitreous
E. Lacrimal sac
ANSWER: B.
 The posterior chamber receives ciliary body secretions first. The ciliary body produces aqueous
humor and is located in the posterior chamber. Increased production of fluid from this site would
cause an increase in intraocular pressure if drainage is inadequate. The irido-scleral angle of the
anterior chamber is the location of drainage of the aqueous humor; therefore, a blockage of
drainage in this location can cause increased intraocular pressure.
Q.4: A 32-YEAR-OLD MAN IS ADMITTED TO THE EMERGENCY DEPARTMENT WITH VISUAL
PROBLEMS. RADIOGRAPHIC EXAMINATION REVEALS A TUMOR OF THE
ADENOHYPOPHYSIS (ANTERIOR PITUITARY GLAND). PHYSICAL EXAMINATION REVEALS A
LOSS OF THE LATERAL HALVES OF THE FIELDS OF VISION OF BOTH EYES (BITEMPORAL
HEMIANOPIA OR “TUNNEL VISION”). WHICH OF THE FOLLOWING STRUCTURES WAS MOST
LIKELY COMPRESSED BY THE TUMOR?
A. Optic nerve
B. Optic chiasm
C. Optic tract
D. Oculomotor
E. Abducens nerve
ANSWER: B.
 Compression of the optic chiasm can cause bitemporal hemianopia due to compression of nerve
fibers coming from the nasal hemiretinas of both eyes. The optic chiasm is located in very close
proximity above the pituitary gland.
Q.5: A 57-YEAR-OLD MAN IS ADMITTED TO THE EMERGENCY DEPARTMENT WITH
DIZZINESS AND SEVERE HEADACHES. A CT SCAN EVALUATION REVEALS A TUMOR IN THE
SUPERIOR ORBITAL FI SSURE. UPON PHYSICAL EXAMINATION THE PATIENT’S EYEBALL IS
FIXED IN AN ABDUCTED POSITION, SLIGHTLY DEPRESSED, AND THE PUPIL IS DILATED. IN
ADDITION, THE SUPERIOR PALPEBRAE ARE PTOTIC. WHEN THE PATIENT IS ASKED TO
MOVE THE PUPIL TOWARD THE NOSE, THE PUPIL ROTATES MEDIALLY. CONSENSUAL
CORNEAL REFLEXES ARE NORMAL. WHICH OF THE FOLLOWING NERVES IS MOST LIKELY
AFFECTED?
A. Trochlear nerve
B. Oulomotor nerve
C. Abducens nerve and sympathetic nerve plexus accompanying the ophthalmic artery
D. Ophthalmic nerve and short ciliary nerve
E. Superior division of oculomotor nerve and the nasociliary nerve
ANSWER: B.
 A lesion of the oculomotor nerve will cause the eye to remain in a “down and out” position. This is
due to the actions of the unopposed lateral rectus (supplied by the abducens nerve) and the
superior oblique (supplied by the trochlear nerve). The tertiary function of the superior oblique is
to cause intorsion (internal rotation) of the eyeball, a function that is not usually seen unless the
oculomotor nerve is paralyzed. The patient is also likely to present with a full or partial ptosis due
to paralysis of the levator palpebrae muscle. The pupil will remain dilated because of loss of
stimulation by parasympathetic fi bers that innervate the constrictor pupillae muscle.
Q.6: A 45-YEAR-OLD MAN WAS SUFFERING FROM TRIGEMINAL NEURALGIA (TIC
DOULOUREUX). THE PAIN WAS SO SEVERE THAT THE PATIENT HAD CONSIDERED SUICIDE
AS A WAY TO ESCAPE THE PAIN. EVEN LIGHT, GENTLE STIMULI TO THE SKIN BETWEEN
THE LOWER EYELID AND THE UPPER LIP RESULTED IN SEVERE, AGONIZING PAIN. IT WAS
DECIDED TO LESION THE NERVE BRANCH INVOLVED BY INJECTION OF ALCOHOL INTO
THE NERVE. TO REACH THE NERVE, THE NEEDLE WILL MOST LIKELY NEED TO BE
INSERTED THROUGH WHICH OF THE FOLLOWING OPENINGS?
A. Foramen ovale

Head & Neck (4TH semester—Unit-I PRACTICAL MANUAL) Page 100


B. Foramen spinosum
C. Infraorbital foramen
D. Mandibular foramen
E. Foramen magnum
ANSWER: C.
 The infraorbital branch of the maxillary division of the trigeminal nerve exits the front of the skull
below the orbit through the infraorbital foramen. A needle inserted into the infraorbital foramen
and directed posteriorly will pass through the foramen rotundum to reach the trigeminal ganglion
and the beginning of the maxillary division of the trigeminal nerve.
Q.7: A 73-YEAR-OLD MALE PATIENT VISITS THE OUTPATIENT CLINIC WITH A COMPLAINT
OF PROGRESSIVE, PAINLESS LOSS OF VISION. RADIOGRAPHIC EXAMINATION REVEALS
HROMBOPHLEBITIS OF THE CAVERNOUS SINUS. THROUGH WHICH OF THE FOLLOWING
STRUCTURES MUST A THROMBUS PASS TO CAUSE THE SYMPTOMS OF THIS PATIENT?
A. Subarachnoid space
B. Central artery of the retina
C. Central vein of the retina
D. Optic chiasm
E. Ciliary ganglion
ANSWER: C.
 The thrombus may pass through the central vein of the retina to reach the cavernous sinus. The
patient would suffer blindness because the central vein is the only vein draining the retina and if it
is occluded, blindness will ensue. The subarachnoid space would not be associated with the
blindness experienced.
Q.8: A 67-YEAR-OLD MAN VISITS THE OUTPATIENT CLINIC WITH COMPLAINTS OF
DETERIORATING VISION. A FORM OF GLAUCOMA IS DIAGNOSED IN WHICH THE AQUEOUS
HUMOR DOES NOT DRAIN PROPERLY INTO THE SCLERAL VENOUS SINUS AT THE
IRIDOSCLERAL ANGLE OF THE EYEBALL. THE AQUEOUS FL UID IS SECRETED BY THE
EPITHELIUM OF THE CILIARY BODY DIRECTLY INTO WHICH OF THE FOLLOWING SPACES?

A. Iridoscleral angle
B. Posterior chamber
C. Pupil
D. Vitreous body
E. Lacrimal sac
ANSWER: B.
 Aqueous humor is secreted by the ciliary body into the posterior chamber of the eye. The humor fl
ows through the pupil into the anterior chamber and then is fi ltered by a trabecular meshwork,
then drained by the canal of Schlemm. The pupil is the opening in the iris, which leads from the
posterior chamber to the anterior chamber. Vitreous humor, not aqueous humor, is found in the
vitreous body. The lacrimal sac is involved with tears, not the secretion of aqueous humor.
Q.9: DURING THE ROUTINE OPHTHALMOLOGIC EXAM, THE GLOBE, THE RETINA, AND THE
CORNEA OF EACH EYE ARE TESTED. WHICH OF THE FOLLOWING NERVES MUST BE
FUNCTIONING PROPERLY IF THE PATIENT IS TO BE ABLE TO TURN THE EYE LATERALLY
(ABDUCTION) WITHOUT DIFFI CULTY AND WITHOUT UPWARD OR DOWNWARD DEVIATION?
A. Superior division of oculomotor, ophthalmic nerve, abducens nerve
B. Trochlear nerve, abducens nerve, nasociliary nerve
C. Inferior division of oculomotor, trochlear, abducens
D. Oculomotor and ophthalmic nerves
E. Superior division of oculomotor, trochlear, and abducens nerves
ANSWER: C.
 For proper movements of the eye to occur, all cranial nerves of the extraocular eye muscles are
required (oculomotor, trochlear, and abducens nerves). The inferior division of the oculomotor
innervates the inferior rectus, the medial rectus, and the inferior oblique. Lateral movement of the
eye is initiated by the lateral rectus (abducens nerve), assisted thereafter by the superior oblique
(trochlear nerve). The inferior rectus (inferior division of the oculomotor nerve) balances the
upward deviation exerted by the superior rectus (superior division of the oculomotor nerve).
Q.10: A 12-YEAR-OLD BOY IS ADMITTED TO THE EMERGENCY DEPARTMENT WITH SIGNS
OF MENINGITIS. TO DETERMINE THE SPECIFI C TYPE OF MENINGITIS, IT IS NECESSARY TO

Head & Neck (4TH semester—Unit-I PRACTICAL MANUAL) Page 101


ASPIRATE CEREBROSPINAL FL UID WITH A LUMBAR PUNCTURE FOR LABORATORY
EXAMINATION. HOWEVER, BEFORE PERFORMING A LUMBAR PUNCTURE, IT MUST BE
ESTABLISHED THAT THE CEREBROSPINAL FL UID PRESSURE IS NOT ELEVATED. HAT
CONDITION IN THE EYE WOULD INDICATE THAT CEREBROSPINAL FL UID PRESSURE IS
TOO ELEVATED FOR A LUMBAR PUNCTURE TO BE PERFORMED?
A. Papilledema
B. Separation of the pars optica retinae anterior to the ora serrata
C. The foveal centralis exhibits hemorrhage from medial retinal branches.
D. Obvious opacity of the lens
E. Pitting or compression of the optic disc
ANSWER: A.
 Papilledema is optic disc swelling (“edema of the papilla”) that is caused by increased intracranial
pressure and increased CSF pressure. If a lumbar puncture is performed in a patient with
elevated CSF pressure and fl uid is withdrawn from the lumbar cistern, the brain can become
displaced caudally and the brainstem is pushed against the tentorial notch. This is a potentially
fatal complication.
Q.11: A 36-YEAR-OLD FEMALE RACQUETBALL PLAYER IS ADMITTED TO THE HOSPITAL
AFTER BEING STRUCK IN THE ORBITAL REGION. RADIOGRAPHIC EXAMINATION REVEALS
A BLOW-OUT FRACTURE OF THE MEDIAL WALL OF THE ORBIT. PHYSICAL EXAMINATION
ALSO REVEALS THAT THE PUPIL OF THE AFFECTED EYE CANNOT BE TURNED
LATERALLY. WHICH OF THE FOLLOWING MUSCLES IS MOST LIKELY INJURED OR
TRAPPED?
A. Lateral rectus
B. Medial and inferior recti
C. Medial rectus
D. Medial rectus and superior oblique
E. Inferior rectus
ANSWER: C.

 A blow-out fracture of the medial wall of the orbit would likely render the medial rectus
nonfunctional by entrapment of the muscle between the edges of the cracked medial wall. The
medial rectus is responsible for adduction of the eye, but in this case the muscle acts as a tether
or anchor on the eyeball, preventing lateral excursion (abduction) of the eye. There is no nerve
damage here, and the muscle is not paralyzed.
Q.12: A 16-YEAR-OLD FEMALE VOLLEYBALL PLAYER IS ADMITTED TO THE HOSPITAL
AFTER BEING HIT IN THE EYE WITH A BALL SPIKED AT THE NET. RADIOGRAPHIC
EXAMINATION REVEALS A BLOW-OUT FRACTURE OF THE INFERIOR WALL OF THE ORBIT.
PHYSICAL EXAMINATION ALSO REVEALS THAT THE PUPIL OF HER EYE CANNOT BE
TURNED UPWARD. WHICH OF THE FOLLOWING MUSCLES IS (ARE) MOST LIKELY INJURED?
A. Inferior rectus and inferior oblique
B. Medial and inferior recti
C. Inferior oblique
D. M edial rectus, inferior rectus, and inferior oblique
E. I nferior rectus
ANSWER: A.
 The inferior rectus and inferior oblique muscles are entrapped in the crack between the parts of
the fractured orbital floor. Normally, the superior rectus and the inferior oblique are responsible for
an upward movement of the eyeball. In this case, however, the broken orbital plate of the maxilla
has snared or entrapped the inferior rectus and inferior oblique muscles, causing them to act as
anchors on the eyeball, preventing upward movement of the eye. The muscles are not
necessarily damaged, nor is there any nerve injury in this patient. Freeing the muscles from the
bone will allow free movement of the eye again, barring any other injury.
Q.13: A 56-YEAR-OLD WOMAN IS ADMITTED TO THE HOSPITAL WITH EYE PAIN. DURING
PHYSICAL EXAMINATION THE PATIENT COMPLAINS OF EXCRUCIATING PAIN WHEN SHE
PERFORMS ANY MOVEMENT OF THE EYE. AN MRI EXAMINATION REVEALS THAT THE
OPTIC NERVE IS INFL AMED. WHAT IS THE MOST LIKELY EXPLANATION?
A. The anular tendon (of Zinn) is infl amed.
B. The infl ammation has affected the nerves innervating the eye muscles.
C. The muscles are contracting due to generalized infl ammation.

Head & Neck (4TH semester—Unit-I PRACTICAL MANUAL) Page 102


D. The nasociliary nerve is affected.
E. The ophthalmic artery is constricted.
ANSWER: A.
 The dural covering of the optic nerve is connected to the anular tendon; therefore, when there is
an infl ammation of the optic nerve, contractions of the recti can evoke severe pain.
Q.14: A 45-YEAR-OLD MAN CAME TO THE OUTPATIENT CLINIC AFTER STUMBLING AND
HITTING HIS HEAD ON A TABLE IN A RESTAURANT. DURING THE NEUROLOGIC
EXAMINATION PHOTOGRAPHS WERE TAKEN OF THE PATIENT’S EYES AS SHOWN IN FIG. 7-
8 . WHICH OF THE FOLLOWING NERVES TO THE LEFT EYE WAS MOST LIKELY INJURED?
A. Trochlea
B. Abducens
C. Oculomotor
D. Optic
E. Oculomotor and abducens
ANSWER: B.

I. THE EAR
1. External ear: (Fig—39)
 AURICLE OR PINNA: The auricle or pinna has a skeleton of resilient elastic cartilage —
EXTERNAL ACOUSTIC MEATUS: The external acoustic meatus is a sinuous tube — Its outer
third is cartilage, its inner two-thirds bone, in both zones the skin is firmly adherent — The
cartilaginous one-third is filled with fibrous tissue, hairs and sebaceous glands. Here also are the
ceruminous glands, long coiled modified sweat glands, which secrete yellowish-brown wax

Head & Neck (4TH semester—Unit-I PRACTICAL MANUAL) Page 103


Fig-39: Different parts of the auricle of the external ear. The arrow indicates the direction that
the auricle should be pulled to straighten the external auditory meatus before insertion of the
otoscope in the adult. B. External and middle portions of the right ear viewed from in front. C.
The right tympanic membrane as seen through the otoscope.

2. Middle ear: (Fig—39, 40 & 41)


a. Boundaries of middle ear cavity
 LATERAL WALL: Largely occupied by the tympanic membrane — MEDIAL WALL: its most
prominent feature is promontory, due to the first turn of the cochlea and indented with fine
grooves by the tympanic plexus — Above and behind the promontory is the oval window
(fenestra vestibuli), closed in life by the foot-piece of the stapes — Below and behind the
promonotory is the round window (fenestra cochleae), closed in life by the fibrous secondary
tympanic membrane — ROOF: The roof of the tympanum is the tegmen tympani — FLOOR:

Head & Neck (4TH semester—Unit-I PRACTICAL MANUAL) Page 104


The floor is a thin plate of bone above the jugular fossa — At the anterior end is a small
opening, called the tympanic canaliculus, which transmits the tympanic branch of the
glossopharyngeal nerve — ANTERIOR WALL: It is perforated by the openings of two
canals…..1.The lower and larger canal is the bony part of the auditory tube & 2.The upper
and smaller canal is the canal for the tensor tympani muscle — The lower part of this wall
forms the posterior wall of the carotid canal and is perforated by tympanic branches of the
internal carotid artery and sympathetic fibres from the internal carotid plexus —
POSTERIOR WALL: The posterior wall is deficient above, where there is an aperture, the aditus,
which leads back into the tympanic antrum — Below the aditus a hollow cone, the pyramid, that
projects into the tympanic cavity, its apex is perforated by the tendon of stapedius — Close to the
posterior margin of the tympanic membrane is tiny posterior canaliculus for the chorda tympani.
b. Auditory ossicle
 The auditory ossicles form by synovial joints, a bony chain for transmission of vibrations from
the tympanic membrane to the internal ear — 1. MALLEOUS: The handle projects backwards
down to the umbo — 2. INCUS: — 3. STAPES: Footpiece is attached to the oval window by
an annular ligament.
c. Muscles of middle ear
 1. TENSOR TYMPANI: Origin: Canal in the anterior wall of the tympanic cavity — Insertion:
Into the handle of the malleus — Nerve supply: Via nerve to medial pterygoid — Actions:
Making the drum more tense — 2. STAPEDIUS:
d. Tympanic membrane
 Two parts of tympanic membrane: 1. PARS FLACCIDA: Between anterior & posterior malleolar
fold, the small upper segment of the membrane is lax (par flaccida, shrapnell’s membrane) —
This part and the handle of the malleus are crossed internally by the chorda tympani — 2. PARS
TENSA: The rest of the membrane, the main part, is the pars tensa.
e. Mastoid antrum & air cells
 The mastoid (tympanic) antrum lies behind the epitympanic recess in the petrous part of the
temporal bone — The mastoid antrum is present at birth and is then almost adult size.
f. Auditory tube
 The auditory tube (pharyngotympanic tube, Eustachian tube) connects the nesopharynx with the
middle ear — It has bony and cartilaginous parts. 1. BONY PART: It tapers down from the
anterior wall of the middle ear & 2. CARTILAGINOUS PART: It joins the bony orifice and is
lodged in the groove between the greater wing of the sphenoid and the apex of the petrous part of
the temporal bone.
g. Blood supply of middle ear cavity & mastoid air cells
 1. CAROTICOTYMPANIC ARTERY: It is branch of the ICA, that enters the tympanic cavity via a
small foramen in the carotid canal — 2. ANTERIOR TYMPANIC ARTERY: It is branch of the
Maxillary artery, that enters the tympanic cavity via petrotympanic fissure — 3. STYLOMASTOID
ARTERY: It is branch of the posterior auricular artery, that enters the tympanic cavity via
stylomastoid foramen.

Head & Neck (4TH semester—Unit-I PRACTICAL MANUAL) Page 105


Fig-40: Parts of right ear in relation to temporal bone viewed from above.
3.Inner ear (Fig—40 & 41)
a. Bony labyrinth
 It consists of: 1. COCHLEA: The cochlea consists of two and three-quarter spiral turns of a
tapering canal — The basal turn projects laterally producing promontory on medial wall of
middle ear — The axial bony stem around which the canal spirals is the modiolus — The
bony cochlea is partitioned by membranous cochlear duct (or scala media containg
endolymph) into scala vestibuli on the apical side of the partition and scala tympani on the
basal side of the partition, they contain perilymph — The both scala vestibule and scala
tympani communicate with each other around the blind apical extremity of the cochlear
duct — The modiolus is perforated by the branches of the cochlear nerve — The spiral
ganglion contains their bipolar cell bodies — 2. VESTIBULE: The vestibule is a
hollow in the petrous bone which contains the membranous saccule and utricle, filled with
endolymph — The scala vestibula of the cochlea opens into the front of the vestibule — 3.
SEMICIRCULAR CANALS: The semicircular canals lie in three lanes at right angles to each

Head & Neck (4TH semester—Unit-I PRACTICAL MANUAL) Page 106


other — Their caliber is 1mm except at one end, where each is dilated as the ampulla to a
caliber of 2 mm.
b. Membranous labyrinth
 It consists of one continuous closed cavity containing endolymph — The membranous covering
consists of three layers: 1. THE OUTER FIBROUS LAYER: is vascular and in places adherent to
the endosteum of the bony labyrinth forming the stria vascularis which produces endolymph —
2. THE INTERMEDIATE LAYER is homogeneous like a basal lamina — 3. THE INNER
EPITHELIAL LAYER is elaborated in three places into receptors of sound, static balance and
kinetic balance.

Fig-41: The middle ear and its relations. Bony (B) and membranous (C) labyrinths

II.CLINICAL CORRELATIONS
Head & Neck (4TH semester—Unit-I PRACTICAL MANUAL) Page 107
1. Tympanic membrane examination
 Otoscopic examination of the tympanic membrane is facilitated by first straightening the external
auditory meatus by gently pulling the auricle upward and backward in the adult, and straight
backward or backward and downward in the infant..
2. Complications of otitis media
 Inadequate treatment of otitis media can result in the spread of the infection into the mastoid
antrum and the mastoid air cells (acute mastoiditis).
 Acute mastoiditis may be followed by the further spread of the organisms beyond the confines of
the middle ear.
 A spread of the infection in this direction could produce a meningitis and a cerebral abscess in
the temporal lobe.
 Beyond the medial wall of the middle ear lie the facial nerve and the internal ear. A spread of
the infection in this direction can cause a facial nerve palsy and labyrinthitis with vertigo.
 The posterior wall of the mastoid antrum is related to the sigmoid venous sinus.
 If the infection spreads in this direction, a thrombosis in the sigmoid sinus may well take place.
These various complications emphasize the importance of knowing the anatomy of this region.
III.REVIEW QUESTIONS
Q.1: A 45-YEAR-OLD WOMAN IS ADMITTED TO THE HOSPITAL FOR SEVERE EAR PAIN.
PHYSICAL EXAMINATION REVEALS CHRONIC INFECTION OF THE MASTOID AIR CELLS
(MASTOIDITIS). THE INFECTION CAN ERODE THE THIN LAYER OF THE BONE BETWEEN THE
MASTOID AIR CELLS AND THE POSTERIOR CRANIAL FOSSA AND SPREAD MOST
COMMONLY INTO WHICH OF THE FOLLOWING VENOUS STRUCTURES?
A. Superior sagittal sinus
B. Inferior sagittal sinus
C. Straight sinus
D. Cavernous sinus
E. Sigmoid sinus
ANSWER: E.
 The sigmoid venous sinus empties into the internal jugular vein and drains the cranial vault. It
runs along the posterior cranial fossa near the suture between the temporal and occipital
bones…….The superior sagittal sinus lies within the superior aspect of the longitudinal fissure,
between the two cerebral hemispheres.
Q.2: A 3-YEAR-OLD GIRL RUPTURED HER EARDRUM WHEN SHE INSERTED A PENCIL INTO
HER EXTERNAL EAR CANAL. SHE WAS URGENTLY ADMITTED TO THE EMERGENCY
DEPARTMENT. PHYSICAL EXAMINATION REVEALED PAIN IN HER EAR AND A FEW DROPS
OF BLOOD IN THE EXTERNAL AUDITORY MEATUS. THERE WAS THE CONCERN THAT
THERE MIGHT POSSIBLY HAVE BEEN AN INJURY TO THE NERVE THAT PRINCIPALLY
INNERVATES THE EXTERNAL SURFACE OF THE TYMPANIC MEMBRANE. WHICH OF THE
FOLLOWING TESTS IS MOST LIKELY TO BE PERFORMED DURING PHYSICAL EXAMINATION
TO CHECK FOR INJURY TO THIS NERVE?
A. Check the taste in the anterior two thirds of the tongue.
B. Check the sensation to the pharynx and palate.
C. Check if there is paraesthesia at the TMJ.
D. Check for sensation in the larynx.
E. Check for sensation in the nasal cavity.
ANSWER: A.
Q.3: A 27-YEAR-OLD WOMAN IS ADMITTED TO THE EMERGENCY DEPARTMENT AFTER SHE
WAS THROWN FROM A MOTOR SCOOTER. RADIOGRAPHIC EVALUATION REVEALS A TYPE
I LEFORT FRACTURE AND COMMINUTED FRACTURE OF THE MANDIBLE AND TMJ. DESPITE
RECONSTRUCTIVE SURGERY, THE PATIENT DEVELOPS HYPERACUSIS (SENSITIVITY TO
LOUD SOUNDS) DUE TO FACIAL NERVE PARALYSIS. WHICH OF THE FOLLOWING MUSCLES
IS MOST LIKELY PARALYZED?

A. Posterior belly of digastric


B. Stapedius

Head & Neck (4TH semester—Unit-I PRACTICAL MANUAL) Page 108


C. Tensor tympani
D. Stylohyoid
E. Cricothyroid
ANSWER: B.
 Both the stapedius and tensor tympani normally function to dampen movements of the middle ear
ossicles, thereby muting sound and preventing hyperacusis. The stapedius would be the source
of hyperacusis in this problem because it receives its innervation from the facial nerve.
Q.4: A 12-YEAR-OLD GIRL IS ADMITTED TO THE EMERGENCY DEPARTMENT WITH A
MIDDLE EAR INFECTION. PHYSICAL EXAMINATION REVEALS A LONG HISTORY OF
CHRONIC MIDDLE EAR INFECTIONS THAT HAVE PRODUCED A LESION IN THE TYMPANIC
PLEXUS IN THE MIDDLE EAR CAVITY. SINCE THE PREGANGLIONIC PARASYMPATHETIC
FIBERS THAT PASS THROUGH THE PLEXUS HAVE BEEN LOST, WHICH OF THE FOLLOWING
CONDITIONS WILL BE DETECTABLE DURING PHYSICAL EXAMINATION?
A. Diminished mucus in the nasal cavity
B. Diminished mucus on the soft palate
C. Diminished saliva production by the parotid gland
D. Diminished saliva production by the submandibular and sublingual glands
E. Diminished tear production by the lacrimal gland
ANSWER: C.
 Parasympathetic innervation of the parotid gland is provided by axons carried by the
glossopharyngeal nerve that emerge from the tympanic plexus of the middle ear as the lesser
petrosal nerve. These preganglionic parasympathetic fibers terminate by synapses in the otic
ganglion, which supplies the secretory parasympathetic innervation to the parotid gland.
Q.5: A 55-YEAR-OLD MAN WITH SEVERE EAR PAIN VISITS THE ENT DOCTOR. DURING
OTOSCOPIC EXAMINATION THE TYMPANIC MEMBRANE IS RUPTURED. WHICH OF THE
FOLLOWING NERVES IS RESPONSIBLE FOR THE SENSORY INNERVATION OF THE INNER
SURFACE OF THE TYMPANIC MEMBRANE?
A. Glossopharyngeal
B. Auricular branch of facial
C. Auricular branch of vagus
D. Great auricular
E. Lingual
ANSWER: A.
 The inner surface of the tympanic membrane is supplied by the glossopharyngeal nerve.
Q.6: A 55-YEAR-OLD WOMAN IS ADMITTED TO THE EMERGENCY DEPARTMENT WITH EAR PAIN,
RINGING IN HER EAR (TINNITUS), DIZZINESS, AND VERTIGO. RADIOGRAPHIC EXAMINATION REVEALS
INDICATIONS OF MÉNIÈRE’S DISEASE. WHICH OF THE FOLLOWING STRUCTURES IS MOST LIKELY
AFFECTED BY THE EDEMA THAT IS ASSOCIATED WITH MÉNIÈRE’S DISEASE?
A. Middle ear
B. Endolymphatic sac
C. Semicircular canals
D. Cochlea
E. Helicotrema
ANSWER: B.
 Hydrops (edema) results from accumulation of excessive fl uid in the endolymphatic sac. Labyrinthine
hydrops or endolymphatic hydrops is known as Ménière disease. This disease can result in hearing loss,
roaring noises in the ear, and episodic dizziness (vertigo) associated with nausea and vomiting..
Q.7: A 40-YEAR OLD LADY HAS HISTORY OF LEFT EAR DISCHARGE FOR PAST 5 YEARS. SHE HAS
RECENTLY DEVELOPED VERTIGO. THE BLOOD SUPPLY OF THE TYMPANIC ANTRUM AND MASTOID
AIR CELL SEEMED TO BE IMPAIRED. WHICH IS THE MOST LIKELY ARTERY TO BE INVOLVED:

A. Ascending pharyngeal artery


B. Carotioco-tympanic artery
C. Middle meningeal artery
D. Posterior auricular artery
E. Tympanic branch of maxillary artery

ANSWER: D.

Head & Neck (4TH semester—Unit-I PRACTICAL MANUAL) Page 109


ARTERIES OF THE HEAD & NECK
I. THE COMMON CAROTID ARTERY (Fig—42 & 43)
1. Beginning
 The right common carotid artery begins at the bifurcation of brachiocephalic Trunk — The left
common carotid artery begins directly from the arch of aorta.
2. Termination
 At the superior border of thyroid cartilage into its two terminal branches— external &
internal carotid arteries.
3. Relations
 Each artery ascends within the carotid sheath — In carotid sheath, IJV lies on its lateral side, the
chain of deep cervical lymph nodes lies lateral to IJV and vagus nerve posterior to them —
Carotid sinus is a slight dilation of the proximal part of internal carotid, which may involve
common carotid artery. It is a baroreceptor that reacts to changes in arterial blood pressure. It is
supplied by Glossopharyngeal Nerve (CN IX) — Carotid body is a small reddish brown ovoid
mass of tissue in life — It is supplied by Glossopharyngeal Nerve (CN IX) — It is a
chemoreceptor that monitors the level of oxygen in blood.
4. Branches
 Gives no branches in neck, except its 2 terminal branches.
II. THE EXTERNAL CAROTID ARTERY (Fig—42 & 43)
1. Beginning
 It begins at the level of superior border of thyroid cartilage, as a result of bifurcation of
common carotid artery.
2. Termination
 Each artery runs superiorly to be embedded in the parotid gland and terminates by dividing
into its 2 branches—maxillary and superficial temporal arteries.
3. Relations
 It starts in the carotid triangle — It runs superiorly to the region between the neck of the
mandible and the lobule of the ear — It is embedded in the parotid gland where it divides
into its two terminal branches.
4. Branches
a. Anterior branches
i. superior thyroid artery ii. lingual artery iii. facial artery
b. Posterior branches
iv. Occipital artery v. posterior auricular artery
c. Medial branches
vi. Ascending pharyngeal artery
d. Terminal branches
vii. Superficial temporal artery viii. Maxillary artery.
III. THE FACIAL ARTERY (Fig—42 & 43)
1. Beginning
 It arises from the anterior aspect of the external carotid artery just above the level of tip of
greater cornu of hyoid bone.
2. Termination
 At the medial angle of the eye, it terminates by anastomosing with the terminal branches of
the opthalmic artery.

Head & Neck (4TH semester—Unit-I PRACTICAL MANUAL) Page 110


3. Relations
 It begins in the carotid triangle — Arches to enter a groove on the posterior surface of the
submandibular gland — Curves under the body of the mandible at the antero-inferior angle
of the masseter — Passes upward to the angle of the mouth — Ascends along the side of the
nose — Ends at the medial commissure of the eye, under the name of the angular artery —
The facial artery is remarkably tortuous — This is to accommodate itself to neck movements
such as those of the pharynx in deglutition; and facial movements such as those of the mandible,
lips, and cheeks — It lies upon the middle pharyngeal constrictor and the superior pharyngeal
constrictor — The superior pharyngeal constrictor separates it from the tonsil.
4. Branches
a. Branches In The Submandibular Region
i. Ascending palatine artery: It supplies the soft palate.
ii. Tonsillar artery: It supplies the tonsils.
iii. Glandular arteries: They supply submandibular gland.
b. Branches On The Face
iv. Submental artery: It supplies the skin of lower lip and chin.
v. Inferior labial artery: It runs medially in the lower lip and anastomosis with its
opposite side.
vi. Superior labial artery: It runs medially in the upper lip and gives branches to the
nasal septum.
vii. Lateral nasal artery: It supplies the skin on the side and dorsum of the nose.
IV. THE MAXILLARY ARTERY (Fig—42 & 43)
1. Beginning
 It starts within the substance of the parotid gland, at the back of the neck of mandible as a
result of bifurcation of the external carotid artery.
2. Relations
 It leaves the parotid gland, and turns around the neck of mandible to enter the infratemporal
fossa — leaves the infratemporal fossa, by passing through the pterygo-maxillary fissure to
enter the pterygopalatine fossa (where its 3rd part lies) — leaves the pterygopalatine fossa, by
passing through the inferior orbital fissure to become continuous with the infra-orbital artery
— In the infratemporal fossa, it is crossed by the lateral pterygoid muscle which divides the
artery (from description point of view) into three parts — 1st part is before, 2nd part is behind
and 3rd part is beyond the crossing of lateral pterygoid muscle.
3. Termination
 It terminates by passing through the inferior orbital fissure to become continuous with the
infraorbital artery.
4. Branches
a. Branches Of 1st Part
i. Inferior alveolar artery: It enters the mandibular foramen in the ramus of
mandible — runs through the mandibular canal and supplies the pulps of
mandibular teeth — before entering the mandibular foramen, it gives off artery to
mylohyoid, that pierces the sphenomandibular ligament.
ii. Middle meningeal artery: It is embraced by the two roots of origin of
auriculotemporal nerve — it leaves the infratemporal fossa by passing through the
foramen spinosum to enter the middle cranial fossa to supply the dura mater.
iii. Accessory meningeal artery: It leaves the infratemporal fossa by passing
through the foramen ovale to enter the middle cranial fossa to supply the
trigeminal ganglion.
iv. Deep auricular artey: It supplies the external acoustic meatus and outer
surface of tympanic membrane.
v. Anterior tympanic artery: It supplies the middle ear cavity.

Head & Neck (4TH semester—Unit-I PRACTICAL MANUAL) Page 111


b. Branches of 2nd part
vi. Artery to lateral pterygoid vii. Artery to medial pterygoid viii. Artery to
masseter
ix. 1ST artery to temporalis x. 2ND artery to temporalis
c. Branches of 3rd part (within pterygopalatine fossa)
xi. Sphenopalatine artery: It passes through the sphenopalatine foramen to
supply the nasal cavity.
xii. Greater palatine artery: It passes through the greater palatine foramen to
supply the hard palate.
xiii. Lesser palatine artery: It passes through the lesser palatine foramen to
supply the soft palate.
xiv. Pharyngeal artery: It passes through the palatovaginal foramen to supply
the nasopharynx.
xv. Posterior superior alveolar artery: It leaves the pterygopalatine fossa
through the pterygo-maxillary fissure, and enters the infratemporal fossa to supply the
pulps of molar teeth.
V. THE INFRAORBITAL ARTERY (Fig—42 & 43)
1. Beginning
 It starts within the inferior orbital fissure, as a result of continuation of maxillary artery.
2. Relations
 It runs in the floor of the orbit— It emerges on the face by passing through the infraorbital
foramen.
3. Termination
 It terminates by passing through the infraorbital foramen to become cutaneous to the face.
4. Branches
i. Middle superior alveolar artery: It supplies the pulps of the premolars and
canines.
ii. Anterior superior alveolar artery: It supplies the pulps of the incisor teeth.
VI. THE INTERNAL CAROTID ARTERY (Fig—42 & 43)
1. Beginning
 It is a direct continuation of common carotid at superior border of thyroid cartilage —
2. Termination
 At inferior surface of brain, it terminates by dividing into anterior and middle cerebral arteries

3. Relations
 It starts in carotid triangle and ascends in carotid sheath — Enters the cranium through
carotid canal (foramen lacerum) — In cranial cavity, it passes through cavernous sinus —
Leaves the cavernous sinus by piercing its roof to reach the inferior surface of brain, where it
terminates into anterior and middle cerebral arteries.
4. Branches
a. Branches In The Neck:
 No named branches in neck.
b. Branches In The Cavernous
i. Superior hypophyseal artery: It supplies pituitary gland.
ii. Branches in the wall of cavernous sinus
c. Branches Outside The Cavernous Sinus
iii. ophthalmic artery: It enters optic canal along with optic nerve.
d. Terminal Branches

Head & Neck (4TH semester—Unit-I PRACTICAL MANUAL) Page 112


iv. Anterior cerebral artery v. Middle cerebral artery.

Fig-42: Branches of carotid arteries.

Head & Neck (4TH semester—Unit-I PRACTICAL MANUAL) Page 113


Fig-43: Main arteries of the head and neck.

VII. THE OPHTHALMIC ARTERY


1. Beginning
 The ophthalmic artery arises from the internal carotid, just as that vessel is emerging from
the cavernous sinus.
2. Termination
 Within the orbital cavity it terminates by dividing into its two terminal branches—
the frontal and dorsal nasal arteries.
3. Relations
 It enters the orbital cavity through the optic canal, below and lateral to the optic nerve —
Passes over the optic nerve to reach the medial wall of the orbit, and divides
into frontal and dorsal nasal branches.
4. Branches
a. Orbital Branches
i. Lacrimal artery
ii. Supraorbital artery: It passes upward to the supraorbital foramen.
iii. Posterior ethmoidal artery
iv. Anterior ethmoidal artery: It supplies lateral wall and septum of the nose, and
appears on the dorsum of the nose between the nasal bone.
v. Medial palpebral arteries: Two in number, leave the orbit to encircle the eyelids.
vi. Frontal artery: It is one of the terminal branches of the ophthalmic, leaves the
orbit at its medial angle ascends on the forehead, supplies the skin, muscles, and
pericranium.
vii. Dorsal nasal artery: It is the other terminal branch of the ophthalmic, runs
along the dorsum of the nose, supplies its outer surface; and anastomoses with the
artery of the opposite side, and with the lateral nasal branch of facial artery.

b. Ocular branches
viii. Central artery of retina: It is the first and one of the smallest branches of the
ophthalmic artery — It runs for a short distance within the dural sheath of the
optic nerve, but about 1.25 cm. behind the eyeball it pierces the nerve obliquely,
and runs forward in the center of its substance to the retina.
ix. Short posterior ciliary arteries: They, six to twelve in number, pass forward
around the optic nerve, pierce the sclera around the entrance of the nerve, and
supply the choroid and ciliary processes.
x. Long posterior ciliary arteries: They, two in number, pierce the sclera and run
forward to the ciliary muscle.
xi. Anterior ciliary arteries: They are derived from the muscular branches of
ophthalmic artery.
xii. Superior muscular branch: It supplies the Levator palpebrae superioris,
Rectus superior, and Obliquus superior.
xiii. Inferior muscular branch: It is distributed to the Recti lateralis, medialis, and
inferior, and the Obliquus inferior. This vessel gives off most of the anterior
ciliary arteries.
VIII. THE SUBCLAVIAN ARTERY (Fig—44)
1. Beginning
 The right subclavian artery arises from the brachiocephalic trunk. — The left subclavian
artery arises directly from the arch of aorta.
2. Termination
 As the artery crosses the outer margin of first rib, its name changes to the axillary artery.

Head & Neck (4TH semester—Unit-I PRACTICAL MANUAL) Page 114


3. Relations
 In the root of the neck, the artery is crossed anteriorly by the scalene anterior muscle —
Three parts of each subclavian artery are described relative to the anterior scalene—the 1ST
part of the artery is medial to anterior scalene muscle, the 2ND part of the artery is posterior to
anterior scalene muscle and the 3RD part of the artery is lateral to anterior scalene muscle —Its
pulsation can be felt by applying deep pressure in the base of the posterior triangle of
neck.
4. Branches
a. Branches of 1st part
i. Vertebral artery
ii. Internal thoracic artery: It arises from the inferior aspect of the 1st part of the
subclavian artery and passes inferiorly into the thorax.
iii. Thyrocervical trunk: It arises from the superior aspect of the 1st part of
subclavian artery — It has 3 branches: 1.INFERIOR THYROID ARTERY: It supplies
the thyroid gland, inferior parathyroid gland, cervical parts of esophagus and
trachea…..2.SUPRASCAPULAR ARTERY: It contributes in anastomoses around
scapula…..3.SUPERFICIAL CERVICAL ARTERY: It supplies muscle in the
cervical region.
b. Branches of the 2nd part
iv. Costocervical trunk: The trunk divides into: 1. SUPERIOR INTERCOSTAL
ARTERY: It supplies the first two posterior intercostal spaces…..2. DEEP
CERVICAL ARTERY: It supplies posterior deep cervical muscles.
c. Branches of the 3rd part:
v. dorsal scapular artery: It participates in the arterial anastomosis around
scapula.

Head & Neck (4TH semester—Unit-I PRACTICAL MANUAL) Page 115


Fig-44: Branches of subclavian artery.

IX. THE VERTEBRAL ARTERY (Fig—44)


1. Beginning
 The vertebral artery arises from the 1ST part the subclavian artery.
2. Termination
 It terminates at the lower border of the pons by uniting with the artery of the opposite side
to form the basilar artery.
3. Relations
 The vertebral artery may be divided into four parts:
a. 1ST part of subclavian artery
 It runs upward in front of transverse process of the C7 vertebra.
b. 2ND part of subclavian artery
 It runs upward through the foramina in the transverse processes of the upper six cervical
vertebrae, it runs an almost vertical course as far as the transverse process of the atlas.
c. 3RD part of subclavian artery
 It issues from the foramen in the transverse process of the atlas, and curves behind in the
groove on the upper surface of the posterior arch of the atlas, and enters the vertebral
canal, this part of the artery is contained in the suboccipital triangle of theRE neck.
d. 4th part of subclavian artery
 It pierces the dura mater and inclines medially to the front of the medulla oblongata.
4. Branches

Head & Neck (4TH semester—Unit-I PRACTICAL MANUAL) Page 116


i. Anterior spinal artery ii. Posterior spinal arteries iii. Medullary arteries iv. Posterior inferior
cerebellar artrey

X.CLINICAL CORRELATIONS
1. Carotid sinus hypersensitivity:
 In cases of carotid sinus hypersitivity, pressure on one or both carotid sinuses can cause
excessive slowing of the heart rate, a fall in blood pressure and cerebral ischemia with
fainting.
2. Taking the carotid pulse:
 The bifurcation of the common carotid artery into the internal and external carotid can be easily
palpated just beneath the anterior border of the sternocleidomastoid muscle at the level of
the superior border of the thyroid cartilage. This is a convenient site to take bccarotid pulse.
3. Arteriosclerosis of the internal carotid artery:
 Extensive arteriosclerosis of the internal carotid artery in the neck can cause visual impairment
or blindness in the eye on the side of the lesion because of insufficient blood flow through the
retinal artery.
 Motor paralysis and sensory loss may also occur on the opposite side of the body because
of insufficient blood flow through the middle cerebral artery.
4. Palpation and compression of the subclavian artery in
patients with upper limb hemorrhage:
 In severe traumatic accidents to the upper limb involving laceration of the brachial or axillary
arteries, it is important to remember that hemorrhage can be stopped by exerting strong
pressure downward and backward in the third part of the subclavian artery.
 The use of a blunt object to exert the pressure is of great help, and the artery is compressed
against the upper surface of the first rib.

XI.REVIEW QUESTIONS
Q.1: THE INTERNAL CAROTID ARTERY:

A. Enters the cranium through the squamous temporal bone


B. Gives a branch to the choroid plexus of the 3rd ventricle
C. Has an ophthalmic branch entering the orbit through the superior orbital fissure
D. Lies with the dural covering of the cavernous sinus
E. Pierces the diaphragm sella medial to the optic nerve

ANSWER: D.

Q.2: COMMON CAROTID ARTERY:

A. Arises from arch of aorta, on right side.


B. Divides into terminal branches at the level of head of mandible.
C. Lies outside the carotid sheath.
D. Lies lateral to internal jugular vein.
E. Has a dilatation at its termination called carotid sinus.
ANSWER: E.

Q.3: THE CAROTID SINUS:

A. Is located at the origin of the external carotid artery


B. Is innervated by the facial nerve

Head & Neck (4TH semester—Unit-I PRACTICAL MANUAL) Page 117


C. Functions as a chemoreceptor
D. Is stimulated by changes in blood pressure
E. Communicates freely with the cavernous sinus
ANSWER: D.

Q.4: A PHYSICIAN WANTS TO PAPLATE AN ARTERY FOR WHICH HE ASKS HIS PATIENT TO
CLENCHED HER TEETH. SO THAT THE MASSETER BECOMES PROMINENT. WHICH ARTERY
HE WANTS TO PALPATE:

A. Facial artery
B. Maxillary artery
C. Superficial temporal artery
D. Transverse facial artery
E. External carotid artery

ANSWER: A.

Q.5: DURING SURGERY A SURGEON NOTICES AN ARTERY CROSSING IN FRONT OF MASSETER


MUSCLE AND GOING UPWARD FROM ANGLE OF MOUTH. WHICH OF THE FOLLOWING IS MOST
LIKELY THE ARTERY:

A. Maxillary artery
B. Fascial artery
C. Mandibular artery
D. Lingual artery
E. External iliac artery

ANSWER: B.

Q.6: PULSATIONS FELT JUST ABOVE THE ZYGOMATIC ARCH AND IN FRONT OF THE EAR ARE FROM
WHICH VESSEL:

A. Facial
B. Internal jugular vein
C. Superficial temporal artery
D. Retromandibular vein
E. Maxillary artery
ANSWER: C.

Q.7: REGARDING MIDDLE MENINGEAL ARTERY FOLLOWING STATEMENT ARE TRUE


EXCEPT:

A. Can be fairly located outside the skull with reference to Pterion


B. Enters the middle cranial fossa through foramen lacerum
C. Is commonly involved in extraduarl haemorrhage
D. Produces grooves on the interior of the cranium
E. Supplies most of supra-tentorial dura mater

ANSWER: B.
Q.8: A 64-YEAR-OLD MAN HAS HAD RECURRING NASAL HEMORRHAGES FOLLOWING
SURGERY TO REMOVE NASAL POLYPS. TO CONTROL THE BLEEDING, THE SURGEON IS
CONSIDERING LIGATING THE PRIMARY ARTERIAL SUPPLY TO THE NASAL MUCOSA. THIS
ARTERY IS A DIRECT BRANCH OF WHICH OF THE FOLLOWING ARTERIES:

A. Facial artery
B. Maxillary artery
C. Superficial temporal artery
D. Superior labial artery
E. Transverse facial artery
ANSWER: B.

Q.9: A PATIENT PRESENTS WITH A CHIEF COMPLAINT OF CHRONIC NOSE BLEEDS. TO


CONTROL THE SEVERITY OF THESE NOSE BLEEDS, HIS PHYSICIAN DECIDES TO LIGATE

Head & Neck (4TH semester—Unit-I PRACTICAL MANUAL) Page 118


THE SPHENOPALATINE ARTERY. FROM WHICH OF THE FOLLOWING ARTERIES DOES THE
SPHENOPALATINE ARTERY ARISE:

A. External carotid
B. Facial
C. Maxillary
D. Ophthalmic
E. Transverse facial
ANSWER: C.
Q.10: DURING SURGERY, A SURGEON NOTICES PROFUSE BLEEDING FROM THE DEEP CERVICAL
ARTERY, WHICH OF THE FOLLOWING ARTERIES MUST BE LIGATED IMMEDIATELY:

A. Inferior thyroid artery.


B. Transverse cervical artery.
C. Thyrocervical trunk.
D. Costocervical trunk.
E. Ascending cervical artery.
ANSWER: C.
Q.11 : AN OCCLUSION OF THE THYROCERVICAL TRUNK COULD PRODUCE A MARKED DECREASE IN
THE BLOOD FOLW IN WHICH OF THE FOLLOWING ARTERIES:B

A. Superior thyroid
B. Inferior thyroid
C. Vertebral artery
D. Maxillary artery
E. Facial artery
ANSWER: B.

Q.12: THE VERTEBRAL ARTERY:C

A. Enters foramen transversarium of the 7th cervical vertebra


B. Enters the cranial cavity through the posterior condylar canal
C. Sends branches to the spinal cord from within the cranial cavity
D. Supplies the cerebellar vermis through its superior cerebellar branch
E. Unites with its fellow of the opposite side at the upper border of the pons to form the basilar artery

ANSWER: C.

VEINS OF THE HEAD & NECK


I. THE ANGULAR VEIN
1. Beginning
 It is formed by the junction of the supratrochlear and supraorbital veins.
2. Termination
 At the level of the lower margin of the orbit, it becomes the anterior facial vein.
3. Relations
 It runs downward, on the side of the root of the nose — It receives the veins of the ala of the
nose — It communicates with the superior ophthalmic vein through the nasofrontal vein,
thus establishing an important anastomosis between the facial vein and the cavernous sinus.
4. Tributaries
i. supratrochlear vein ii. supraorbital vein

II. THE FACIAL VEIN (Fig—45)


1. Beginning

Head & Neck (4TH semester—Unit-I PRACTICAL MANUAL) Page 119


 The facial vein is a direct continuation of the angular vein.

2. Termination
 It terminates by draining into the internal jugular vein.

3. Relations
 It lies behind the facial artery and follows a less tortuous course — it runs obliquely
downward on the superficial surface of the Masseter — It crosses over the body of the
mandible — It passes superficial to the submandibular gland — The facial vein has
no valves.

4. Tributaries
i. Supraorbital Vein ii. Supratrochlear Vein iii. External Nasal Vein
iv. Deep facial vein (from the pterygoid venous plexus)
v. Superior and inferior palpebral veins vi. Superior and inferior labial
veins
vii. The buccinator vein viii. The masseteric vein
ix. Submental, palatine, and submaxillary veins (below the mandible)
x. The vena comitans of the hypoglossal nerve.

III. THE INTERNAL JUGULAR VEIN (Fig—45)


 It is the largest vein in the neck — Most veins in the anterior cervical region are tributaries
of the IJV.
1. Beginning
 It commences at the jugular foramen in the posterior cranial fossa as the direct continuation of
the sigmoid sinus.
2. Termination
 Posterior to the sternal end of the clavicle, the IJV merges with the subclavian vein to form
the brachiocephalic vein.
3. Relations
 From a dilation at its origin, called the superior bulb of the IJV, the vein descends in the carotid
sheath, accompanying the internal carotid artery superior to the carotid bifurcation and the
common carotid artery and inferiorly — The vagus nerve lies posterior to IJV and carotid
arteries through out its course in the carotid sheath — The inferior end of the vein passes
deep to the gap between the sternal and the clavicular heads of this SCM muscle —
Posterior to the sternal end of the clavicle, the IJV merges with the subclavian vein to form
the brachiocephalic vein — The inferior end of the IJV dilates to form the inferior bulb of the
IJV. This bulb has a bicuspid valve that permits blood to flow toward the heart while preventing
backflow into the vein, as might occur if inverted (e.g., standing on one's head or when
intrathoracic pressure is increased).
4. Tributaries
i. inferior petrosal sinus: It leaves the cranium through the jugular foramen and
enters the superior bulb of the IJV.
ii.facial vein iii.lingual veins
iv.pharyngeal veins: They arise from the venous plexus on the pharyngeal wall
and empty into the IJV at the level of the angle of the mandible
v. superior thyroid vein vi. middle thyroid vein
vii. occipital vein: It usually drains into the suboccipital venous plexus, drained by
the deep cervical vein and the vertebral vein, but it may drain into the IJV.

Head & Neck (4TH semester—Unit-I PRACTICAL MANUAL) Page 120


Fig-45: Major veins of the head and neck and their tributaries.

IV. THE SUBCLAVIAN VEIN (Fig—45)


1. Beginning
 The subclavian vein begins at the outer border of the 1st rib, as a result of continuation of the
axillary vein.

2. Termination
 It ends posterior to the sternoclavicular joint by joining the IJV to form the brachiocephalic
vein.

3. Relations
 The subclavian vein is separated from artery by the anterior scalene muscle.

4. Tributaries
 It usually has only one named tributary, the EJV.

V. THE RETROMANDIBULAR VEIN (Fig—45)


1. Beginning

Head & Neck (4TH semester—Unit-I PRACTICAL MANUAL) Page 121


 It is formed by the union of the superficial temporal and maxillary veins.

2. Termination
 It terminates by dividing into two branches:
a. Anterior Division Of Retromandibular Vein
 It unites with the facial vein to form the common facial vein.
b. Posterior Division Of Retromandibular Vein
 It is joined by the posterior auricular vein to form the external jugular vein.

3. Relations
 It descends in the substance of the parotid gland.

4. Tributaries
i. Superficial temporal vein: It receives in its course some parotid veins, articular
veins from the temporomandibular joint, anterior auricular veins from the auricle, and
the transverse facial from the side of the face.
ii. Maxillary vein: It is formed by a confluence of the veins of the pterygoid plexus.

VI. THE PTERYGOID PLEXUS OF VEINS


1. Introduction
 The pterygoid plexus is partly embedded in pterygoid muscle.

2. Beginning
 The plexus is formed by confluence of the veins corresponding with the branches of the
maxillary artery.

3. Termination
 The plexus ends by forming one or two maxillary veins, which join the superficial temporal vein
to form the retromandibular vein, within the substance of the parotid gland.

4. Relations
 This plexus communicates freely with the facial vein through the deep facial vein — It also
communicates with the cavernous sinus by through foramen ovale, and foramen lacerum —
Due to its communication with the cavernous sinus, infection of the superficial face may spread to
the cavernous sinus, causing cavernous sinus syndrome.

5. Tributaries
 It receives tributaries corresponding with the branches of the maxillary artery:
i. Sphenopalatine vein ii. Middle meningeal vein iii. Deep temporal (anterior &
posterior) vein iv. Pterygoid veins v. Masseteric vein vi. Buccal vein
vii. Alveolar veins viii. Some palatine veins ix. Infraorbital vein

VII. THE EXTERNAL JUGULAR VEIN (Fig—45)


1. Introduction

Head & Neck (4TH semester—Unit-I PRACTICAL MANUAL) Page 122


 The external jugular vein receives the greater part of the blood from the exterior of
the cranium and the deep parts of the face.

2. Beginning
 It is formed by the junction of the posterior division of the retromandibular vein with
the posterior auricular vein.

3. Termination
 It terminates by joining the subclavian vein.

4. Relations
 It commences on a level with the angle of the mandible — It runs down the neck & in
the supraclavicular triangle, perforates the investing fascia to end in the subclavian vein —
It is provided with two pairs of valves, which do not prevent the regurgitation of the blood.

5. Tributaries
i. Posterior division of retromandibular vein ii. Posterior auricular vein iv. Posterior external
jugular v. Transverse cervical vi. Transverse scapular vein vii. Anterior jugular vein

VIII. THE ANTERIOR JUGULAR VEIN (Fig—45)


1. Introduction:
 Most frequently there are two anterior jugulars, a right and left; but sometimes only one — Just
above the sternum the two anterior jugular veins communicate by the venous jugular
arch, which receive tributaries from the inferior thyroid veins — There are no valves in this
vein.

2. Beginning
 It begins near hyoid bone by confluence of several superficial veins from submandibular
region.

3. Termination
 At the lower part of the neck, it passes beneath the sternocleidomastoid to open into the
termination of the external jugular.

4. Relations
 It descends between the median line and the anterior border of the Sternocleidomastoideus
— At the lower part of the neck, it passes beneath that muscle to open into the termination of
the external jugular.

5. Tributaries
 Some laryngeal veins, and occasionally a small thyroid vein.

IX. THE VERTEBRAL VEIN (Fig—45)


1. Beginning
 The vertebral vein is formed in the suboccipital triangle, from numerous small tributaries which
spring from the internal vertebral venous plexuses —

2. Termination

Head & Neck (4TH semester—Unit-I PRACTICAL MANUAL) Page 123


 It terminates by draining into the subclavian vein —
3. Relations
 plexus emerges from the vertebral canal above the posterior arch of the atlas — enters the
foramen in the transverse process of the atlas, and descends around the vertebral artery —
plexus ends in a single trunk, which emerges from the foramen transversarium of the sixth
cervical vertebra, and opens at the root of the neck into the brachiocephalic vein.

4. Tributaries
i. Occipital vein ii. Emissary veins that communicates with the transverse
sinus
iii. Veins from prevertebral muscles iv. External vertebral venous plexus
v. Internal vertebral venous plexus vi. Deep cervical vein
vii. First posterior intercostal vein

X.DURAL VENOUS SINUSES (Fig—46 & 47)


 The dural venous sinuses are endothelium-lined spaces between the endosteal and the
meningeal layers of the dura —Large veins from the surface of the brain empty into these
sinuses and most of the blood from the brain ultimately drains through them into IJVs —
Emissary veins connect the dural venous sinuses with the veins outside the cranium —
Although emissary veins are valveless and blood may flow in both directions, flow of emissary
veins is usually away from brain.
1. Superior sagittal sinus
a. Location: It lies in the attached border of the falx cerebri.
b. Drainage: It drains into the right transverse sinus.

2. Inferior sagittal sinus


a. Location: It lies in the inferior concave free border of the falx cerebri.
b. Drainage: It joins the Great Cerebral Vein to form the straight sinus.

3. Straight sinus
a. Location: It is formed by the union of the inferior sagittal sinus with the great
cerebral vein — It runs along the line of attachment of the falx cerebri to the tentorium
cerebelli where it joins the confluence of sinuses.

b. Drainage: It drains into the left transverse sinus.

4. Transverse sinus
a. Location: It courses along the attached margin of the tentorium cerebelli.
b. Drainage: It drains into the sigmoid sinus.

5. Sigmoid sinus
a. Location: The sigmoid sinuses follow S-shaped courses in the posterior cranial fossa.
b. Drainage: Each continues inferiorly as the Internal Jugular Vein after traversing the
jugular foramen.

Head & Neck (4TH semester—Unit-I PRACTICAL MANUAL) Page 124


Occipital sinus
a. Location: It lies in the attached border of the falx cerebelli.
b. Drainage: It communicates inferiorly with the internal vertebral venous plexus.

6. Cavernous sinus
a. Location: It is located on each side of the sella turcica —It consists of a venous
plexus that extends from the superior orbital fissure anteriorly to the apex of the petrous part
of the temporal bone posteriorly.

b. Connections: It receives blood from the superior and inferior ophthalmic veins,
superficial middle cerebral vein, and sphenopareital sinus — The venous channels
communicate with each other through by the anterior and posterior intercavernous
sinuses.

c. Drainage: The cavernous sinuses drain posteroinferiorly: 1.through the superior


petrosal sinus into the meeting point of transverse and sigmoid sinus & — 2. through
the inferior petrosal sinus directly into IJV.

d. Contents: 1. internal carotid artery & 2. abducent nerve (CN VI) lie within the
cavernous sinus 3. oculmotor nerve (CN III) 4. trochlear nerve (CN IV) 5. ophthalmic
division of Trigeminal nerve (V1) 6. maxillary division of Trigeminal nerve (V2)—last four
nerves are embedded in the lateral wall of the sinus.

7. Superior petrosal sinus


a. Location: It lies in the attached margin of the tentorium cerebelli.
b. Drainage: It drains in transverse sinuses.

8. Inferior petrosal sinus


a. Location: It runs in a groove between the petrous part of temporal bone and basilar
part of the occipital bone.

b. Drainage: It drains into the origin of the IJVs.

Head & Neck (4TH semester—Unit-I PRACTICAL MANUAL) Page 125


Fig-46: Dural venous sinuses.

XI. CLINICAL CORRELATIONS


1. Penetrating wounds of the internal jugular vein
 The hemorrhage of low-pressure venous blood into the loose connective tissue beneath the
investing layer of deep cervical fascia may present as a large, slowly expanding hematoma.
 Air embolism is a serious complication of a lacerated wall of the internal jugular vein.
 Because the wall of this large vein contains little smooth muscle, its injury is not followed by
contraction and retraction (as occurs with arterial injuries).
 Moreover, the adventitia of the vein wall is attached to the deep fascia of the carotid sheath,
which hinders the collapse of the vein. Blind clamping of the vein is prohibited because the
vagus and hypoglossal nerves are in the vicinity.
2. Subclavian vein thrombosis
 Spontaneous thrombosis of the subclavian and/or axillary veins occasionally occurs after
excessive and unaccustomed use of the arm at the shoulder joint. The close relationship of
these veins to the first rib and the clavicle and the possibility of repeated minor trauma from these
structures is probably a factor in its development.
 Secondary thrombosis of subclavian and/or axillary veins is a common complication of an
indwelling venous catheter. Rarely, the condition may follow a radical mastectomy with a
block dissection of the lymph nodes of the axilla. Persistent pain, heaviness, or edema of the
upper limb, especially after exercise, is a complication of this condition.
3. Visibility of the external jugular vein
 The external jugular vein is less obvious in children and women because their subcutaneous
tissue tends to be thicker than the tissue of men. In obese individuals the vein may be difficult to
identify even when they are asked to hold their breath, which impedes the venous rectum to the
right side of the heart and distends the vein.

Head & Neck (4TH semester—Unit-I PRACTICAL MANUAL) Page 126


 The superficial vein of the neck tends to be enlarged and often tortuous in professional
singers because of prolonged periods of raised intrathoracic pressure.
4. The external jugular vein as a venous manometer
 The external jugular vein serves as a useful venous manometer normally, when the patient is
laying at a horizontal angle of 30o the level of the blood in the external jugular veins reaches about
one-third of the way up the neck. As the patient sits up, the blood level falls until it is no longer
visible behind the clavicle.
5. External jugular vein catheterization
 The external jugular vein can be sued for catheterization, but the presences of valves or
tortuosity may make the passage of the catheter difficult. Because the right external jugular
vein is in the most direct line with the superior vena cava, it is the one most commonly used.
 The vein is catheterized about halfway between the level of the cricoids cartilage and the
clavicle.
 The passage of the catheter should be performed during inspiration when the valves are
open.
6. Scalp Infections and cavernous sinus thrombosis
 Occasionally, an infection of the scalp spreads by the emissary veins, which are valveless, to
the skull bones, causing osteomyelitis.
 Infected blood in the diploic veins may travel by the emissary veins farther into the venous
sinuses and produce venous sinus thrombosis.
7. Facial Infections and Cavernous Sinus Thrombosis
 The area of facial skin bounded by the nose, the eye, and the upper lip is a potentially dangerous
zone to have an infection. For example, a boil in this region can cause thrombosis of the
facial vein, with spread of organisms through the inferior ophthalmic veins to the
cavernous sinus. The resulting cavernous sinus thrombosis may be fatal unless adequately
treated with antibiotics.
XII.REVIEW QUESTIONS
Q.1: A 72-YEARS-OLD MAN HAS UNDERGONE AN ESOPHAGECTOMY DUE TO CARCINOMA
OF ESOPHAGUS . A CENTRAL VENOUS LINE PASSED THROUGH INTERNAL JUGULAR VEIN
POSTOPERATIVELY FOR PARENTERAL NUTRITION. REGARDING THIS VEIN ALL OF THE
FOLLOWING IS TRUE EXCEPT:

A. Is a continuation of sigmoid sinus


B. Lies in carotid sheath
C. Is crossed by hyogessal nerve
D. Receives facial vein
E. Receives lingual vein
ANSWER: C.
Q.3: A 32-YEAR-OLD WOMAN PRESENTS TO HER PHYSICIAN WITH COMPLAINTS OF
HEADACHE AND SEIZURES. HER PAST MEDICAL HISTORY IS NOTABLE FOR A NORMAL
DELIVERY OF A FULL-TERM GIRL TWO WEEKS PRIOR. EXAMINATION REVEALS
PAPILLOEDEMA. THROMBOSIS OF WHICH OF THE FOLLOWING MOST LIKELY OCCURRED
IN THIS PATIENT:

A. Cavernous sinus
B. Great vein of Galen
C. Middle cerebral artery
D. Middle meningeal artery
E. Posterior cerebral artery
ANSWER: A.
Q.4: A 46-YEAR-OLD MAN SUSTAINS A SPIDER BITE ON HIS UPPER EYELID, AND AN
INFECTION DEVELOPS. THE PHYSICIAN IS VERY CONCERNED ABOUT SPREAD OF THE
INFECTION TO THE DURAL VENOUS SINUSES OF THE BRAIN VIA EMISSARY VEINS. WITH
WHICH OF THE FOLLOWING DURAL VENOUS SINUSES DOES THE SUPERIOR OPHTHALMIC
VEIN DIRECTLY COMMUNICATE:

Head & Neck (4TH semester—Unit-I PRACTICAL MANUAL) Page 127


A. Cavernous sinus
B. Occipital sinus
C. Sigmoid sinus
D. Superior petrosal sinus
E. Straight sinus
ANSWER: A.

Fig-47: Cavernous sinus.

Q.5: INTERNAL JUGULAR VEIN TRIBUTARIES ARE ALL OF THE FOLLOWING EXCEPT:

A. Superior thyroid vein


B. Inferior thyroid vein
C. Facial vein
D. Pharyngeal plexus
E. Inferior petrosal sinus

ANSWER: B.

Head & Neck (4TH semester—Unit-I PRACTICAL MANUAL) Page 128


Q.6: THE TRUE STATEMENT REGARDING THE INTERNAL JUGULAR VEIN IS THAT IT:

A. Has deep cervical lymph nodes along its course


B. Is the direct continuation of transverse sinus
C. Joins the external jugular vein to form the brachiocephalic vein
D. Receives inferior petrosal sinus near its termination
E. Runs downward through the neck in the carotid sheath deep to sternoceidomastiod muscle.

ANSWER: E.
Q.7: EXTERNAL JUGULAR VEIN MOST LIKELY:

A. Begins on surface of sternocleidomastoid muscle behind angle of mandible


B. Terminates by joining axillary vein
C. Formed by union of posterior branch of retromandibular vein and anterior auricular vein
D. Pierces the investing fascia near the anterior border of sternocleidomastoid muscte
E. Receives no tributaries

ANSWER: A.
Q.8: THE SUBCLAVIAN VEIN:

A. Forms the brachiocephalic vein at the level of the First costal cartilage
B. Receives the anterior jugular vein
C. Is crossed anteriorly by the phrenic nerve
D. Receives the cephalic vein
E. Is anterior to the scalenus muscle

ANSWER: E.

Head & Neck (4TH semester—Unit-I PRACTICAL MANUAL) Page 129


LYMPH NODES OF HEAD & NECK
 The lymph nodes of the head and neck are arranged as a regional collar that extends from
below the chin to the back of the head and as a deep vertical terminal group that is
embedded in the carotid sheath in the neck.

Fig-48: Lymph nodes of head and neck.

I.REGIONAL LYMPH NODES (Fig—48)


 The regional nodes are arranged as follows:
1. Occipital nodes
a. Location: These are situated over the occipital bone on the back of the skull.
b. Area of drainage: They receive lymph from the back of the scalp.
2. Retroauricular (mastoid) nodes
a. Location: These lie behind the ear over the mastoid process.
b. Area of drainage: They receive lymph from the scalp above the ear, the auricle,
and the external auditory meatus.

Head & Neck (4TH semester—Unit-I PRACTICAL MANUAL) Page 130


3. Parotid nodes
a. Location: These are situated on or within the parotid salivary gland.
b. Area of drainage: They receive lymph from the scalp above the parotid gland, the
eyelids, the parotid gland, the auricle, and the external auditory meatus.
4. Buccal (facial) nodes
a. Location: One or two nodes lie in the cheek over the buccinator muscle.
b. Area of drainage: They drain lymph that ultimately passes into submandibular
nodes.
5. Submandibular nodes
a. Location: These lie superficial to submandibular salivary gland just below the lower
margin of the jaw.
b. Area of drainage: They receive lymph from the front of the scalp; the nose; the
cheek; the upper lip and the lower lip (except the central part); the frontal, maxillary,
and ethmoid sinuses; the upper and lower teeth (except the lower incisors); the
anterior two thirds of the tongue (except the tip); the floor of the mouth and vestibule;
and the gums.
6. Submental nodes
a. Location: These lie in the submental triangle just below the chin.
b. Area of drainage: They drain lymph from the tip of the tongue, the floor of the
anterior part of the mouth, the incisor teeth, the center part of the lower lip, and the
skin over the chin.
7. Anterior cervical nodes
a. Location: These lie along the course of the anterior jugular veins in the front of the
neck.
b. Area of drainage: They receive lymph from the skin and superficial tissues of the
front of the neck.
8. Superficial cervical nodes
a. Location: These lie along the course of the external jugular vein on the side of the
neck.
b. Area of drainage: They drain lymph from the skin over the angle of the jaw, the
skin over the lower part of the parotid gland, and the lobe of the ear.
9. Retropharyngeal nodes
a. Location: These lie behind the pharynx and in front of the vertebral column.
b. Area of drainage: They receive lymph from the nasal pharynx, the auditory tube,
and the vertebral column.
10. Laryngeal nodes
a. Location: These lie in front of the larynx.
b. Area of drainage: They receive lymph from the larynx.
11. Tracheal (paratracheal) nodes
a. Location: These lie alongside the trachea.
b. Area of drainage: They receive lymph from neighboring structures, including the
thyroid gland.
II. DEEP CERVICAL LYMPH NODES (Fig—48)
 The deep cervical nodes form a vertical chain along the course of the internal jugular vein
within the carotid sheath — They receive lymph from all the groups of regional nodes.

Head & Neck (4TH semester—Unit-I PRACTICAL MANUAL) Page 131


1. Jugulo-digastric lymph node
a. Location: The jugulo-digastric node (a deep cervical node), is located below and
behind the angle of the jaw.
b. Area of drainage: It is mainly concerned with drainage of the tonsil and the
tongue.

2. Jugulo-omohyoid lymph node


a. Location: The jugulo-omohyoid node(a deep cervical node), is situated close to the
omohyoid muscle.
b. Area of drainage: It is mainly associated with drainage of the tongue.
 THE EFFERENT LYMPH VESSELS FROM THE DEEP CERVICAL LYMPH NODES JOIN TO
FORM THE JUGULAR TRUNK, WHICH DRAINS INTO THE THORACIC DUCT OR THE RIGHT
LYMPHATIC DUCT.
III. CLINICAL CORRELATIONS
1. Clinical significance of the cervical lymph nodes
 Knowledge of the lymph drainage of an organ or region is of great clinical importance.
 Examination of a patient may reveal an enlarged lymph node. It is the physician's responsibility to
determine the cause and be knowledgeable about the area of the body that drains its lymph into a
particular node.
 For example, an enlarged submandibular node can be caused by a pathologic condition in
the scalp, the face, the maxillary sinus, or the tongue.
 An infected tooth of the upper or lower jaw may be responsible. Often a physician has to search
systematically the various areas known to drain into a node to discover the cause.
2. Examination of the deep cervical lymph nodes
 Lymph nodes in the neck should be examined from behind the patient. The examination is made
easier by asking the patient to flex the neck slightly to reduce the tension of the muscles.
 The groups of nodes should be examined in a definite order to avoid omitting any.
 After the identification of enlarged lymph nodes, possible sites of infection or neoplastic growth
should be examined, including the face, scalp, tongue, mouth, tonsil, and pharynx.
3. Carcinoma metastases in the deep cervical lymph
node
 When cervical metastases occur, the surgeon usually decides to perform a block dissection of the
cervical nodes. This procedure involves the removal en bloc of the internal jugular vein, the
fascia, the lymph nodes, and the submandibular salivary gland. The aim of the operation is
removal of all the lymph tissues on the affected side of the neck.
 The carotid arteries and the vagus nerve are carefully preserved. It is often necessary to sacrifice
the hypoglossal and vagus nerves, which may be involved in the cancerous deposits.
IV.REVIEW QUESTIONS
Q.1: A 12 YEAR BOY PRESENTED IN OPD, WITH POSTERIOR CERVICAL LYMPH
ADENOPATHY. DURING BIOPSY WHICH NERVE IS MOST LIKELY TO BE INJURED:

A. Cranial part of accessory


B. Glossopharyngeal
C. Hypoglossal
D. Spinal part of accessory
E. Vagus

ANSWER: D.

Head & Neck (4TH semester—Unit-I PRACTICAL MANUAL) Page 132


Q.2: LOWER LIP CARCINOMA IS LIKELY TO METASTASIZED INTO THE FOLLOWING LYMPH
NOSES:

A. Deep cervical lymph nodes


B. Submandibular lymph nodes
C. Submental lymph nodes
D. Jugulodigastric lymph nodes
E. Submental & submandibular lymph node

ANSWER: E.
Q.3: SINGLE LYMPHOID TISSUE IN THE HEAD & NECK AREA IS:

A. Lingual tonsil
B. Phyrangeal tonsil
C. Palatine tonsil
D. Tubal tonsil
E. None of the above

ANSWER: B.
Q .4: A 65 YEAR OLD WOMAN PRESENTED WITH A NODULE IN THE TONGUE FOR THE LAST 6
MONTHS. ON EXAMINATION, IT WAS FOUND TO BE 1CM IN SIZE, SITUATED IN THE ANTERIOR 2/3 OF
THE LEFT SIDE OF THE TONGUE. THE GROUP OF LYMPH NODES MOST LIKELY TO BE INVOLVED
FIRST IS:

A. Left inferior deep cervical


B. Left superior deep cervical
C. Right inferior deep cervical
D. Right superior deep cervical
E. Submental

ANSWER: A.

CASE STUDY
Case—1
AN 8-YEAR-OLD GIRL WAS TAKEN TO A PEDIATRICIAN BECAUSE HER MOTHER HAD
NOTICED A SMALL PAINLESS SWELLING BELOW AND BEHIND THE ANGLE OF THE JAW ON
THE RIGHT SIDE. ON EXAMINATION, THE SWELLING WAS SUPERFICIAL, COOL TO TOUCH,
AND SHOWED NO REDNESS. CAREFUL PALPATION OF THE NECK REVEALED TWO FIRM
LUMPS MATTED TOGETHER BENEATH THE ANTERIOR BORDER OF THE RIGHT
STERNOCLEIDOMASTOID MUSCLE. EXAMINATION OF THE PALATINE TONSILS SHOWED
MODERATE HYPERTROPHY ON BOTH SIDES WITH A FEW PUSTULES EXUDING FROM THE
TONSILLAR CRYPTS ON THE RIGHT SIDE. THE PATIENT DID NOT HAVE A PYREXIA. SHE
WAS HAVING TUBERCULOUS CERVICAL LYMPHADENOPATHY.

1. Which lymph node is located behind the angle of mandible?


ANS: The lymph drains from the tonsil into the jugulodigastric member of the deep cervical lymph
nodes, which when enlarged produce a swelling below and behind the angle of the jaw.

2. Why chronic tonsillitis results in cervical lymphadenitis?


ANS: Tuberculous cervical lymphadenitis is a chronic infection that can enter the tonsil and spread to
the lymph nodes.

3. Which segment of deep cervical fascia limits the spread of such infection?
ANS: The investing layer of deep cervical fascia can limit the spread of infection in the neck.

4. Why the lymph nodes become matted together?


ANS: Tuberculous infection of a lymph node commonly spreads to other nodes in the group and they
become matted together.

5. Why a large cold abcess appears beneath the skin?

Head & Neck (4TH semester—Unit-I PRACTICAL MANUAL) Page 133


ANS: Tuberculous infection results in the destruction of the node with the formation of pus that later
erodes through the deep fascia, producing a large cold abscess beneath the skin.

6. Why a discharging sinus is formed in the cold abscess?


ANS: Secondary infection of a cold abscess causes the abscess to break through the skin to form a
discharging sinus.

Case—2
A 70-YEAR-OLD MAN HAVING CARCINOMATOUS ULCER ON THE RIGHT SIDE OF THE
TONGUE NEAR THE TIP COMPLAINING OF A SMALL PAINLESS SWELLING BELOW HIS CHIN
VISITED HIS PHYSICIAN. ON QUESTIONING, HE SAID THAT HE HAD FIRST NOTICED THE
SWELLING 4 MONTHS EARLIER AND THAT IT WAS GRADUALLY INCREASING IN SIZE.
BECAUSE IT HAD NOT CAUSED ANY DISCOMFORT, HE HAD CHOSEN TO IGNORE IT. ON
EXAMINATION, A SINGLE, SMALL, HARD SWELLING COULD BE PALPATED IN THE
SUBMENTAL TRIANGLE. IT WAS MOBILE ON THE DEEP TISSUES AND NOT ATTACHED TO
THE SKIN.

1. Which lymph node group is involved in this case?


ANS: Submental lymph nodes.

2. Where are the submental lymph node located?


ANS: The submental lymph nodes are located in the submental triangle just below the chin.

3. What is the area of drainage for the submental lymph nodes?


ANS: The submental lymph nodes drain the tip of the tongue, the floor of the mouth in the region of
the frenulum of the tongue, the gums and incisor teeth, the middle third of the lower lip, and the skin
over the chin.

4. Which lymph nodes receive lymph from the submental lymph nodes?
ANS: The deep cervical group of lymph nodes beneath the sternocleidomastoid muscle receive lymph
from the submental lymph nodes.

5. Why carcinomatous submental lymph nodes are mobile and not attached to the deep
tissue?
ANS: The submental lymph nodes are not covered by the superficial parts of the submandibular
salivary glands.

Case—3
A 17-YEAR-OLD GIRL VISITED HER DERMATOLOGIST BECAUSE OF SEVERE ACNE OF THE
FACE. ON EXAMINATION, IT WAS FOUND THAT A SMALL ABSCESS WAS PRESENT ON THE
SIDE OF THE NOSE. THE PATIENT WAS GIVEN ANTIBIOTICS AND WAS WARNED NOT TO
PRESS THE ABSCESS.

1. What is danger area of the face?


ANS: The skin area between the eye, the upper lip, and the side of the nose is a danger area to have
an infection of the skin.

2. Which vein drains the danger area?


ANS: The danger area is drained by the facial vein.

3. Why this patient was warned not to press the abscess?


ANS: Interference with a abscess by squeezing or pricking it can lead to spread of the infection and
thrombosis of the facial vein.

4. How facial vein communicates with the cavernous sinus?


ANS: The facial vein communicates with the cavernous sinus via the superior and inferior ophthalmic
veins. The facial and ophthalmic veins do not possess valves so that infected blood from the face can
spread to the cavernous sinus.

Head & Neck (4TH semester—Unit-I PRACTICAL MANUAL) Page 134


CRANIAL NERVES
I.OLFACTORY NERVE—CN I (Fig—49)
1. Functional components
 Special sensory (SVA).
2. Main actions
 Smell from nasal mucosa (olfactory mucosa) of roof of each nasal cavity and superior sides of
nasal septum and superior concha.
3. Site of attachment to brain
 Cerebrum.
4. Location of the cell bodies
 Olfactory epithelium (olfactory cells) — The cell bodies of olfactory receptor neurons are
located in the olfactory organ (the olfactory part of the nasal mucosa) — The apical surfaces of
the neurons possess fine olfactory cilia, bathed by a film of watery mucus secreted by the
olfactory glands of the epithelium — The cilia are stimulated by molecules of an odiferous
gas dissolved in the fluid.
5. Distribution
 The basal surfaces of the bipolar olfactory receptor neurons of the nasal cavity of one side
give rise to central processes that are collected into approximately 20 olfactory nerves,
constituting the right or left olfactory nerve (CN I) — They pass through foramina in the
cribriform plate of the ethmoid bone and enter the olfactory bulb in the anterior cranial fossa
— The olfactory bulb lies in contact with the orbital surface of the frontal lobe of the cerebral
hemisphere — The olfactory nerve fibers synapse with mitral cells in the olfactory bulb — The
axons of the mitral cells (secondary neurons) form the olfactory tract — The olfactory bulbs
and tracts are technically anterior extensions of the forebrain — Each olfactory tract divides
into lateral and medial olfactory striae (distinct fiber bands) — The lateral olfactory stria
terminates in the piriform cortex of the anterior part of the temporal lobe, and the medial
olfactory stria projects through the anterior commissure to contralateral olfactory
structures..
6. Cranial Exit
 Foramina in cribriform plate of ethmoid bone.
7. Clinical correlates
Anosmia (Loss of Smell)
 Loss of olfactory fibers usually occurs with aging — Consequently, elderly people often have
reduced acuity of the sensation of smell, resulting from progressive reduction in the number
of olfactory receptor neurons in the olfactory epithelium — The chief complaint of most people
with anosmia is the loss or alteration of taste; however, clinical studies reveal that in all but a
few people, the dysfunction is in the olfactory system — The reason is that most people
confuse taste with flavor.

Head & Neck (4TH semester—Unit-I PRACTICAL MANUAL) Page 135


Fig-49: Distribution of the olfactory nerves on the nasal septum and the lateral wall of the
nose.

Fig-50: The optic nerve and its connections.

Head & Neck (4TH semester—Unit-I PRACTICAL MANUAL) Page 136


II.OPTIC NERVE—CN II (Fig—50)
1. Functional components: Special sensory (SSA).
2. Main actions: Vision from retina.
3. Site of attachment to brain: Cerebrum.
4. Location of the cell bodies: Ganglionic cells in retina.
5. Distribution: Although they are officially nerves by convention, the optic nerves
(CN II) develop in a completely different manner i.e evaginations of the diencephalon — The
optic nerves are surrounded by extensions of the cranial meninges and subarachnoid space,
which is filled with cerebrospinal fluid (CSF) — The central artery and vein of the retina
traverse the meningeal layers and course in the anterior part of the optic nerve — CN II begins
where the unmyelinated axons of retinal ganglion cells pierce the sclera and become
myelinated deep to the optic disc — The nerve exits through the optic canal to enter the
middle cranial fossa, where it forms the optic chiasm — Here, fibers from the nasal (medial) half
of each retina decussate in the chiasm and join uncrossed fibers from the temporal (lateral) half of
the retina to form the optic tract — Most fibers in the optic tracts terminate in the lateral
geniculate bodies of the thalamus — From thalamic nuclei, axons are relayed to the visual
cortices of the occipital lobes of the brain.
6. Cranial Exit: Optic canal.
7. Clinical correlates
a. Demyelinating diseases and the optic nerve
 Because the optic nerves are actually CNS tracts, the myelin sheath that surrounds the sensory
fibers from the point at which the fibers penetrate the sclera is formed by oligodendrocytes (glial
cells) rather than by Schwann cells, as in other cranial or spinal nerves — Consequently, the
optic nerves are susceptible to the effects of demyelinating diseases of the CNS, such as
multiple sclerosis (MS), which usually do not affect other nerves of the PNS.
b. Visual field defects
 Complete section of an optic nerve results in blindness in the temporal (T) and nasal (N) visual
fields of the ipsilateral eye — Complete section of the optic chiasm reduces peripheral vision
and results in bitemporal hemianopsia, the loss of vision of one half of the visual field of both
eyes — Complete section of the right optic tract eliminates vision from the left temporal and
right nasal visual fields — A lesion of the right or left optic tract causes a contralateral
homonymous hemianopsia, indicating that visual loss is in similar fields. This defect is most
common form of visual field loss and is often observed in patients with strokes.
III.OCULOMOTOR NERVE—CN III (Fig—51)
1. Functional components: 1. Somatic motor (GSE) — 2. Parasympathetic
(GVE).
2. Main actions: 1. Somatic motor (GSE): to superior rectus, inferior rectus,
medial rectus, inferior oblique, and levator palpebrae superioris muscles — 2.
Parasympathetic (GVE): to sphincter pupillae (constricts pupil) and ciliary muscle
(accommodates lens of eye).
3. Site of attachment to brain: Midbrain.
4. Location of the cell bodies: Oculomotor nucleus (Somatic motor) —
Edinger-Westphal nucleus (Preganglionic parasympathetic fibres).
5. Distribution: It emerges from the midbrain, pierces the dura lateral to the sellar
diaphragm, and then runs through the lateral wall of the cavernous sinus — CN III leaves the
cranial cavity and enters the orbit through the superior orbital fissure — Within the superior
orbital fissure, it divides into: SUPERIOR DIVISION (which supplies the superior rectus and
levator palpebrae superioris), INFERIOR DIVISION (which supplies the inferior and medial rectus

Head & Neck (4TH semester—Unit-I PRACTICAL MANUAL) Page 137


and inferior oblique) — The inferior division also carries preganglionic parasympathetic
fibers to the ciliary ganglion, where they synapse — Postganglionic fibers from ciliary
ganglion pass to the eyeball in the short ciliary nerves to innervate the ciliary muscle and the
sphincter papillae muscle.
6. Cranial Exit:Superior orbital fissure.
7. Clinical correlates
Compression of the Oculomotor Nerve
 Rapidly increasing intracranial pressure (e.g., resulting from an extradural hematoma) often
compresses CN III against the crest of the petrous part of the temporal bone — Because
autonomic fibers in CN III are superficial, they are affected first — As a result, the pupil
dilates progressively on the injured side — Consequently, the first sign of CN III compression
is ipsilateral slowness of the pupillary response to light.

Fig-51: Origin and distribution of the oculomotor nerve

IV.TROCHLEAR NERVE—CN IV (Fig—52)


1. Functional components: Somatic motor (GSE).
2. Main actions: Somatic Motor (GSE) to superior oblique that assists in turning eye
infero-laterally (or inferiorly when adducted).

3. Site of attachment to brain: Posterior aspect of the midbrain.

4. Location of the cell bodies: The nucleus of the trochlear nerve is located in
the midbrain, immediately caudal to the oculomotor nucleus.

5. Distribution: The trochlear nerve (CN IV) is the smallest cranial nerve — It emerges
from the posterior surface of the midbrain (the only cranial nerve to do so), passing anteriorly
around the brainstem, running the longest intracranial course of the cranial nerves — It passes
anteriorly in the lateral wall of the cavernous sinus and passes through the superior orbital
fissure into the orbit, where it supplies the superior oblique, the only extraocular muscle that uses a
pulley, or trochlea, to redirect its line of action (hence the nerve's name).

6. Cranial Exit: Superior orbital fissure.

Head & Neck (4TH semester—Unit-I PRACTICAL MANUAL) Page 138


7. Clinical correlates
Injury to the Trochlear Nerve
 CN IV is rarely paralyzed alone — Lesions of this nerve or its nucleus cause paralysis of the
superior oblique and impair the ability to turn the affected eyeball inferomedially — CN IV may
be torn when there are severe head injuries because of its long intracranial course — The
characteristic sign of trochlear nerve injury is diplopia (double vision) when looking down
(e.g., when going down stairs).

Fig-52: Origin and distribution of the trochlear nerve.

V.TRIGEMINAL NERVE—CN V (Fig—53)


1. Functional components: 1. General sensory (GSA) — 2. Branchial motor
(SVE).
2. Main actions: 1. General sensory (GSA): OPHTHALMIC NERVE (V1) from
cornea, skin of forehead, scalp, eyelids, and mucosa of nasal cavity and paranasal sinuses
—MAXILLARY NERVE (V2) from skin of face over maxilla, including upper lip, maxillary
teeth, mucosa of nose, maxillary sinuses, and palate—MANDIBULAR NERVE (V3) from skin
and over side of head mandible including lower lip, mandibular teeth, TMJ, mucosa of
mouth and anterior two thirds of tongue —2. Branchial motor (SVE): to muscles of
mastication, mylohyoid, anterior belly of digastric, tensor veli palatini, and tensor tympani
(all derived from 1ST pharyngeal arch).
3. Site of attachment to brain: Pons.
4. Location of the cell bodies: 1. General sensory (GSA): Trigeminal
ganglion — 2. Branchial motor (SVE): Motor nucleus of trigeminal nerve.
5. Distribution: It emerges from the pons by a large sensory root and a small motor
root — The large sensory root of CN V is composed mainly of the central processes of the
pseudounipolar neurons located in the trigeminal ganglion — The large sensory root of CN V
is divided into THREE divisions at the anterior end of the trigeminal ganglion: 1. Ophthalmic
nerve, 2. Maxillary nerve & 3. Sensory component of the mandibular nerve.
 The fibers of the motor root of CN V pass inferior to the trigeminal ganglion and blend with the
sensory fibers as the nerve traverses the foramen ovale — The THREE divisions of the
trigeminal nerve are:

Head & Neck (4TH semester—Unit-I PRACTICAL MANUAL) Page 139


a. Ophthalmic nerve (CN V1)
 The ophthalmic nerve is purely sensory — The sensory (somatic afferent) fibers of CN V1 are
distributed to skin and mucous membranes and conjunctiva of the front of the head and
nose — It runs forward in the lateral wall of the cavernous sinus and divides into THREE
branches--the lacrimal, frontal, and nasociliary nerves, which enter the orbital cavity through
the superior orbital fissure.
― 1. THE LACRIMAL NERVE: enters the lacrimal gland and gives branches to the
conjunctiva and the skin of the upper eyelid.
― 2. THE FRONTAL NERVE: divides into:
 Supraorbital and nerve: supply the frontal air sinus and the skin of the
forehead and the scalp
 Supratrochlear nerve: supply the frontal air sinus and the skin of the
forehead and the scalp
― 3. THE NASOCILIARY NERVE: continues as the anterior ethmoid nerve through the
anterior ethmoidal foramen to enter the cranial cavity— It then descends through a slit at
the side of the crista galli to enter the nasal cavity — It then supplies the skin of the tip of
the nose as external nasal nerve — The branches of nasociliary include the following:
 Sensory fibers to the ciliary ganglion.
 Long ciliary nerves that contain sympathetic fibers to the dilator pupillae
muscle and sensory fibers to the cornea.
 Infratrochlear nerve that supplies the skin of the eyelids.
 Posterior ethmoidal nerve that is sensory to the ethmoid and sphenoid
sinuses.
b. Maxillary nerve (CN V2)
 The maxillary nerve arises from the trigeminal ganglion — It passes forward in the lateral wall
of the cavernous sinus and leaves the skull through the foramen rotundum — It crosses the
pterygopalatine fossa to enter the orbit through the inferior orbital fissure — It then continues
as the infraorbital nerve in the infraorbital groove, and emerges on the face through the
infraorbital foramen — It gives off:
― 1. MENINGEAL BRANCHES:
― 2. ZYGOMATIC BRANCH: Which divides into:
 Zygomaticotemporal nerve: that supplies the skin of the face and
scalp.
 Zygomaticofacial nerve: that supplies the skin of the face.
― 3. GANGLIONIC BRANCHES: Which are two short nerves that suspend the
pterygopalatine ganglion in the pterygopalatine fossa — They contain sensory fibers
that have passed through the ganglion from the nose, the palate, and the pharynx —
They also contain postganglionic parasympathetic fibers that are going to the lacrimal
gland.
― 4. POSTERIOR SUPERIOR ALVEOLAR NERVE: Which supplies the maxillary sinus
as well as the upper molar teeth and adjoining parts of the gum and the cheek.
― 5. MIDDLE SUPERIOR ALVEOLAR NERVE: Which supplies the maxillary sinus as
well as the upper premolar teeth, the gums, and the cheek.
― 6. ANTERIOR SUPERIOR ALVEOLAR NERVE: Which supplies the maxillary sinus as
well as the upper canine and the incisor teeth.
PTERYGOPALATINE GANGLION
 The pterygopalatine ganglion is a parasympathetic ganglion, which is suspended from the
maxillary nerve in the pterygopalatine fossa — It is secretomotor to the lacrimal and nasal
glands — It gives off:
 7. ORBITAL BRANCHES: which enter the orbit through the inferior orbital fissure.
 8. GREATER AND LESSER PALATINE NERVES: which supply the palate, the tonsil, and the
nasal cavity.

Head & Neck (4TH semester—Unit-I PRACTICAL MANUAL) Page 140


 8. PHARYNGEAL BRANCH: which supplies the roof of the nasopharynx.
 9. SPHENOPALATINE BRANCH: which supplies the nasal cavity.

c. Mandibular nerve (CN V3)


 CN V3 is the only division of CN V to convey motor (branchial efferent) fibers, distributed to the
striated muscle derived from mandibular prominence mesoderm, primarily the muscles of
mastication — The mandibular nerve is both motor and sensory — The sensory root leaves
the trigeminal ganglion and passes out of the skull through the foramen ovale to enter the
infratemporal fossa — The motor root of the trigeminal nerve also leaves the skull through
the foramen ovale and joins the sensory root to form the trunk of the mandibular nerve, and then
divides into a small anterior and a large posterior division:
i. Branches from the main trunk of the mandibular nerve

― 1. MENINGEAL BRANCH: which returns through the foramen ovale or foramen


spinosus and supplies the dura mater of the middle cranial fossa.
― 2. NERVE TO THE MEDIAL PTERYGOID MUSCLE: which supplies not only the
medial pterygoid, but also the tensor veli palatini and tensor tympani muscles.

ii. Branches from the anterior division of the mandibular nerve

― 4. MASSETERIC NERVE: to the masseter muscle — It also give sensory fibres to


the TMJ
― 5. DEEP TEMPORAL NERVES: to the temporalis muscle.
― 6. NERVE TO THE LATERAL PTERYGOID MUSCLE: to the lateral pterygoid muscle.
― 7. BUCCAL NERVE: to the skin and the mucous membrane of the cheek and it is the
only sensory branch of the anterior division of the mandibular nerve.

iii. Branches from the posterior division of the mandibular nerve

― 8. AURICULOTEMPORAL NERVE: supplies the skin of the auricle, the external


auditory meatus and the scalp— It also give sensory fibres to the TMJ.
― 9. LINGUAL NERVE, which enters the mouth and runs forward on the side of the
tongue — It is joined by the chorda tympani nerve, and it supplies the mucous
membrane of the anterior two thirds of the tongue and the floor of the mouth.
― 10. INFERIOR ALVEOLAR NERVE: enters the mandibular canal to supply the teeth of
the lower jaw and emerges through the mental foramen (as mental nerve) to supply
the skin of the chin — Before entering the canal, it gives off the mylohyoid nerve,
which supplies the mylohyoid muscle and the anterior belly of the digastric muscle.

6. Cranial Exit: OPHTHALMIC (V1): Superior orbital fissure — MAXILLARY (V2):


Foramen rotundum— MANDIBULAR (V3): Foramen ovale.

7. Clinical correlates
Injury to the Trigeminal Nerve
 CN V may be injured by trauma, tumors, aneurysms, or meningeal infections — It may be
involved occasionally in poliomyelitis and generalized polyneuropathy, a disease process
involving several nerves — Injury to the CN V causes the following:
 Paralysis of the muscles of mastication with deviation of the mandible toward the side of the
lesion.
 Loss of the ability to appreciate soft tactile, thermal, or painful sensations in the face.
 Loss of corneal reflex (blinking in response to the cornea being touched) and the sneezing
reflex (stimulated by irritants to clear the respiratory tract).

Head & Neck (4TH semester—Unit-I PRACTICAL MANUAL) Page 141


Fig-53: Distribution of the trigeminal nerve.

VI.ABDUCENT NERVE—CN VI (Fig—54)


1. Functional components: Voluntary motor (GSE).
2. Main actions: Voluntary motor to lateral rectus (abducts the eyeball).
3. Site of attachment to brain: Pons.
4. Location of the cell bodies: The abducent nucleus is in the pons near
the median plane.
5. Distribution: The abducent nerves (CN VI) emerge from the brainstem between the
pons and the medulla and traverse the pontine cistern of the subarachnoid space — Each
abducent nerve then pierces the dura to run the longest intradural course within the cranial
cavity — it exits from the cranium via the superior orbital fissure — During its intradural
course, it bends sharply over the crest of the petrous part of the temporal bone and then
courses through the cavernous sinus — It traverses the common tendinous ring as it enters the
orbit, running on and penetrating the the lateral rectus, which abducts the eye (this function
being the basis for the name of the nerve).
6. Cranial Exit: Superior orbital fissure.
7. Clinical correlates
Injury to the Abducent Nerve
 Because CN VI has a long intracranial course, it is often stretched when intracranial
pressure rises, partly because of the sharp bend it makes over the crest of the petrous part

Head & Neck (4TH semester—Unit-I PRACTICAL MANUAL) Page 142


of the temporal bone after entering the dura — A space-occupying lesion, such as a brain
tumor, may compress CN VI, causing paralysis of the lateral rectus.

Fig-54: Origin and distribution of the abducent nerve.

VII.FACIAL NERVE—CN VII (Fig—55)


1. Functional components: 1. Special sensory (SVA)…….2. General sensory
(GSA)…..3. Branchial motor (SVE)……4. Parasympathetic (GVE).
2. Main actions: 1. Special sensory (SVA): Taste from anterior two thirds of
tongue and the palate….2. General sensory (GSA): General sensation from a part of the
external ear…..3. Branchial motor (SVE): to muscles of facial expression, stapedius of
middle ear, stylohyoid, and posterior belly of digastrics (all derived from 2ND pharyngeal arch)
….. 4. Parasympathetic (GVE): Parasympathetic to submandibular and sublingual salivary
glands, lacrimal gland, and glands of nose and palate.
3. Site of attachment to brain: Pons.
4. Location of the cell bodies: 1. Special sensory (SVA): Geniculate
ganglion….2. General sensory (GSA): Geniculate ganglion …..3. Branchial motor (SVE):
motor nucleus of the facial nerve located in the ventrolateral part of the pons ….. 4.
Parasympathetic (GVE): Superior salivatory nucleus (Preganglionic parasympathetic fibres)
located in the lower part of the pons.
5. Distribution: The facial nerve (CN VII) emerges from the junction of the pons and
medulla as two divisions— the motor root and the nervus intermedius — The larger motor
root (facial nerve proper) innervates the muscles of facial expression, and the smaller
nervus intermedius carries taste and parasympathetic fibers — During its course, CN VII
traverses the internal acoustic meatus, facial canal, stylomastoid foramen of the temporal
bone, and parotid gland — After traversing the internal acoustic meatus, the nerve turns
abruptly to course along the medial wall of the middle ear cavity, the sharp bend is called the
geniculum of the facial nerve, the site of the geniculate ganglion (sensory ganglion of CN VII)
― 1. GREATER PETROSAL NERVE: Greater petrosal nerve arises from the nerve at the
geniculate ganglion — It contains preganglionic parasympathetic fibers that synapse
in the pterygopalatine ganglion — The postganglionic fibers are secretomotor to the
lacrimal gland and the glands of the nose and the palate — The greater petrosal
nerve also contains taste fibers from the palate.
 While passing within the facial canal, CN VII gives rise to the:

Head & Neck (4TH semester—Unit-I PRACTICAL MANUAL) Page 143


― 2. NERVE TO THE STAPEDIUS: Nerve to stapedius supplies the stapedius muscle in
middle ear.
― 3. CHORDA TYMPANI NERVE: Chorda tympani enters the posterior wall of the
middle ear — It runs forward over the medial surface of tympanic membrane —
leaves the middle ear through the petrotympanic fissure to enter the infratemporal
fossa where it joins the lingual nerve — The chorda tympani contains preganglionic
parasympathetic fibers to submandibular and sublingual salivary glands — It also
contains taste fibers from anterior two thirds of the tongue.

 After running the longest intraosseous course of any cranial nerve, CN VII emerges from the
cranium via the stylomastoid foramen; gives off:
― 4. POSTERIOR AURICULAR BRANCH: which supplies occipital belly of
occipitofrontalis muscle.
― 5. MUSCULAR NERVE: which supply stylohyoid and posterior belly of digastric.
 Finally, it enters the parotid gland; and forms the parotid plexus, which gives rise to the
following five terminal motor branches to the muscles of facial expression:
― 6. TEMPORAL NERVE
― 7. ZYGOMATIC NERVE
― 8. BUCCAL NERVE
― 9. MARGINAL MANDIBULAR NERVE
― 10. CERVICAL NERVE
6. Cranial Exit: Internal acoustic meatus.
7. Clinical correlates
a. Injury to the Facial Nerve
 CN VII is the most frequently paralyzed of all the cranial nerves — Depending on the part of the nerve
involved, injury to CN VII may cause paralysis of facial muscles without loss of taste on the anterior two thirds of the
tongue, or altered secretion of the lacrimal and salivary glands — A lesion of CN VII near its origin or near
the geniculate ganglion is accompanied by loss of motor, gustatory (taste), and autonomic
functions — A central lesion of CN VII (lesion of the CNS) results in paralysis of muscles in the
inferior face on the contralateral side; consequently, forehead wrinkling is not visibly impaired
because it is innervated bilaterally — Lesions between the geniculate ganglion and the origin
of the chorda tympani produce the same effects as that resulting from injury near the ganglion,
except that lacrimal secretion is not affected — Because it passes through the facial canal in the
temporal bone, CN VII is vulnerable to compression when a viral infection produces inflammation
(viral neuritis) and swelling of the nerve just before it emerges from the stylomastoid foramen —
Because the branches of CN VII are superficial, they are subject to injury from knife and gunshot
wounds, cuts, and birth injury — Damage to CN VII is common with fracture of the temporal
bone and is usually detectable immediately after the injury — Although injuries to CN VII cause
paralysis of facial muscles, sensory loss in the small area of skin on the posteromedial surface
of the auricle and around the opening of the external acoustic meatus is rare — Similarly,
hearing is not usually impaired, but the ear may become more sensitive to low tones when the
stapedius (supplied by CN VII) is paralyzed; this muscle dampens vibration of the stapes.
b. Bell’s palsy
 Bell palsy is a unilateral facial paralysis of sudden onset resulting from a lesion of CN VII.
VIII.VESTIBULOCOCHLEAR NERVE (CN VIII) (Fig—
56)
1. Functional components: Special sensory (SSA).
2. Main actions: Vestibular sensation from semicircular ducts, utricle, and
saccule related to position and movement of head…….Hearing from Choclea
3. Site of attachment to brain: Cerebello-pontine angle.
Head & Neck (4TH semester—Unit-I PRACTICAL MANUAL) Page 144
4. Location of the cell bodies: VESTIBULAR: Bipolar neurons in the
vestibular ganglion…..COCHLEAR: Bipolar neurons in the spiral ganglion.
5. Distribution: The vestibulocochlear nerve (CN VIII) emerges from the junction of
the pons and medulla and enters the internal acoustic meatus — Here it separates into the
vestibular and cochlear nerves.
― THE VESTIBULAR NERVE is concerned with equilibrium — It is composed of the central
processes of bipolar neurons in the vestibular ganglion & the peripheral processes of the
neurons extend to the maculae of the utricle and saccule (sensitive to the linear
acceleration) and to the ampullae of the semicircular ducts (sensitive to rotational
acceleration).
― THE COCHLEAR NERVE is concerned with hearing — It is composed of the central
processes of bipolar neurons in the spiral ganglion & the peripheral processes of the
neurons extend to the spiral organ of Corti.
 Within the internal acoustic meatus, the two divisions of CN VIII are accompanied by the motor
root and nervus intermedius of CN VII and the labyrinthine artery.
6. Cranial Exit: Internal acoustic meatus
7. Clinical correlates
a. Injuries of the Vestibulocochlear Nerve
 Although the vestibular and cochlear nerves are essentially independent, peripheral lesions
often produce concurrent clinical effects because of their close relationship — Hence lesions of
CN VIII may cause tinnitus (ringing or buzzing in ears), vertigo (dizziness, loss of balance),
and impairment or loss of hearing — Central lesions may involve either the cochlear or
vestibular divisions of CN VIII
b. Deafness
 There are two kinds of deafness: CONDUCTIVE DEAFNESS: involving the external or middle
ear (e.g., otitis media, inflammation in the middle ear) & SENSORINEURAL DEAFNESS: which
results from disease in the cochlea or in the pathway from the cochlea to the brain.
c. Acoustic Neuroma
An acoustic neuroma is a slow-growing benign tumor of the neurolemma (Schwannoma) — The
tumor begins in the vestibular nerve while it is in the internal acoustic meatus, but the early
symptom of an acoustic neuroma is usually loss of hearing — Dysequilibrium and tinnitus occur
in approximately 70% of patients.

Head & Neck (4TH semester—Unit-I PRACTICAL MANUAL) Page 145


Fig-55: A. Distribution of the facial nerve. B. Branches of the facial nerve within the petrous
part of the temporal bone; the taste fibers are shown in black. The glossopharyngeal nerve is
also shown.

Head & Neck (4TH semester—Unit-I PRACTICAL MANUAL) Page 146


Fig-56: Origin and distribution of the vestibulocochlear nerve.

IX.GLOSSOPHARYNGEAL NERVE—CN IX (Fig—57)


1. Functional components: 1. General sensory (GSA)……2. Special sensory
(SVA)……3. Visceral afferent (GVA)……4. Branchial motor (SVE)…….5. Parasympathetic (GVE).
2. Main actions: 1. General sensory (GSA): Cutaneous sensation from external
ear…..2. Special sensory (SVA): Taste from posterior third of tongue…..3. Visceral afferent
(GVA): from parotid gland, carotid body and sinus, pharynx, and middle ear….4. Branchial
motor (SVE): to stylopharyngeus (derived from 3RD pharyngeal arch) to assist in swallowing…..5.
Parasympathetic (GVE): to parotid gland.
3. Site of attachment to brain: Medulla.
4. Location of the cell bodies: 1. General sensory (GSA): inferior and
superior ganglia of the CN IX…..2. Special sensory (SVA): inferior and superior ganglia of
the CN IX…..3. Visceral afferent (GVA): inferior and superior ganglia of the CN IX ….4.
Branchial motor (SVE): Nucleus ambiguous located in the ventrolateral part of the medulla.
…..5. Parasympathetic (GVE): Inferior salivatory nucleus (Preganglionic fibres).
5. Distribution: The glossopharyngeal nerve (CN IX) emerges from the lateral
aspect of the medulla and leaves the cranium through the anterior aspect of the jugular
foramen — At this foramen are superior and inferior (sensory) ganglia — Within the jugular
foramen, it gives off:
― 1. TYMPANIC NERVE which enters the middle ear through the tympanic canaliculi
and splits into tympanic plexus (located over the promontory) — Preganglionic
parasympathetic fibers for the parotid gland now leave the tympanic plexus as
lesser petrosal nerve, enters the middle cranial fossa and exits via foramen ovale
to synapse the neurons in the otic ganglion — Postganglionic parasympathetic
fibers for the parotid gland are carried by the auriculotemporal nerve.
 Exiting the jugular foramen, CN IX follows the stylopharyngeus, and gives off:
― 2. NERVE TO STYLOPHARYNGEUS
― 3. THE CAROTID SINUS NERVE to the carotid sinus, a baroreceptor, sensitive to
changes in blood pressure.
― 4. THE CAROTID BODY NERVE to the carotid body, a chemoreceptor sensitive to
blood gas (low oxygen levels)
 CN IX then passes between the superior and the middle constrictor muscles of the pharynx
to reach the oropharynx and tongue, by giving off:
― 5. PHARYNGEAL BRANCHES which contribute sensory fibers to the pharyngeal
plexus of nerves.

Head & Neck (4TH semester—Unit-I PRACTICAL MANUAL) Page 147


― LINGUAL BRANCHES which supply the mucosa of the oropharynx and isthmus
of the fauces, including palatine tonsil, soft palate, and posterior third of the
tongue (general and taste sensation).
6. Cranial Exit: Jugular foramen.

Fig-57: Distribution of the glossopharyngeal nerve.

X.VAGUS NERVE—CN X (Fig—58)


1. Functional components: 1. General sensory (GSA)……2. Special sensory
(SVA)……3. Visceral afferent (GVA)…… 4. Branchial motor (SVE).…….5. Parasympathetic
(GVE).
2. Main actions: 1. General sensory (GSA): from a part of the tympanic membrane in
the external ear….….2. Special sensory (SVA): Taste from the root of the tongue and taste buds
on the epiglottis.…….. 3. Visceral afferent (GVA): from inferior pharynx, larynx, and thoracic
and abdominal organs…….4. Branchial motor (SVE): to the soft palate (except tensor veli
palatini); pharynx (except stylopharyngeus); intrinsic laryngeal muscles; and one tongue muscle
(palatoglossus), all derived from 4TH & 6TH pharyngeal arches…..5. Parasympathetic (GVE): to
thoracic and abdominal viscera.
3. Site of attachment to brain: Medulla.
4. Location of the cell bodies: 1. General sensory (GSA): inferior and
superior ganglia of the CN X…..2. Special sensory (SVA): inferior and superior ganglia of
the CN X…..3. Visceral afferent (GVA): inferior and superior ganglia of the CN X ….4.
Branchial motor (SVE): Nucleus ambiguous located in the ventrolateral part of the medulla.
…..5. Parasympathetic (GVE): Dorsal vagus nucleus (Preganglionic fibres).

Head & Neck (4TH semester—Unit-I PRACTICAL MANUAL) Page 148


5. Distribution: The vagus nerve (CN X) has the longest course and most extensive
distribution of all the cranial nerves, most of which is outside of the head — It arises by a
series of rootlets from the lateral aspect of the medulla that merge and leave the cranium
through the jugular foramen positioned between CN IX and CN XI — WHAT WAS FORMERLY
CALLED THE CRANIAL ROOT OF THE ACCESSORY NERVE IS ACTUALLY A PART OF CN
X — CN X has a superior ganglion in the jugular foramen that is mainly concerned with the
general sensory component of the nerve — Inferior to the jugular foramen is an inferior
ganglion (nodose ganglion) concerned with the visceral sensory components of the nerve
 Within the jugular foramen, it gives off:
― 1. MENINGEAL NERVE: to supply the dura mater of the posterior cranial
fossa.
― 2. AURICULAR NERVE: which leaves the jugular foramen, enters the
auricular canaliculi to supply a part of tympanic membrane……(Both
these sensory nerves have their cell bodies located in the superior ganglion,
whereas all other sensory nerves have their cells bodies located in the
inferior ganglion).
 CN X continues inferiorly in the carotid sheath to the root of the neck.
 Within the neck, it gives off:
― 3. CARDIAC BRANCHES: which descend into the thorax, and end in the
cardiac plexus.
― 4. PHARYNGEAL BRANCH: which joins the pharyngeal plexus and
supplies all the muscles of the pharynx (except the stylopharyngeus) and of
the soft palate (except the tensor veli palatini).
― 5. SUPERIOR LARYNGEAL NERVE which divides into:
 The internal laryngeal nerve: which is sensory to the mucous
membrane of the piriform fossa and the larynx down as far as the
vocal cords.
 The external laryngeal nerve: which is motor and is located close
to the superior thyroid artery; it supplies the cricothyroid muscle.
― 6. RECURRENT LARYNGEAL NERVE:
 On the right side, the nerve hooks around the first part of the
subclavian artery and then ascends in the groove between the
trachea and the esophagus.

 On the left side, the nerve hooks around the arch of the aorta and
then ascends into the neck between the trachea and the
esophagus.
 The recurrent laryngeal nerve is closely related to the inferior
thyroid artery, and it supplies all the muscles of the larynx,
except the cricothyroid muscle, the mucous membrane of the
larynx below the vocal cords, and the mucous membrane of the
upper part of the trachea.

 Vagi enter thorax through superior thoracic aperture; left vagus contributes to anterior
esophageal plexus; right vagus to posterior plexus; form anterior and posterior trunks.
 In thorax, it gives off:
― Pulmonary branches
― Esophageal plexus
 The esophageal plexus follows the esophagus through the diaphragm into the abdomen, where
the anterior and posterior vagal trunks break up into branches that innervate the esophagus,
stomach, and intestinal tract as far as the left colic flexure.
6. Cranial Exit: Jugular foramen

Head & Neck (4TH semester—Unit-I PRACTICAL MANUAL) Page 149


7. Clinical correlates
Lesions of the Vagus Nerve
 Isolated lesions of CN X are uncommon — Injury to pharyngeal branches of CN X results in
dysphagia (difficulty in swallowing) — Lesions of the superior laryngeal nerve produce
anesthesia of the superior part of the larynx and paralysis of the cricothyroid muscle —
The voice is weak and tires easily — Injury of a recurrent laryngeal nerve may be caused by
aneurysms of the arch of the aorta and may occur during neck operations — Injury of the
recurrent laryngeal nerve causes hoarseness and dysphonia (difficulty in speaking) because
of paralysis of the vocal folds (cords) — Paralysis of both recurrent laryngeal nerves
causes aphonia (loss of voice) and inspiratory stridor (a harsh, high pitched respiratory
sound). — Paralysis of recurrent laryngeal nerves usually results from cancer of the larynx
and thyroid gland and/or from injury during surgery on the thyroid gland, neck, esophagus,
heart, and lungs. — Because of its longer course, lesions of the left recurrent laryngeal nerve
are more common than those of the right.
XI.ACCESSORY NERVE (CN XI) (Fig—59)
1. Functional components: Voluntary motor.
2. Main actions; CRANIAL ROOT: the traditional cranial root of CN XI is actually a
part of CN X— It is united for a short distance with the spinal root of CN XI—It supplies motor
fibres to pharyngeal and laryngeal muscles via the branches of the vagus nerve……SPINAL
ROOT: motor to sternocleidomastoid and trapezius muscles.
3. Site of attachment to brain: Medulla
4. Location of nerve cell bodies: CRANIAL ROOT: Nucleus ambiguous
situated in the medulla…..SPINAL ROOT: Anterior horn motor neurons in upper five or six
cervical segments of the spinal cord.
5. Distribution: The accessory nerve is a motor nerve.
 It consists of a cranial root and a spinal root.
a. Cranial root
 The traditional cranial root of CN XI is actually a part of CN X — It emerges from the anterior
surface of the medulla oblongata — The nerve runs laterally in the posterior cranial fossa and
joins the spinal root.
b. Spinal root
 The spinal root arises from anterior horn of upper five cervical segments of the spinal cord
— The nerve ascends alongside the spinal cord and enters the skull through the foramen
magnum — It then turns laterally to join the cranial root — The two roots unite and leave the
skull through the jugular foramen.
 The two roots then separate:
a. Cranial Part
 The cranial part joins the vagus nerves and is distributed in its branches to the muscles of the
soft palate and pharynx (via the pharyngeal plexus) and to the muscles of the larynx (via
superior laryngeal and recurrent laryngeal nerves).
b. Spinal part
 The spinal part enters the deep surface of the sternocleidomastoid muscle, which it
supplies, and then crosses the posterior triangle of the neck to supply the trapezius muscle.
6. Cranial Exit: Jugular foramen.
7. Clinical correlates
Injury to the Spinal Accessory Nerve
 Because of its nearly subcutaneous passage through the posterior cervical region, CN XI is
susceptible to injury during surgical procedures such as lymph node biopsy, cannulation of
the internal jugular vein, and carotid endarterectomy.

Head & Neck (4TH semester—Unit-I PRACTICAL MANUAL) Page 150


Fig-58: Distribution of the vagus nerve.

Fig-59: Origin and distribution of the accessory nerve.

Head & Neck (4TH semester—Unit-I PRACTICAL MANUAL) Page 151


XII.HYPOGLOSSAL NERVE (CN XII) (Fig—60)
1. Functional components: Somatic motor (GSE).
2. Main actions: Somatic motor to all the intrinsic and extrinsic muscles of the
tongue except palatoglossus.
3. Site of attachment to brain: Medulla.
4. Location of nerve cell bodies: The hypoglossal nucleus is in the
medulla near the median plane.
5. Distribution: The hypoglossal nerve (CN XII) arises as a purely motor nerve by
several rootlets from the medulla and leaves the cranium through the hypoglossal canal
 After exiting the cranial cavity, it is joined by a branch (C1) of the cervical plexus conveying
motor fibers to the hyoid muscles & with some of the sensory fibers passing retrograde along
the CN XII to reach the dura mater of the posterior cranial fossa.
 CN XII passes inferiorly deep to mylohyoid muscle and then curves anteriorly to enter
tongue.
 CN XII ends in many branches that supply all the extrinsic muscles of the tongue, except the
palatoglossus.
 CN XII has the following branches:
― 1. A MENINGEAL BRANCH (C1): returns to the cranium through the hypoglossal
canal and innervates dura mater on the floor and posterior wall of the posterior
cranial fossa.
― 2. THE SUPERIOR ROOT OF THE ANSA CERVICALIS: branches from CN XII to supply
the infrahyoid muscles (sternohyoid, sternothyroid, and omohyoid) — This branch
actually conveys only fibers from the cervical plexus — Some fibers continue to reach
the thyrohyoid and geniohyoid muscles.
― 3. TERMINAL LINGUAL BRANCHES: supply the hyoglossus, genioglossus,
styloglossus, and all intrinsic muscles of the tongue.

6. Cranial Exit: Hypoglossal canal


7. Clinical correlates
Injury to the Hypoglossal Nerve
 Injury to CN XII paralyzes the ipsilateral half of the tongue — After some time, the tongue
atrophies, making it appear shrunken and wrinkled — When the tongue is protruded, its apex
deviates toward the paralyzed side because of the unopposed action of the genioglossus
muscle on the normal side of the tongue.
XIII.REVIEW QUESTIONS
Q.1: AN UNCONSCIOUS 48-YEAR-OLD WOMAN IS ADMITTED TO THE HOSPITAL. CT SCAN
REVEALS A TUMOR IN HER BRAIN. WHEN SHE REGAINS CONSCIOUSNESS, HER RIGHT EYE IS
DIRECTED LATERALLY AND DOWNWARD, WITH COMPLETE PTOSIS OF HER UPPER EYELID,
AND HER PUPIL IS DILATED. WHICH OF THE FOLLOWING STRUCTURES WAS MOST LIKELY
AFFECTED BY THE TUMOR TO RESULT IN THESE SYMPTOMS:
A. Oculomotor nerve
B. Optic nerve
C. Facial nerve
D. Ciliary ganglion
E. Superior cervical ganglion
ANSWER: A.
 An injury to the oculomotor nerve would cause the eye to point downward and laterally due to the
unopposed contractions of the trochlear and abducens nerves. The oculomotor nerve also provides
innervation to the levator palpebrae superioris; thus, any injury would cause complete ptosis or
drooping of the eyelid. The constriction of the pupil is provided by parasympathetic nerves via the
oculomotor nerve.

Head & Neck (4TH semester—Unit-I PRACTICAL MANUAL) Page 152


Fig-60: Distribution of the hypoglossal nerve

Q.2: A 33-YEAR-OLD WOMAN IS UNCONSCIOUS WHEN SHE IS ADMITTED TO THE HOSPITAL


AFTER SHE FELL, HITTING HER HEAD. THE PHYSICIAN IN THE EMERGENCY DEPARTMENT
PERFORMS A PUPILLARY LIGHT REFLEX TEST. THE INTEGRITY OF WHICH OF THE
FOLLOWING NERVES IS BEING CHECKED?
A. Optic and facial
B. Optic and oculomotor
C. Maxillary and facial
D. Ophthalmic and oculomotor
E. Ophthalmic and facial
ANSWER: B.
 The optic and oculomotor nerves are responsible for the sensory and motor portions, respectively,
of the pupillary light reflex.
Q.3: A 44-YEAR-OLD PATIENT IS ADMITTED TO THE HOSPITAL WITH RAYNAUD’S DISEASE.
A SYMPATHETIC BLOCKING DRUG IS ADMINISTERED IN HIGH DOSES. WHICH OF THE
FOLLOWING CONDITIONS WILL BE EXPECTED TO OCCUR AS AN ADVERSE EFFECT OF THE
DRUG?
A. Exophthalmos and dilated pupil
B. Enophthalmos and dry eye
C. Dry eye and inability to accommodate for reading
D. Wide-open eyelids and loss of depth perception
E. Ptosis and miosis
ANSWER: E.
 Ptosis and miosis occur in response to blocking of sympathetic innervation. Ptosis (drooping of
the eyelid) results from lack of innervation of the superior tarsal muscle (of Müller), and miosis
(papillary constriction) results from unopposed parasympathetic innervation of the pupil.

Head & Neck (4TH semester—Unit-I PRACTICAL MANUAL) Page 153


Q.4: A 65-YEAR-OLD MAN IS ADMITTED TO THE EMERGENCY DEPARTMENT AFTER AN
EPISODE OF A TRANSIENT ISCHEMIC ATTACK. RADIOGRAPHIC EXAMINATION REVEALS AN
ANEURYSM IN THE REGION BETWEEN THE POSTERIOR CEREBRAL ARTERY AND
SUPERIOR CEREBELLAR ARTERY. WHICH OF THE FOLLOWING NERVES WILL MOST LIKELY
BE COMPRESSED FROM THE ANEURYSM?
A. Trochlear
B. Abducens
C. Oculomotor
D. Vagus
E. Optic
ANSWER: C.
 The oculomotor nerve passes between the posterior cerebral artery (PCA) and the superior
cerebellar artery near the junction of the midbrain and pons.
Q.5: A 56-YEAR-OLD FEMALE COMPLAINS OF DIPLOPIA (DOUBLE VISION) WHEN WALKING
DOWN STAIRS. A LESION OF WHICH OF THE FOLLOWING NERVES IS MOST LIKELY
RESPONSIBLE FOR THIS PATIENT’S COMPLAINT?
A. Optic
B. Oculomotor
C. Abducens
D. Trochlear
E. Frontal
ANSWER: D.
 The trochlear nerve innervates the superior oblique muscle, which acts to move the pupil
downward and laterally. It is the only muscle that can depress the pupil when the eye is adducted.
When an individual walks down stairs, this eye motion is initiated, and diplopia results if it is not
functioning properly.
Q.6: A 43-YEAR-OLD MALE IS ADMITTED TO THE HOSPITAL COMPLAINING OF DIPLOPIA
(DOUBLE VISION) WHEN WALKING DOWN STAIRS. DURING PHYSICAL EXAMINATION OF
THE EXTRAOCULAR MUSCLES THE PATIENT EXPERIENCES DIPLOPIA, AND WHEN HE IS
ASKED TO TURN HIS RIGHT EYE INWARD TOWARD HIS NOSE AND LOOK DOWN, HE IS
ABLE TO LOOK INWARD BUT NOT DOWN. WHICH NERVE IS MOST LIKELY INVOLVED?
A. Abducens
B. Nasociliary
C. Oculomotor, inferior division
D. Oculomotor, superior division
E. Trochlear
ANSWER: E.
 The superior oblique muscle turns the pupil downward from the adducted position. Inability to
perform this motion, in conjunction with diplopia when walking down stairs, indicates damage to
the trochlear nerve.
Q.7: A 34-YEAR-OLD WOMAN IS ADMITTED TO THE EMERGENCY DEPARTMENT AFTER HER
RIGHT CHEEK BONE AND BONY ORBIT HIT THE DASHBOARD IN AN AUTOMOBILE CRASH.
PHYSICAL EXAMINATION REVEALS THAT THE PATIENT HAS LOST THE ABILITY FOR THE
AFFECTED EYE TO BE DIRECTED DOWNWARD WHEN THE PUPIL IS IN THE ADDUCTED
POSITION. AN MRI EXAMINATION REVEALS A TORN NERVE. WHAT IS THE MOST COMMON
LOCATION AT WHICH THIS NERVE WILL BE INJURED?
A. As it pierces the dura of the tentorium cerebella in the tentorial notch
B. At the cavernous sinus
C. At the sella turcica
D. At the inferior orbital fi ssure
E. At the superior orbital fi ssure
ANSWER: A.
 Paralysis of the trochlear nerve results in loss of ability for the affected eye to be directed
downward when the pupil is in the adducted position (the primary action of the superior oblique
muscle).
Q.8: A 50-YEAR-OLD WOMAN COMPLAINED OF PAIN OVER HER CHIN AND LOWER LIP. A
FEW DAYS LATER SMALL VESICLES APPEARED OVER THE SAME AREA AND SOON BEGAN
ERUPTING. SHE WAS DIAGNOSED WITH A DERMATOMAL HERPES ZOSTER INFLAMMATION
(SHINGLES). WHICH OF THE FOLLOWING NERVES WAS MOST LIKELY RESPONSIBLE FOR
THE TRANSMISSION OF THE VIRUS IN THIS CASE?

Head & Neck (4TH semester—Unit-I PRACTICAL MANUAL) Page 154


A. Auriculotemporal
B. Buccal
C. Lesser petrosal
D. Mental
E. Infraorbital
ANSWER: D.
 The chin and lower lip area are supplied by the mental nerve, a branch of the inferior alveolar
nerve, which in turn is a branch of the mandibular division of the trigeminal nerve.
Q.9: A 68-YEAR-OLD WOMAN IS SUFFERING FROM EXCRUCIATING, SUDDEN BOUTS OF
PAIN OVER THE AREA OF HER MIDFACE. LABORATORY STUDIES INDICATE THAT SHE HAS
TIC DOULOUREUX (TRIGEMINAL NEURALGIA). WHICH GANGLION IS THE LOCATION OF THE
NEURAL CELL BODIES OF THE NERVE MEDIATING THE PAIN?
A. Geniculate
B. Trigeminal (semilunar or Gasserian)
C. Inferior glossopharyngeal
D. Otic
E. Pterygopalatine
ANSWER: B.
 The semilunar ganglion, also known as the trigeminal or Gasserian ganglion, is the location of the
sensory neuron cell bodies of the trigeminal nerve. Tic douloureux is a condition in which pain
occurs over the area of distribution of trigeminal nerve branches.

Q.10: A 17-YEAR-OLD WOMAN IS ADMITTED TO THE HOSPITAL WITH SIGNS OF


CAVERNOUS SINUS THROMBOSIS, AS REVEALED BY RADIOGRAPHIC AND PHYSICAL
EXAMINATIONS. THROMBOPHLEBITIS IN THE “DANGER AREA” OF THE FACE CAN SPREAD
TO THE CAVERNOUS SINUS AND INVOLVE THE OPHTHALMIC BRANCH OF THE TRIGEMINAL
NERVE. WHICH OF THE FOLLOWING SYMPTOMS WILL MOST LIKELY BE PRESENT DURING
PHYSICAL EXAMINATION?
A. Pain in the hard palate
B. Anesthesia of the upper lip
C. Pain from the eyeball
D. Pain over the lower eyelid
E. Tingling sensation over the buccal region of the face
ANSWER: C.
 The ophthalmic branch of the trigeminal nerve supplies sensory innervation to the eyeball, leading
to pain upon damage.
Q.11: A 35-YEAR-OLD MALE PATIENT IS ADMITTED TO THE HOSPITAL WITH SEVERE
HEADACHES. A CT SCAN EVALUATION REVEALS A TUMOR IN THE INFRATEMPORAL
FOSSA. PHYSICAL EXAMINATION REVEALS LOSS OF GENERAL SENSATION FROM THE
ANTERIOR TWO THIRDS OF HIS TONGUE, BUT TASTE AND SALIVATION ARE INTACT.
WHICH OF THE FOLLOWING NERVES IS MOST LIKELY AFFECTED BY THE TUMOR?
A. Lingual proximal to its junction with the chorda tympani
B. Chorda tympani
C. Inferior alveolar
D. Lesser petrosal
E. Glossopharyngeal
ANSWER: A.
 The lingual nerve joins the chorda tympani in the infratemporal fossa, and a lesion to the lingual
nerve before it joins the chorda tympani would account for the loss of general sensation, with no
loss to the special sense of taste and saliva production.
Q.12: A 48-YEAR-OLD MALE PATIENT COMPLAINS OF DIPLOPIA (DOUBLE VISION). ON
NEUROLOGIC EXAMINATION HE IS UNABLE TO ADDUCT HIS LEFT EYE AND LACKS A
CORNEAL REFLEX ON THE LEFT SIDE. WHERE IS THE MOST LIKELY LOCATION OF THE
LESION RESULTING IN THE SYMPTOMS?
A. Inferior orbital fissure
B. Optic canal
C. Superior orbital fissure
D. Foramen rotundum
E. Foramen ovale
ANSWER: C.

Head & Neck (4TH semester—Unit-I PRACTICAL MANUAL) Page 155


 The superior orbital fissure is the opening that allows the passage of the oculomotor nerve and
the trochlear nerve; the lacrimal, frontal, and nasociliary branches of ophthalmic division of the
trigeminal nerve; the abducens nerve; the superior and inferior divisions of the ophthalmic vein;
and the sympathetic fibers from the cavernous plexus. The sensory and motor components of the
corneal reflex are the ophthalmic division of the trigeminal nerve and the oculomotor nerve,
whereas the eye impairment is due to a lesion to the oculomotor nerve, all of which are
transmitted through the superior orbital fissure.
Q.13: A 24-YEAR-OLD MALE HAD A THIRD MOLAR (WISDOM TOOTH) EXTRACTED FROM HIS
LOWER JAW. THIS RESULTED IN THE LOSS OF GENERAL SENSE AND TASTE SENSATION
FROM THE ANTERIOR TWO THIRDS OF THE TONGUE. THIS LOSS WAS MOST LIKELY DUE
TO INJURY OF WHICH OF THE FOLLOWING NERVES?
A. Auriculotemporal
B. Chorda tympani
C. Lingual
D. Mental
E. Inferior alveolar
ANSWER: C.
 The lingual nerve is the most likely nerve damaged because there is loss both of taste and
general sensory supply to the anterior two thirds of the tongue, which is innervated by the lingual
nerve, which at this point has been joined by the chorda tympani. The chorda tympani would be a
likely choice; however, it carries only taste and does not mediate other general sensation to the
tongue.
Q.14: A 56-YEAR-OLD WOMAN IS ADMITTED TO THE HOSPITAL WITH RHEUMATOID
ARTHRITIS OF HER TEMPOROMANDIBULAR JOINT (TMJ) AND SEVERE EAR PAIN. WHICH OF
THE FOLLOWING NERVES IS MOST LIKELY RESPONSIBLE FOR CONDUCTING THE PAIN
SENSATION?
A. Facial
B. Auriculotemporal
C. Lesser petrosal
D. Vestibulocochlear
E. Chorda tympani
ANSWER: B.
 The auriculotemporal nerve is a posterior branch of the mandibular division of the trigeminal
nerve. It encircles the middle meningeal artery and courses medially to the TMJ and then ascends
up near the auricle. Because this nerve supplies the TMJ and skin of the external auditory canal,
pain from the joint can be referred to the ear as in this case.

Q.15: A 38-YEAR-OLD PATIENT IS ADMITTED TO THE DENTAL CLINIC WITH ACUTE DENTAL
PAIN. THE ATTENDING DENTIST FOUND PENETRATING DENTAL CARIES (TOOTH DECAY)
AFFECTING ONE OF THE MANDIBULAR MOLAR TEETH. WHICH OF THE FOLLOWING
NERVES WOULD THE DENTIST NEED TO ANESTHETIZE TO REMOVE THE CARIES IN THAT
TOOTH?
A. Lingual
B. Inferior alveolar
C. Buccal
D. Mental
E. Mylohyoid
ANSWER: B.
 The inferior alveolar branch of the mandibular division of the trigeminal nerve provides sensory
innervations to the mandibular teeth and would require anesthesia to abolish painful sensation.
Q.16: A 52-YEAR-OLD MAN IS ADMITTED TO THE EMERGENCY DEPARTMENT WITH A BULLET WOUND
IN THE INFRATEMPORAL FOSSA. DURING PHYSICAL EXAMINATION IT IS OBSERVED THAT THE
PATIENT HAS LOST UNILATERAL SENSATION OF HOT, COLD, PAIN, AND PRESSURE FROM THE
FRONT PART OF THE TONGUE, BUT TASTE AND SALIVARY FUNCTION ARE PRESERVED. WHICH OF
THE FOLLOWING IS THE MOST LIKELY DIAGNOSIS?
A. The facial nerve was transected distal to the origin of the chorda tympani.
B. Receptors for hot, cold, pain, and pressure are absent in the patient’s tongue.
C. The glossopharyngeal nerve has been injured in the pharynx.
D. The superior laryngeal nerve was obviously severed by the bullet.
E. The lingual nerve was injured at its origin near the foramen ovale.
ANSWER: E.

Head & Neck (4TH semester—Unit-I PRACTICAL MANUAL) Page 156


 The lingual nerve supplies sensory innervations to the mucous membrane of the anterior two thirds of the
tongue, taste sensation to the anterior part of the tongue, and parasympathetic fibers to the oral salivary
glands. The chorda tympani branch of the facial nerve is responsible for carrying taste fibers from the
anterior two thirds of the tongue and preganglionic parasympathetic fibers for the submandibular ganglion.
Injury to the lingual nerve at its origin, before it joins with the chorda tympani, will result in loss of general
sensation of the tongue, but with preservation of taste and salivary function.
Q.17: A 24-YEAR-OLD MAN IS ADMITTED TO THE HOSPITAL AFTER A STREET FI GHT.
RADIOGRAPHIC EXAMINATION REVEALS AN INFERIOR BLOW-OUT FRACTURE OF THE
ORBIT. WHICH OF THE FOLLOWING NERVES IS PARTICULARLY VULNERABLE TO THIS
TYPE OF INJURY?
A. Infraorbital
B. Supratrochlear
C. Frontal
D. I nferior alveolar
E. Optic
ANSWER: A.
 An inferior fracture of the orbit would likely damage the infraorbital nerve. A blow-out fracture
often results in a displaced orbital wall, and in this case, the inferior wall. The infraorbital nerve
leaves the skull immediately inferior to the inferior aspect of the orbit, via the infraorbital foramen.
Thus, this nerve is the most likely to be damaged.
Q.18: A 36-YEAR-OLD MAN IS ADMITTED TO THE EMERGENCY DEPARTMENT WITH A
PAINFUL SKIN RASH ON THE DORSUM OF HIS NOSE. PHYSICAL EXAMINATION REVEALS
THAT A HERPETIC LESION IS AFFECTING THE DORSUM OF THE NOSE AND THE EYEBALL.
WHICH OF THE FOLLOWING NERVES IS MOST LIKELY TO BE RESPONSIBLE FOR
TRANSMISSION OF THE VIRUS TO THE EYE?
A. Nasociliary
B. Supratrochlear
C. Infraorbital
D. Posterior ethmoidal
E. Anterior ethmoidal
ANSWER: A.
 A herpes rash on the dorsum of the nose is known as Hutchinson’s sign. This indicates that the
virus is located in cell bodies of the ophthalmic division of the trigeminal nerve. This nerve
branches into nasociliary, frontal, and lacrimal branches. The nasociliary nerve has direct
branches that carry sensory innervations from the eye. The nasociliary nerve also gives off the
ethmoidal nerves that innervate the superior nasal mucosa, in addition to providing the origin of
the dorsal nasal nerve.
Q.19: A 22-YEAR-OLD MAN IS ADMITTED TO THE EMERGENCY DEPARTMENT AFTER HE WAS BEATEN
UP IN A STREET FI GHT. RADIOGRAPHIC EXAMINATION REVEALS THAT HE HAS SUFFERED A
FOREHEAD FRACTURE FROM A BLOW WITH A CLUB, RESULTING IN BLACK AND SWOLLEN EYES
( FIG. 7-6 ). BECAUSE THE PATIENT IS SUFFERING FROM SEVERE PAIN, AN ANESTHETIC SOLUTION IS
ORDERED TO BE INJECTED INTO HIS ORBIT. WHICH OF THE FOLLOWING NERVES IS MOST LIKELY TO
BE ANESTHETIZED?
A. Ophthalmic
B. Infraorbital
C. A nterior ethmoidal
D. Frontal
E. Optic
ANSWER: A.
 Anesthetics are injected into the submuscular layer of delicate (areolar) connective tissue, the layer that
contains nerves of the eyelid. This space is continuous with the “danger zone” of the scalp. A blow to the
forehead can result in a “black eye,” with the passage of blood into the submuscular space. Infections can,
likewise, pass within this space.
Q.20: A 6-YEAR-OLD BOY IS ADMITTED TO THE HOSPITAL WITH HIGH FEVER AND PAIN OVER THE
PAROTID GLAND ( FIG. 7-7 ). A DIAGNOSIS OF PAROTIDITIS (MUMPS) IS ESTABLISHED, AND THE BOY
IS SENT BACK HOME. WHICH OF THE FOLLOWING NERVES IS RESPONSIBLE FOR PAINFUL
SENSATIONS FROM THE REGION OF THE PAROTID GLAND?
A. Facial
B. Auriculotemporal
C. L esser petrosal
D. Lingual
E. C horda tympani
ANSWER: B.

Head & Neck (4TH semester—Unit-I PRACTICAL MANUAL) Page 157


 The auriculotemporal nerve leads into the parotid gland, and its compression in mumps can be associated
with severe pain. The compressive effects are due in large part to the continuity of the facial capsule of the
parotid gland with the tough layer of superfi cial investing fascia of the neck, a layer that is almost non-
distensible. When the gland swells, sensory fibers for pain are triggered rapidly, and can be referred to the
ear.
Q.21: A 54-YEAR-OLD MAN IS ADMITTED TO THE HOSPITAL WITH SEVERE HEADACHES. A
CT SCAN REVEALS A TUMOR IN HIS BRAIN OCCUPYING A PORTION OF THE ANTERIOR
CRANIAL FOSSA. WHICH OF THE FOLLOWING IS RESPONSIBLE FOR THE SENSATION OF
PAIN FROM HEADACHE IN THIS CASE?
A. Meningeal branches of the maxillary nerve
B. Meningeal branches of the mandibular nerve
C. Meningeal branches of the ethmoidal nerve
D. Tentorial nerve
E. C2 and C3 fi bers
ANSWER: C.
 A tumor involving the meningeal branches of the ethmoidal nerves that originate from the
ophthalmic division of the trigeminal nerve is likely to cause pain from pressure and nerve injury in
the anterior cranial fossa.
Q.22: A 54-YEAR-OLD MAN IS ADMITTED TO THE HOSPITAL DUE TO SEVERE HEADACHES.
A CT EXAMINATION REVEALS AN INTERNAL CAROTID ARTERY ANEURYSM INSIDE THE
CAVERNOUS SINUS. WHICH OF THE FOLLOWING NERVES WOULD BE TYPICALLY
AFFECTED FIRST?
A. Abducens nerve
B. Oculomotor nerve
C. Ophthalmic nerve
D. Maxillary nerve
E. Trochlear nerve
ANSWER: A.
 The abducens nerve would be affected first due to aneurysmal dilation of the internal carotid
artery (ICA) because the nerve runs in closest proximity to the artery within the cavernous sinus.
Q.23: A 68-YEAR-OLD MAN IS ADMITTED TO THE EMERGENCY DEPARTMENT AFTER AN
ACUTE CEREBRAL VASCULAR ACCIDENT (STROKE). RADIOGRAPHIC STUDIES REVEAL
THAT THE PRIMARY DAMAGE WAS TO THE ANTERIOR INFERIOR CEREBELLAR ARTERY,
RESULTING IN A SMALL HEMORRHAGE OF THE ARTERY AT ITS ORIGIN FROM THE MAIN
TRUNK. WHICH OF THE FOLLOWING NERVES WILL MOST LIKELY BE IMMEDIATELY
AFFECTED BY THE HEMORRHAGE?
A. Optic nerve
B. Oculomotor nerve
C. Trochlear nerve
D. Trigeminal nerve
E. Abducens nerve
ANSWER: E.
 The anterior inferior cerebellar artery (AICA) is a major supplier of the anterior inferior portion of
the cerebellum. Nerves located in close proximity would likely be affected by hemorrhage of this
artery.
Q.24: A 34-YEAR-OLD MALE COMPLAINS OF HYPERACUSIS (SENSITIVITY TO LOUD
SOUNDS). INJURY TO WHICH OF THE FOLLOWING CRANIAL NERVES IS RESPONSIBLE?
A. Hypoglossal
B. Facial
C. Accessory
D. Vagus
E. Glossopharyngeal
ANSWER: B.
 The facial nerve innervates the stapedius muscle, which is responsible for limiting movement of
the stapes, thereby reducing the intensity of the sound entering the inner ear.
Q.25: A 29-YEAR-OLD WOMAN UNDERWENT A THYROIDECTOMY. POSTOPERATIVELY, THE
PATIENT PRESENTED WITH HOARSENESS. WHICH OF THE FOLLOWING NERVES WAS
MOST LIKELY INJURED DURING THE OPERATION?

Head & Neck (4TH semester—Unit-I PRACTICAL MANUAL) Page 158


A. Internal laryngeal
B. External laryngeal
C. Recurrent laryngeal
D. Superior laryngeal
E. Glossopharyngeal
ANSWER: C.
 The recurrent laryngeal nerve supplies most of the motor innervation to the larynx and sensation
below the true vocal folds. The thyroid gland and the recurrent laryngeal nerve are in close
proximity and thus the nerve is likely to be injured with a thyroidectomy. Injury to the recurrent
laryngeal nerve can result in speech defects, including hoarseness.
Q.26: A 55-YEAR-OLD WOMAN IS DIAGNOSED WITH A TUMOR AT THE BASE OF THE SKULL,
RESULTING IN A DECREASE IN TEAR PRODUCTION. WHICH OF THE FOLLOWING NERVES IS
MOST LIKELY INJURED?
A. Chorda tympani
B. Deep petrosal
C. Greater petrosal
D. Lesser petrosal
E. Nasociliary
ANSWER: C.
 The greater petrosal nerve, a parasympathetic branch of the facial nerve, provides innervation to
the lacrimal gland of the orbit.
Q.27: A 7-YEAR-OLD BOY WAS SUFFERING FROM A SEVERE INFECTION OF THE MIDDLE
EAR (OTITIS MEDIA), WHICH SPREAD TO THE MASTOID AIR CELLS (MASTOIDITIS).
SURGERY WAS REQUIRED BUT RESULTED IN THE FOLLOWING: RIGHT CORNER OF THE
MOUTH DROOPING, UNABLE TO CLOSE HIS RIGHT EYE, FOOD COLLECTION IN HIS RIGHT
ORAL VESTIBULE. WHICH NERVE WAS INJURED?
A. Glossopharyngeal
B. Vagus
C. Facial
D. Maxillary division of the trigeminal nerve
E. Mandibular division of the trigeminal nerve
ANSWER: C.
 A lesion of the facial nerve is likely to lead to the symptoms described (drooping mouth, unable to
close right eye, and food collection in the oral vestibule) because the muscles of facial expression
are paralyzed. There is a bony prominence over the facial nerve located on the medial wall of the
middle ear. Because of its close proximity, the facial nerve can be damaged due to otitis media.
Q.28: A 43-YEAR-OLD MAN IS ADMITTED TO THE EMERGENCY DEPARTMENT WITH A
FRACTURE OF THE BASE OF HIS SKULL. A THOROUGH PHYSICAL EXAMINATION REVEALS
THAT A NUMBER OF STRUCTURES HAVE BEEN INJURED, POSSIBLY INCLUDING THE RIGHT
GREATER PETROSAL NERVE. WHICH OF THE FOLLOWING CONDITIONS NEEDS TO BE
IDENTIFIED DURING PHYSICAL EXAMINATION TO CONFIRM THE DIAGNOSIS OF GREATER
PETROSAL NERVE INJURY?
A. Partial dryness of the mouth due to lack of salivary secretions from the submandibular and
sublingual glands
B. Partial dryness of the mouth due to lack of salivary secretions from the parotid gland
C. Dryness of the right cornea due to lack of lacrimal secretion
D. Loss of taste sensation from the right anterior two thirds of the tongue
E. Loss of general sensation from the right anterior two thirds of the tongue
ANSWER: C.
 The greater petrosal nerve carries parasympathetic fibers that are involved in the innervation of
the lacrimal gland, as well as the mucosal glands of the nose, palate, and pharynx. As a result, an
injury to the right greater petrosal nerve would be expected to result in decreased lacrimal
secretions for the right eye.
Q.29: A 55-YEAR-OLD WOMAN HAS UNDERGONE FACIAL SURGERY FOR THE EXCISION OF
A MALIGNANT PAROTID TUMOR. A WEEK POSTOPERATIVELY, MARKED WEAKNESS IS
SEEN IN THE MUSCULATURE OF THE PATIENT’S LOWER LIP. WHICH OF THE FOLLOWING
NERVES WAS MOST LIKELY INJURED DURING THE PAROTIDECTOMY ?

Head & Neck (4TH semester—Unit-I PRACTICAL MANUAL) Page 159


A. Marginal mandibular branch of facial
B. Zygomatic branch of facial
C. Mandibular division of the trigeminal nerve
D. Buccal branch of facial
E. Buccal nerve
ANSWER: A.
 The anterior division of the facial nerve passes through the parotid gland and is therefore at risk
during surgery of the parotid gland. Since this patient’s symptoms involved paralysis of the
muscles of the lower lip, the branch of the facial nerve that supplies these muscles, the marginal
mandibular branch, is the one that has suffered the iatrogenic injury.
Q.30: A 2-MONTH-OLD MALE INFANT IS ADMITTED TO THE HOSPITAL AFTER FALLING
FROM HIS STROLLER. DURING PHYSICAL EXAMINATION THE INFANT SHOWS SIGNS OF
FACIAL NERVE INJURY. WHAT IS THE MOST COMMON PLACE FOR FACIAL NERVE INJURY
IN AN INFANT?
A. At the stylomastoid foramen
B. Posterior to the parotid gland
C. Anterior to the parotid gland
D. Proximal to the stylomastoid foramen
E. Mandibular involvement of zygomatic and buccal branches
ANSWER: A.
 At the point where the facial nerve exits the stylomastoid foramen it is most susceptible to
shearing forces. In the absence of a skull fracture whereby the facial nerve can be damaged
within the facial canal, the nerve is most commonly injured as it exits the stylomastoid foramen. In
infants, in whom the mastoid process has not yet developed, the facial nerve lies unprotected, just
beneath the skin.
Q.31: A 62-YEAR-OLD MAN VISITS THE OUTPATIENT CLINIC COMPLAINING OF
SPONTANEOUS LACRIMATION DURING EATING. WHICH OF THE FOLLOWING NERVES HAS
DEVELOPED A LESION TO CAUSE THIS CONDITION?
A. Facial nerve proximal to the geniculate ganglion
B. Greater petrosal nerve
C. Lesser petrosal nerve
D. Lacrimal nerve
E. Chorda tympani
ANSWER: A.
 There is a lesion of the facial nerve proximal to the geniculate ganglion. At the geniculate ganglion
the greater petrosal nerve branches from the facial nerve and ultimately runs to the
pterygopalatine ganglion where preganglionic fi bers synapse on postganglionic neurons that
innervate the lacrimal gland. There is a disruption of the facial nerve proximal to this branch that
allows the greater petrosal nerve to be stimulated by factors that would normally stimulate the
submandibular and sublingual glands. These glands are innervated via the chorda tympani that
comes off the facial nerve distal to the geniculate ganglion.
Q.32: A 63-YEAR-OLD MAN WITH HEARING LOSS IN HIS LEFT EAR COMPLAINS OF A LOSS
OF TASTE AND DROOLING FROM THE LEFT SIDE OF HIS MOUTH. A CT SCAN SHOWS A
TUMOR COMPRESSING THE NERVE ENTERING THE SKULL THROUGH WHICH OF THE
FOLLOWING OPENINGS?
A. Foramen ovale
B. Foramen rotundum
C. Internal acoustic meatus
D. Jugular foramen
E. Superior orbital fissure
ANSWER: C.

 The tumor is compressing the facial nerve, which runs through the internal acoustic meatus along
with the vestibulocochlear nerve, which provides sense of taste to the anterior two thirds of the
tongue via the chorda tympani and also mediates all of the facial muscles, except the muscles of
mastication.

Head & Neck (4TH semester—Unit-I PRACTICAL MANUAL) Page 160


Q.33: IF THE LINGUAL NERVE IS DAMAGED AS IT ENTERS THE ORAL CAVITY WHICH OF
THE FOLLOWING STRUCTURES CONTAIN CELL BODIES OF INJURED NERVE FIBERS:

A. Geniculate and Otic ganglia


B. Trigeminal and Submandibular ganglia
C. Trigeminal and dorsal root ganglia
D. Geniculate and trigeminal ganglia
E. Geniculate and Pterygopalatine ganglia.
ANSWER: D.

Q.34: A 70-YEAR-OLD MAN IS ADMITTED TO THE HOSPITAL WITH CHRONIC HEADACHE


AND ENLARGED LYMPH NODES. A CT SCAN SHOWS A TUMOR AT THE JUGULAR
FORAMEN. WHICH OF THE FOLLOWING WOULD BE THE MOST LIKELY NEUROLOGIC
DEFICIT?
A. Loss of tongue movements
B. Loss of facial expression
C. Loss of sensation from the face and the scalp
D. Loss of hearing
E. Loss of gag reflex
ANSWER: E.
 The jugular foramen is the route of exit for three nerves (glossopharyngeal, vagus, and accessory
nerves) and one vein (internal jugular) from the cranial cavity. The glossopharyngeal nerve
provides the sensory input for the gag reflex, whereas the vagus nerve provides the motor output.
Nerve compression within this foramen would lead to a loss of both systems and thus no gag
reflex.
Q.35: A 36-YEAR-OLD WOMAN IS ADMITTED TO THE HOSPITAL WITH SEVERE HEAD
INJURIES AFTER A CAR CRASH. DURING NEUROLOGIC EXAMINATION HER UVULA IS
DEVIATED TO THE RIGHT. WHICH NERVE IS MOST LIKELY AFFECTED TO RESULT IN THIS
DEVIATION?
A. Left vagus
B. Right vagus
C. Right hypoglossal
D. Left glossopharyngeal
E. Right glossopharyngeal
ANSWER: A.
 An injury to the left vagus nerve would cause the uvula to become deviated to the right. This is
because of the innervation of the musculus uvulae muscle that makes up the core of the uvula. If
only one side is effectively innervated, contraction of the active muscle will deviate the uvula to
the contralateral side of the injury (ipsilateral side of the uninjured vagus nerve). In addition, the
intact levator veli palatini will pull the uvula to the intact side.
Q.36: A 34-YEAR-OLD SWIMMER PRESENTS TO YOUR OFFI CE WITH AN EXTERNAL EAR
CANAL INFECTION (OTITIS EXTERNA). THE PATIENT COUGHS DURING INSPECTION OF THE
EXTERNAL AUDITORY MEATUS WITH A SPECULUM. THE COUGH RESULTS FROM THE
IRRITATION OF WHICH NERVE THAT INNERVATES AN AREA OF THE EXTERNAL AUDITORY
MEATUS?
A. Vestibulocochlear
B. Vagus
C. Trigeminal
D. Facial
E. Accessory
ANSWER: B.
 The vagus nerve innervates a part of the external auditory meatus and, when stimulated, can
trigger a cough reflex in about 20% of people. This is thought to be due to “referred sensation”
from the vestibule of the larynx, which is innervated by the vagus nerve.
Q.37: A 34-YEAR-OLD WOMAN IS ADMITTED TO THE HOSPITAL WITH A LARGE MASS AT
HER THYROID GLAND. ULTRASOUND EXAMINATION REVEALS A BENIGN TUMOR. TWENTY-
FOUR HOURS FOLLOWING A PARTIAL THYROIDECTOMY, IN WHICH THE INFERIOR THYROID
ARTERY WAS ALSO LIGATED, THE PATIENT SPEAKS WITH A HOARSE VOICE AND HAS
DIFFICULTY IN BREATHING ON EXERTION. WHICH OF THE FOLLOWING NERVES WAS MOST
LIKELY INJURED DURING THE SURGICAL PROCEDURE?

Head & Neck (4TH semester—Unit-I PRACTICAL MANUAL) Page 161


A. Internal branch of superior laryngeal
B. Ansa cervicalis
C. Ansa subclavian
D. Recurrent laryngeal
E. External branch of superior laryngeal
ANSWER: D.
 The recurrent laryngeal nerve is the most likely nerve damaged during the surgery because it
runs in close proximity to the inferior thyroid artery and is easily injured or transected with the
artery if extreme care is not exercised during operative procedures. The recurrent laryngeal nerve
innervates the majority of the vocal muscles that open and close the rima glottidis, in addition to
providing sensory supply to the larynx below the vocal folds. Even relatively mild trauma to the
nerve can result in hoarseness.
Q.38: A 55-YEAR-OLD WOMAN IS ADMITTED TO THE HOSPITAL WITH DIFFI CULTY
SWALLOWING. PHYSICAL EXAMINATION REVEALS THAT THE PATIENT HAS EPISODES OF
SEVERE HEADACHES AND FREQUENTLY ASPIRATES FL UIDS WHEN DRINKING THEM. A
RADIOGRAPHIC EXAMINATION REVEALS A SKULL BASE TUMOR OCCUPYING THE SPACE
BEHIND THE JUGULAR FORAMEN. INVOLVEMENT OF WHICH OF THE FOLLOWING
STRUCTURES IS MOST LIKELY RESPONSIBLE FOR THE FI NDINGS IN THE PATIENT?
A. Ansa cervicalis
B. Cervical sympathetic trunk
C. External laryngeal nerve
D. Hypoglossal nerve
E. Vagus nerve
ANSWER: E.
 The vagus nerve exits the skull at the jugular foramen and is responsible for motor innervation to
the smooth muscles of the trachea, bronchi, and digestive tract, in addition to the muscles of the
palate, pharynx, larynx, and superior two thirds of the esophagus.
Q.39: A 34-YEAR-OLD WOMAN IS ADMITTED TO THE HOSPITAL WITH A LARGE MASS IN HER LOWER
ANTERIOR NECK. ULTRASONIC EXAMINATION REVEALS A BENIGN TUMOR OF HER THYROID GLAND.
TWENTY-FOUR HOURS FOLLOWING A PARTIAL THYROIDECTOMY, IT WAS NOTED THAT THE PATIENT
FREQUENTLY ASPIRATES FLUID INTO HER LUNGS. UPON EXAMINATION IT WAS DETERMINED THAT
THE AREA OF THE PIRIFORM RECESS ABOVE THE VOCAL FOLD OF THE LARYNX WAS
ANESTHETIZED. WHICH OF THE FOLLOWING NERVES WAS MOST LIKELY IATROGENICALLY
INJURED?
A. External branch of the superior pharyngeal
B. Hypoglossal
C. Internal branch of the superior laryngeal
D. Lingual
E. Recurrent laryngeal
ANSWER: C.
 During removal of the tumor, the internal branch of the superior laryngeal nerve was injured. Injury to this
nerve results in loss of sensation above the vocal cords, at the entrance to the larynx, and loss of taste on
the epiglottis. Loss of sensation in the laryngeal vestibule can precipitate aspiration of fluid into the larynx,
trachea, and lungs.
Q.40: A 56-YEAR-OLD WOMAN HAS JUST UNDERGONE A COMPLETE THYROIDECTOMY. AFTER SHE
RECOVERS FROM THE ANESTHESIA A HOARSENESS OF HER VOICE IS NOTED THAT PERSISTS FOR 3
WEEKS. SUBSEQUENT EXAMINATION SHOWS A PERMANENTLY ADDUCTED VOCAL FOLD ON THE
RIGHT SIDE. SURGICAL TRAUMA TO THE INNERVATION OF WHICH OF THE FOLLOWING MUSCLES IS
MOST LIKELY TO BE RESPONSIBLE FOR THE POSITION OF THE RIGHT VOCAL FOLD?
A. Aryepiglottic
B. Posterior cricoarytenoid
C. Thyroarytenoid
D. Transverse arytenoids
E. Vocalis
ANSWER: B.
 The recurrent laryngeal nerve is often at risk of being damaged during a thyroidectomy. Patients who have a
transected or damaged recurrent laryngeal will often present with a characteristic hoarseness following
surgery. The posterior cricoarytenoid is supplied by the recurrent laryngeal and would thus be impaired
following damage to the nerve. The posterior cricoarytenoid is the only muscle responsible for abduction of
the vocal cords, and paralysis of this muscle would result in a permanently adducted position of the involved
vocal cord.

Head & Neck (4TH semester—Unit-I PRACTICAL MANUAL) Page 162


Q.41: A 32-YEAR-OLD WOMAN UNDERWENT A THYROIDECTOMY. TWO MONTHS
POSTOPERATIVELY, IT WAS OBSERVED THAT THE PATIENT HAD LOST THE ABILITY TO
NOTICE THE PRESENCE OF FOREIGN OBJECTS IN THE LARYNGEAL VESTIBULE. WHICH OF
THE FOLLOWING NERVES WAS MOST LIKELY INJURED?
A. Internal laryngeal nerve
B. External laryngeal nerve
C. Glossopharyngeal nerve
D. Hypoglossal nerve
E. Recurrent laryngeal nerve
ANSWER: A.
 Damage to the internal laryngeal nerve would result in a general loss of sensation to the larynx
above the vocal cords, leaving the patient with an inability to detect food or foreign objects in the
laryngeal vestibule
Q.42: A 67-YEAR-OLD FEMALE IS ADMITTED TO THE EMERGENCY DEPARTMENT WITH A
SEVERE SWELLING ON THE RIGHT SIDE OF HER NECK. AN MRI EXAMINATION REVEALS AN
ABSCESS. THE ABSCESS IS SURGICALLY REMOVED FROM THE MIDDLE OF THE
POSTERIOR CERVICAL TRIANGLE ON THE RIGHT SIDE. DURING RECOVERY THE PATIENT
NOTICES THAT HER SHOULDER DROOPS AND SHE CAN NO LONGER RAISE HER RIGHT
HAND ABOVE HER HEAD TO BRUSH HER HAIR. WHICH OF THE FOLLOWING NERVES HAS
MOST LIKELY BEEN IATROGENICALLY INJURED?
A. Accessory
B. Ansa cervicalis
C. Facial
D. Hypoglossal
E. Suprascapular
ANSWER: A.
 The spinal accessory nerve innervates both the trapezius muscle and the sternocleidomastoid
muscle for the respective side of the body. Upon surgical division of the nerve, the patient will lose
the ability to raise the respective shoulder and will demonstrate weakness in turning the head to
the opposite side. The trapezius will also lose tone and the shoulder will droop.
Q.43: A 34-YEAR-OLD MAN IS ADMITTED TO THE HOSPITAL WITH SEVERE HEADACHES,
DIZZINESS, AND VOMITING. IMAGING STUDIES REVEAL A TUMOR AT THE HYPOGLOSSAL
CANAL. WHICH OF THE FOLLOWING MUSCLES WILL MOST LIKELY BE AFFECTED?
A. Geniohyoid
B. Mylohyoid
C. Palatoglossus
D. Genioglossus
E. Thyrohyoid
ANSWER: D.
 A tumor at the hypoglossal canal would compress the hypoglossal nerve and affect the
genioglossus, a muscle it supplies.
Q.44: A 56-YEAR-OLD MAN IS DIAGNOSED WITH AN EXTRADURAL TUMOR IN THE
POSTERIOR CRANIAL FOSSA. WHEN THE PATIENT PROTRUDED HIS TONGUE DURING
PHYSICAL EXAMINATION, THE TONGUE DEVIATED TO THE RIGHT. WHICH OF THE
FOLLOWING MUSCLES AND NERVES ARE MOST LIKELY INJURED?
A. Right hypoglossal nerve and right genioglossus
B. Left hypoglossal nerve and left genioglossus
C. Right hyoglossus and left styloglossus
D. Right geniohyoid and fi rst cervical nerve
E. Contralateral vagus and hypoglossal nerves
ANSWER: A.
 The hypoglossal nerve innervates the muscles of the tongue and is therefore directly involved in
alteration of shape and movement of the tongue. A lesion in this nerve would cause deviation of
the tongue toward the injured side, which could be observed upon protrusion of the tongue. The
genioglossus is the major muscle involved in protrusion of the tongue. The genioglossus muscles
arise from the inside of the mandible and pass posteriorly to insert into the deep aspect of the
tongue. When the genioglossi contract, they pull the tongue forward, and out of the mouth, in
protrusion. If one genioglossus is paralyzed, it acts like a brake on one side of the tongue when
the tongue is pulled forward, causing the tip of the tongue to point to the nonmoving side.

Head & Neck (4TH semester—Unit-I PRACTICAL MANUAL) Page 163


Q.45: A 45-YEAR-OLD MAN COMES TO YOU WITH DROOPING OF THE RIGHT ANGLE OF
MOUTH WITH WEAKNESS OF RIGHT SIDE OF THE FACE, THE MOST LIKELY DIAGNOSIS IS:

A. Right facial nerve palsy


B. Left facial nerve palsy
C. Right mandibular nerve palsy
D. Left mandibular nerve palsy
E. Right trigeminal nerve palsy
ANSWER: A.
Q.46: A 33-YEARS-OLD GENTLEMAN DEVELOPS VESICLES OVER UPPER EYELID. HE HAS
THE HISTORY OF CHICKEN POX INFECTION IN CHILDHOOD. IF THESE LESIONS ARE DUE
TO REACTIVATION OF VARICELLA ZOSTER VIRUS, WHICH NERVE IS MOST LIKELY
INVOLVED:

A. Supraorbital nerve
B. Trochlear nerve
C. Ophthalmic nerve
D. Facial nerve
E. Lacrimal nerve
ANSWER: A.
Q.47: A MIDDLE AGED WOMAN MET WITH AN ACCIDENT. SHE WAS RUSHED TO
EMERGENCY DEPARTMENT WITH INJURY ON HER LEFT FACE. RADIOGRAPHY REVEALED
FRACTURE OF RAMUS OF MANDIBLE. ON EXAMINATION IT WAS FOUND THAT THERE WAS
LOSS OF SENSATION IN THE LOWER TEETH & SKIN OF CHIN & LOWER LIP OF THAT SIDE
OF FACE. WHICH NERVE IS MOST LIKELY INVOLVED:

A. Inferior alveolar
B. Infraorbital
C. Maxillary
D. Mandibular
E. Mental
ANSWER: A.
Q.48: A PATIENT COMES WITH HEADACHE, DIPLOPIA, LOSS OF ACCOMMODATION & LIGHT
REFLEXES. LESION IS DIAGNOSED IN THE CILIARY GANGLION. PREGANGLIONIC FIBRES
TO CILIARY GANGLION ARE IN:

A. Short ciliary nerves


B. Long ciliary nerves
C. Oculomotor nerve
D. Nerve to inferior oblique
E. Nasociliary nerve
ANSWER: D.
Q.49: FOLLOWING A TONSILLECTOMY, YOUR PATIENT REPORTS NUMBNESS ON THE BACK
OF THE TONGUE & A REDUCED AWARENESS OF TASTE. AFTER EXAMINATION, YOU
CONFIRM A LOSS OF GENERAL SENSATION & TASTE FROM THE POSTERIOR THIRD OF THE
TONGUE. YOU SUSPECT INJURY TO THE:

A. Lingual nerve
B. Hypoglossal nerve
C. Facial nerve
D. Glossopharyngeal nerve
E. Vagus nerve
ANSWER: D.

Head & Neck (4TH semester—Unit-I PRACTICAL MANUAL) Page 164


Q.50: TO EVALUATE HYPOGLOSSAL NERVE FUNCTION, A NEUROLOGIST ASKS HER
PATIENT TO PROTRUDE HIS TONGUE. ON DOING SO, HIS TONGUE DEVIATES TO THE RIGHT
SIDE. THIS FINDING RESULTS FROM PARALYSIS OF WHICH OF THE FOLLOWING MUSCLES:

A. Left genioglossus
B. Left hyoglossus
C. Left palatoglossus
D. Right genioglossus
E. Right hyoglossus
ANSWER: D.

Q.51: A 24-YEAR-OLD WOMAN PRESENTS TO HER PHYSICIAN WITH AN INABILITY TO


CLOSE HER RIGHT EYE. PHYSICAL EXAMINATION REVEALS WEAKNESS OF THE RIGHT
ORBICULARIS OCULI. WHICH OF THE FOLLOWING ADDITIONAL SYMPTOMS WOULD LIKELY
ALSO BE PRESENT:

A. Blurred vision
B. Hyperacusis
C. Inability to chew
D. Inability to feel the face
E. Inability to shrug the shoulder
ANSWER: B.

Q.52: A PATIENT WITH CROCODILE TEARS SYNDROME HAS SPONTANEOUS LACRIMATION


DURING EATING DUE TO MISDIRECTION OF REGENERATING AUTONOMIC NERVE FIBERS.
WHICH OF THE FOLLOWING NERVES HAS BEEN INJURED:

A. Facial nerve proximal to the geniculate ganglion.


B. Auriculotemporal nerve.
C. Chorda tympani in the infratemporal fossa.
D. Facial nerve at the stylomastoid foramen.
E. Lacrimal nerve.
ANSWER: A.

XIV.CASE STUDY
Case—1
A 45-YEAR-OLD MAN WITH EXTENSIVE MAXILLOFACIAL INJURIES WAS BROUGHT TO THE
EMERGENCY DEPARTMENT. EVALUATION OF THE AIRWAY REVEALED PARTIAL
OBSTRUCTION. DESPITE AN OBVIOUS FRACTURED MANDIBLE, AN ATTEMPT WAS MADE
TO MOVE THE TONGUE FORWARD FROM THE POSTERIOR PHARYNGEAL WALL BY
PUSHING THE ANGLES OF THE MANDIBLE FORWARD. THIS MANEUVER FAILED TO MOVE
THE TONGUE, AND IT BECAME NECESSARY TO HOLD THE TONGUE FORWARD DIRECTLY
TO PULL IT AWAY FROM THE POSTERIOR PHARYNGEAL WALL.

1. The most likely reason the physician was unable to pull the tongue forward in this
patient is which?
ANS: The mandibular origin of the genioglossus muscles was floating because of bilateral fractures of
the body of the mandible — The genioglossus muscles arise from the superior mental spines behind
the symphysis menti of the mandible.

Case—2
A 43-YEAR-OLD WOMAN WAS SEEN IN THE EMERGENCY DEPARTMENT WITH A LARGE
ABSCESS IN THE MIDDLE OF THE RIGHT POSTERIOR TRIANGLE OF THE NECK. THE
ABSCESS WAS RED, HOT, AND FLUCTUANT. THE PHYSICIAN DECIDED TO INCISE THE
ABSCESS AND INSERT A DRAIN. THE PATIENT RETURNED TO THE DEPARTMENT FOR THE

Head & Neck (4TH semester—Unit-I PRACTICAL MANUAL) Page 165


DRESSINGS TO BE CHANGED 5 DAYS LATER. SHE STATED THAT SHE FELT MUCH BETTER
AND THAT HER NECK WAS NO LONGER PAINFUL. HOWEVER, THERE WAS ONE THING
THAT SHE COULD NOT UNDERSTAND. SHE COULD NO LONGER RAISE HER RIGHT HAND
ABOVE HER HEAD TO BRUSH HER HAIR.

1. What is the likely diagnosis?


ANS: The signs and symptoms in this case suggest that the spinal part of the accessory nerve had
been incised.

2. Which muscles are are likely to be paralysed?


ANS: The trapezius and sternocleidomastoid muscles, which are innervated by the spinal part of the
accessory nerve.

3. Why she is unable to raise her right hand above her head?
ANS: To raise the hand above the head, it is necessary for the trapezius muscle, assisted by the
serratus anterior, to contract and rotate the scapula so that the glenoid cavity faces upward.

Case—3
A 35-YEAR-OLD WOMAN HAD A PARTIAL THYROIDECTOMY FOR THE TREATMENT OF
HYROTOXICOSIS. DURING THE OPERATION A LIGATURE SLIPPED OFF THE RIGHT
SUPERIOR THYROID ARTERY. TO STOP THE HEMORRHAGE, THE SURGEON BLINDLY
GRABBED FOR THE ARTERY WITH ARTERY FORCEPS. THE OPERATION WAS COMPLETED
WITHOUT FURTHER INCIDENT. THE FOLLOWING MORNING THE PATIENT SPOKE WITH A
HUSKY VOICE.

1. Why the patient’s voice become husky?


ANS: The vocal cord is tensed by the contraction of the cricothyroid muscle. The cricothyroid muscle
is innervated by the external laryngeal nerve, which was damaged in this patient.

Case—4
A 43-YEAR-OLD WOMAN VISITED HER PHYSICIAN COMPLAINING OF SEVERE
INTERMITTENT PAIN ON THE RIGHT SIDE OF HER FACE. THE PAIN WAS PRECIPITATED BY
EXPOSING THE RIGHT SIDE OF HER FACE TO A DRAFT OF COLD AIR. THE PAIN WAS
STABBING IN NATURE AND LASTED ABOUT 12 HOURS BEFORE FINALLY DISAPPEARING.
WHEN ASKED TO POINT OUT ON HER FACE THE AREA WHERE THE PAIN WAS
EXPERIENCED, THE PATIENT MAPPED OUT THE SKIN AREA OVER THE RIGHT SIDE OF THE
LOWER JAW EXTENDING BACKWARD AND UPWARD OVER THE SIDE OF THE HEAD TO THE
VERTEX.

1. Which signs and symptoms in this patient strongly suggest a diagnosis of trigeminal
neuralgia?
ANS: The skin area where the patient experienced the pain was innervated by the mandibular division
of the trigeminal nerve. The stabbing nature of the pain is characteristic of the disease. The trigger
mechanism, stimulation of an area that received its sensory innervation from the trigeminal nerve, is
characteristic of trigeminal neuralgia. The motor portion of the trigeminal nerve is unaffected in
patients with trigeminal neuralgia.

Case—5
A 10-YEAR-OLD BOY WAS PLAYING DARTS WITH HIS FRIENDS. HE BENT DOWN TO PICK UP
A FALLEN DART WHEN ANOTHER DART FELL FROM THE DART BOARD AND HIT HIM ON
THE SIDE OF HIS FACE. ON EXAMINATION IN THE EMERGENCY DEPARTMENT A SMALL
SKIN WOUND WAS FOUND OVER THE RIGHT PAROTID SALIVARY GLAND. THEN 6 MONTHS
LATER, THE BOY'S MOTHER NOTICED THAT BEFORE MEALTIMES THE BOY BEGAN TO
SWEAT PROFUSELY ON THE FACIAL SKIN CLOSE TO THE HEALED DART WOUND.

Head & Neck (4TH semester—Unit-I PRACTICAL MANUAL) Page 166


1. On what basis the patient is diagnosed to have developed Frey’s syndrome
ANS: The point of the dart had entered the parotid salivary gland and damaged the parasympathetic
secretomotor fibers to the gland. The skin over the parotid salivary gland is innervated by the great
auricular nerve, which was also damaged by the dart. On regeneration of the damaged nerves some
of the parasympathetic nerves to the parotid salivary gland had crossed over and joined the
sympathetic secretomotor nerves to the sweat glands in the distal end of the great auricular nerve.

Case—6
A 35-YEAR-OLD WOMAN COMPLAINS OF A 2-MONTH HISTORY OF HOARSENESS OF HER
VOICE AND SOME CHOKING WHILE DRINKING LIQUIDS. SHE DENIES VIRAL ILLNESSES.
SHE UNDERWENT SURGERY FOR A COLD NODULE OF THE THYROID GLAND 9 WEEKS
AGO. HER ONLY MEDICATION IS ACETAMINOPHEN WITH CODEINE.
1. What is the most likely diagnosis?
ANS: Injury to the recurrent laryngeal nerve
2. What is the anatomical explanation for her symptoms?
ANS: Vocal cord paralysis
3. Which of the following muscles is most important to allow air movement through the
larynx?
ANS: The posterior cricoarytenoid muscles are the only muscles that abduct the vocal folds and are
necessary to widen the rima glottidis for breathing.
Case—7
A 33-YEAR-OLD WOMAN UNDERWENT PARTIAL THYROIDECTOMY FOR HYPERTHYROIDISM
IN WHICH THE THYROID FAILED TO TAKE UP RADIOACTIVE IODINE. SHE IS NOTED TO
HAVE SOME HOARSENESS OF VOICE 1 MONTH LATER. WHICH OF THE FOLLOWING IS THE
MOST LIKELY EXPLANATION?

ANS: Injury to the recurrent laryngeal nerve is common during thyroid surgery and may lead to the
inability to tightly adduct the two vocal folds, resulting in hoarseness. In addition, the protective
function of the rima glottidis may be lost, and food or liquid that does not go down the esophagus may
flow into the trachea and cause a choking response.

Case—8
A 15-YEAR-OLD BOY IS EATING A FISH DINNER AND INADVERTENTLY HAS A BONE
“CAUGHT IN HIS THROAT.” HE COMPLAINS OF SIGNIFICANT PAIN ABOVE THE VOCAL
CORDS. WHICH OF THE FOLLOWING NERVES IS RESPONSIBLE FOR CARRYING THE
SENSATION FOR THIS PAIN?

ANS: The laryngeal mucosa above the vocal cords is innervated by the superior laryngeal nerve,
whereas mucosa below the vocal cords is innervated by the recurrent laryngeal nerve.

Case—9
A 25-YEAR-OLD WOMAN UNDERWENT SURGERY FOR A THYROID NODULE. TWO MONTHS
LATER, SHE COMPLAINS OF DRYNESS OF SKIN AND MUSCLE SPASMS. WHICH OF THE
FOLLOWING IS THE MOST LIKELY EXPLANATION?

ANS: This patient likely has hypocalcemia due to excision of the parathyroid glands.

Case—10
A 28-YEAR-OLD WOMAN AT 19 WEEKS OF PREGNANCY COMPLAINS OF ACUTE ONSET OF
NUMBNESS OF THE RIGHT CHEEK AND DROOPING OF THE RIGHT FACE THAT OCCURRED
OVER 1 HOUR. SHE DENIES TRAUMA TO THE HEAD. ON EXAMINATION, THE PATIENT HAS
DIFFICULTY CLOSING HER RIGHT EYELID, AND HER RIGHT NASOLABIAL FOLD IS
SMOOTHER THAN ON THE LEFT. SHE ALSO IS DROOLING FROM THE RIGHT SIDE OF HER
MOUTH. THE REMAINDER OF THE NEUROLOGICAL EXAMINATION IS NORMAL.

Head & Neck (4TH semester—Unit-I PRACTICAL MANUAL) Page 167


1. What is the most likely diagnosis?
ANS: Bell palsy (idiopathic seventh cranial nerve palsy)

2. What is the anatomical mechanism for this condition?


ANS: Dysfunction of the peripheral portion of the seventh cranial nerve

3. Why eye is vulnerable?


ANS: The eye is vulnerable to dryness due to impaired blinking. Damage to the intracranial course of
parasympathetic fibers in the greater petrosal nerve may also contribute to decreased stimulation of
the lacrimal gland.

4. What is Bell’s palsy?


ANS: Idiopathic palsy of peripheral CN VII leading to ipsilateral facial weakness.

Case—11
A 44-YEAR-OLD MAN COMPLAINS OF DIFFICULTY HEARING FROM THE RIGHT EAR AND
HEADACHES. HE ALSO HAS WEAKNESS OF THE FACIAL MUSCLES. WHICH OF THE
FOLLOWING IS THE MOST LIKELY EXPLANATION?

ANS: When multiple nerves are affected, it is unlikely to be a peripheral disorder. Cranial nerves VII
and VIII exit in close proximity from the pons. A schwannoma involving the cerebellopontine angle can
affect both cranial nerves.

Case—12
AN INJURY TO THE FACIAL NERVE (CN VII) AS IT LEAVES THE STYLOMASTOID FORAMEN
WOULD DISRUPT WHICH FUNCTION?

ANS: Wrinkling the forehead is produced by contraction of the frontalis muscle, which is innervated by
the facial nerve. The facial nerve is responsible for taste in the anterior two-thirds of the tongue, but
the chorda tympani emerges before the main trunk exits through the stylomastoid foramen. Sensation
of the cornea and sensation to the cheek are supplied by the trigeminal nerve.

Case—13
A 33-YEAR-OLD WOMAN SUFFERED A SKULL FRACTURE THAT LED TO A UNILATERAL
FACIAL NERVE PALSY. WHICH OF THE FOLLOWING FRACTURES WAS MOST LIKELY
RESPONSIBLE?

ANS: The basilar fracture involving the mastoid region of the temporal bone may impinge on the facial
nerve as it exits the stylomastoid foramen.

Head & Neck (4TH semester—Unit-I PRACTICAL MANUAL) Page 168


AUTONOMIC NERVES & CERVICAL
PLEXUS
I. PARASYMPATHETIC GANGLIA
1. Ciliary ganglion:
LOCATION PREGANGLIONIC POSTGANGLIONIC
NEURONS NEURONS
It is situated within the Their cell bodies are situated Their cell bodies are situated
orbit, on the lateral within the Edinger- within the ciliary ganglion.
side of the optic nerve. westphal nucleus located
in the midbrain. Their axons are carried by
the short ciliary nerves,
Their axons are carried by the which pierce the sclera of
oculomotor nerve, then its the eyeball to supply :
inferior division and finally
its branch to the inferior The constrictor papillae
oblique muscle. muscle of the iris (whose
contraction narrows down
These axons relay in the ciliary the size of pupil)
ganglion.
The ciliary muscle (whose
contraction increases the
convergence power of the
lens to cause accommodation
for the near vision).

2. Otic ganglion:
LOCATION PREGANGLIONIC POSTGANGLIONIC
NEURONS NEURONS
It is situated within the Their cell bodies are situated Their cell bodies arte situated
infratemporal fossa, just within the inferior within the otic ganglion.
close to the foramen ovale. salivatory nucleus, located
in the pons. Their axons are carried by the
auriculotemporal nerve,
Their axons are carried by the which is one of the
glossopharyngeal nerve, branches of the mandibular
then its tympanic branch nerve.
and finally its lesser
petrosal branch which These axons provide
passes through the secretomotor supply to the
foramen ovale. parotid gland.

These axons relay in the otic


ganglion.

Head & Neck (4TH semester—Unit-I PRACTICAL MANUAL) Page 169


3. Submandibular ganglion:
LOCATION PREGANGLIONIC POSTGANGLIONIC
NEURONS NEURONS
It is situated within the Their cell bodies are situated withinTheir cell bodies are situated
submandibular region, on the the superior within the submandibular
hyoglossus muscle, under the salivatory nucleus, located ganglion.
cover of mylohyoid. in the pons.
Their axons are carried by
Their axons are carried by the lingual nerve.
the facial nerve, then its
chorda tympani branch and These axons provide
which passes through the secretomotor supply to the
petrotympanic fissure and submandibular and
immediately joins the lingual sublingual glands.
nerve ( a branch of the
posterior division of the
mandibular nerve).

These axons relay in the


submandibular ganglion.
It is situated within the
submandibular region, on the
hyoglossus muscle, under the
cover of mylohyoid.

4. Pterygopalatine ganglion:
LOCATION PREGANGLIONIC POSTGANGLIONIC
NEURONS NEURONS
It is situated within the Their cell bodies are situated within Their cell bodies are situated
pterygopalatine fossa, close the superior salivatory within the pterygopalatine
to the maxillary nerve. nucleus, located in the pons. ganglion.
Their axons are carried by
Their axons are carried by the the:
facial nerve, then its greater =Greater palatine nerve,
petrosal branch and which =Lesser palatine nerve,
passes through the greater =Pharyngeal nerve,
petrosal foramen and =Sphenopalatine nerve and
immediately joins the deep =Zygomaticotemporal branch of
petrosal nerve (a branch of the zygomatic nerve.
the carotid plexus of nerves),
to form the nerve of pterygoid These axons provide
canal. secretomotor supply to the
pharyngeal, nasal and
The nerve of pterygoid canal palatal and lacrimal glands.
enters the pterygopalatine
fossa and relay in the
pterygopalatine ganglion.

Head & Neck (4TH semester—Unit-I PRACTICAL MANUAL) Page 170


II. CERVICAL SYMPATHETIC TRUNK
1. Introduction
 The cervical portion of sympathetic trunk extends superiorly to the level of C1 vertebra — It is
covered by the prevertebral fascia, and lies deep to the carotid sheath —The cervical portion of
the trunks includes three cervical sympathetic ganglia—superior, middle and inferior — These
ganglia receive preganglionic sympathetic fibres via upper 3-4 thoracic spinal nerves —
After synapsing with the neurons in the cervical sympathetic ganglia, postganglionic fibres go
to:
Cervical spinal nerves
Thoracic viscera
Head and viscera of the neck

2. Distribution of postganglionic fibers of superior


cervical ganglion (SCG)
 Internal carotid nerve: it contains postganglionic fibers that pass from SCG to form the
internal carotid sympathetic plexus to enter the cranial cavity.
 External carotid nerve: it contains postganglionic fibers that pass from SCG to form the
external carotid sympathetic plexus to be distributes to the face and scalp.
 Pharyngeal branches: they join the pharyngeal branches of the glossopharyngeal and
vagus nerves to contribute the vasoconstrictor component of the pharyngeal plexus.
 Gray rami to the anterior rami of the superior four cervical spinal nerves.
 Superior cervical cardiac nerve: which is connected to the cardiac plexus.

3. Distribution of postganglionic fibers of middle cranial


ganglion (MCG)
 Gray rami to the anterior rami of the 5th and 6th cervical spinal nerves (roots
of the brachial plexus)
 Middle cervical cardiac nerve: This is connected to the cardiac plexus.
 Thyroid plexus: it contains postganglionic fibers that pass from MCG by means of
periarterial branches around the inferior thyroid artery running to form plexuses to the thyroid
gland.

4. Distribution of postganglionic fibers of inferior


cervical ganglion (ICG)
 Gray rami to the anterior rami of 7th and 8th cervical spinal nerves (roots of
the brachial pluxes)
 Inferior cervical cardiac nerve: which is connected to cardiac plexus.
 Vertebral plexus: it contains postganglionic fibers that pass from ICG by means of
periarterial branches around the vertebral artery running into the cranial cavity.

Head & Neck (4TH semester—Unit-I PRACTICAL MANUAL) Page 171


III. CERVICAL PLEXUS (Fig—61)
 It is formed the anterior rami of the C, 2, 3 & 4 nerves.

Fig-61: Cervical plexus.

1. Menigeal branches
 Upper 3 cervical nerves have meningeal branches for dura mater of the posterior cranial fossa.
a. Meningeal branch from C1
b. Meningeal branches from C2 & 3

2. Segmental muscular branches


 They give segment muscular branches to the prevertebral muscles.
3. Phrenic nerve
 It is formed mainly from C4, with contribution from C3 and C5 — It is the sole motor supply to its own
half of the diaphragm.
4. Cutaneous branches
a. Lesser occipital nerve: supplies skin of the posterior part of neck.
b. Great auricular nerve: supplies skin over the angle of mandible.
c. Transverse cervical nerve: supplies skin from chin to jugular notch.
d.Supraclavicular nerves: supplies skin as far down as the 2nd rib.

5.Formation of ansa cervicalis


 Ansa cervicalis usually lies in the anterior fascial layer of the CAROTID SHEATH.

Head & Neck (4TH semester—Unit-I PRACTICAL MANUAL) Page 172


a. Superior root of ansa cerviacalis
 The hypolossal nerve is joined by a loop from C1 — After joining the C1, the hypoglossal nerve gives off a
superior root of ansa cervicalis — It contains the fibres of C1 ( not the hypoglossal nerve).
b. Infrior root of ansa cervicalis
 It is derived from the C2 & 3 — It joins the superior root to from the ansa cerviclis — The superior root
itself provides motor fiber to superior belly of omohyoid, muscle.
6. Other musclear branches of hypoglossal nerve
a. Nerve to thyrohyoid
b. Nerve to geniohyoid

IV.REVIEW QUESTIONS
Q.1: A 31-YEAR-OLD FEMALE IS ADMITTED TO THE HOSPITAL AFTER AN AUTOMOBILE
COLLISION. A CT SCAN EXAMINATION REVEALS A LARGE HEMATOMA INFERIOR TO THE
RIGHT JUGULAR FORAMEN. PHYSICAL EXAMINATION REVEALS RIGHT PUPILLARY
CONSTRICTION (MIOSIS) AND ANHYDROSIS (LOSS OF SWEATING) OF THE FACE. WHICH OF
THE FOLLOWING GANGLIA IS MOST LIKELY AFFECTED BY THE HEMATOMA?
A. Submandibular
B. Trigeminal (semilunar or Gasserian)
C. Superior cervical
D. Geniculate
E. Ciliary
ANSWER: C.
 The superior cervical ganglion (SCG), which is the uppermost part of the sympathetic chain,
supplies sympathetic innervation to the head and neck. The usual symptoms for SCG injury are
miosis and anhydrosis in the head and neck region. Postganglionic sympathetic nerves usually
run alongside the arteries leading into the head and neck region
Q.2: A 48-YEAR-OLD MAN PRESENTS WITH A CONSTRICTED RIGHT PUPIL THAT DOES NOT
REACT TO LIGHT. HIS LEFT PUPIL AND VISION IN BOTH EYES ARE NORMAL. THESE
FINDINGS ARE MOST LIKELY DUE TO A LESION INVOLVING WHICH OF THE FOLLOWING
RIGHT-SIDED STRUCTURES?
A. Oculomotor nerve
B. Superior cervical ganglion
C. Nervus intermedius
D. Edinger-Westphal nucleus
E. Trigeminal (semilunar, Gasserian) ganglion
ANSWER: B.
 The superior cervical ganglion provides sympathetic innervation to the face and neck regions.
Sympathetics travel along the branches of the internal carotid artery, and one result of stimulation
of these nerves is to dilate the pupil during a sympathetic response (“flight or fight”)…… The
oculomotor nerve would not affect the dilation of the pupil; rather, its stimulation results in the
constriction (parasympathetic nerves).
Q.3: AN 8-YEAR-OLD MALE IS ADMITTED TO THE HOSPITAL WITH A DROOPING RIGHT
EYELID (PTOSIS). THE INITIAL DIAGNOSIS IS HORNER’S SYNDROME. WHICH OF THE
FOLLOWING ADDITIONAL SIGNS ON THE RIGHT SIDE WOULD CONFIRM THE DIAGNOSIS?
A. Constricted pupil
B. Dry eye
C. Exophthalmos
D. Pale, blanched face
E. Sweaty face
ANSWER: A.

Head & Neck (4TH semester—Unit-I PRACTICAL MANUAL) Page 173


 Horner’s syndrome involves interruption of sympathetic supply to the face. This results in ptosis
(drooping eyelid), miosis (constricted pupil), and anhydrosis (lack of sweating) of the face……..
The eye is lubricated by the lacrimal gland, which secretes in response to parasympathetic
stimulation, and would be unaffected.
Q.4: A 72-YEAR-OLD WOMAN IS ADMITTED TO THE EMERGENCY DEPARTMENT WITH
TENDERNESS IN THE UPPER RIGHT THORAX, PAINFUL TO COMPRESSION. DURING
PHYSICAL EXAMINATION THE PATIENT PRESENTS WITH SLIGHT PTOSIS OF HER RIGHT
EYE. THE RIGHT PUPIL IS CONSTRICTED MORE DISTINCTLY THAN THE CONTRALATERAL
PUPIL. WHICH OF THE FOLLOWING IS THE MOST LIKELY DIAGNOSIS?
A. Raynaud’s disease
B. Frey syndrome
C. Bell palsy
D. Quinsy
E. Pancoast tumor

ANSWER: E.
 A Pancoast tumor is located in the pulmonary apex, usually in the right lung. (This is because
inhaled gases tend to collect preferentially in the upper right lung, in part because of the manner
of branching of the tertiary bronchi.) These tumors can involve the sympathetic chain ganglia and
cause Horner’s syndrome (slight ptosis and miosis).
Q.5: A 32-YEAR-OLD MAN IS ADMITTED TO THE HOSPITAL WITH SEVERE HEADACHE AND
VISUAL PROBLEMS. THE DILATOR PUPILLAE MUSCLE, THE SMOOTH MUSCLE CELL FI
BERS OF THE SUPERIOR TARSAL MUSCLE (OF MÜLLER, PART OF THE LEVATOR
PALPEBRAE SUPERIORIS), AND THE SMOOTH MUSCLE CELLS OF THE BLOOD VESSELS OF
THE CILIARY BODY ARE SUPPLIED BY EFFERENT NERVE FIBERS. WHICH OF THE
FOLLOWING STRUCTURES CONTAINS THE NEURAL CELL BODIES OF THESE FIBERS?
A. Pterygopalatine ganglion
B. IML (lateral horn) C1 to C4
C. Geniculate ganglion
D. Nucleus solitarius
E. Superior cervical ganglion
ANSWER: E.
 The dilator pupillae, levator palpebrae superioris, and smooth muscle cells of blood vessels in the
ciliary body all receive sympathetic innervation. The postsynaptic cell bodies of the sympathetic
neurons that innervate these structures are located in the superior cervical ganglion. The
intermediolateral cell column contains presynaptic sympathetic neurons, but it is located only at
spinal cord levels T1 to L2.
Q.6: WHERE IS THE LOCATION OF THE POSTGANGLIONIC PARASYMPATHETIC NEURAL
CELL BODIES THAT DIRECTLY INNERVATE THE PAROTID GLAND?
A. Trigeminal (semilunar, Gasserian) ganglion
B. Inferior salivatory nucleus
C. Superior cervical ganglion
D. Otic ganglion
E. Submandibular ganglion
ANSWER: D.
 The otic ganglion is the location of the postganglionic parasympathetic neural cell bodies
innervating the parotid gland. The ganglion lies on the mandibular division of the trigeminal nerve
near the foramen ovale.
Q.7: A 43-YEAR-OLD MAN IS DIAGNOSED WITH LARYNGEAL CARCINOMA. A SURGICAL
PROCEDURE IS PERFORMED AND THE TUMOR IS SUCCESSFULLY REMOVED FROM THE
LARYNX. THE RIGHT ANSA CERVICALIS IS ANASTOMOSED WITH THE RIGHT RECURRENT
LARYNGEAL NERVE IN ORDER TO REINNERVATE THE MUSCLES OF THE LARYNX AND
RESTORE PHONATION. WHICH OF THE FOLLOWING MUSCLES WILL MOST LIKELY BE
PARALYZED AFTER THIS OPERATION?
A. Sternocleidomastoid
B. Platysma
C. Sternohyoid
D. Trapezius
E. Cricothyroid
ANSWER: C.

Head & Neck (4TH semester—Unit-I PRACTICAL MANUAL) Page 174


 Because of the surgical division of the ansa cervicalis, the sternohyoid muscle will most likely be
paralyzed following this tumor resection. The ansa cervicalis innervates the strap muscles,
including the sternohyoid, sternothyroid, and omohyoid muscles.
Q.8: A 20-YEAR-OLD MAN IS ADMITTED TO THE EMERGENCY DEPARTMENT WITH A STAB
WOUND IN THE SUPERIOR REGION OF HIS NECK. A RADIOGRAPHIC EXAMINATION
REVEALS THAT THE WOUND HAS NOT AFFECTED ANY MAJOR STRUCTURES. PHYSICAL
EXAMINATION REVEALS THAT THE PATIENT HAS LOST SENSATION FROM THE SKIN OVER
THE ANGLE OF THE JAW. WHICH OF THE FOLLOWING NERVES IS MOST LIKELY INJURED?
A. Supraclavicular
B. Transverse cervical
C. Great auricular
D. Greater occipital
E. Lesser occipital
ANSWER: C.
 Four nerves participate in providing cutaneous supply to the neck. The nerves are the
supraclavicular, great auricular, transverse cervical, and the lesser occipital. The area over the
angle of the jaw is innervated by the great auricular nerve. It ascends from spinal segments from
C2 and C3 and innervates the skin over the angle of the jaw and posteroinferior to the auricle of
the ear.
Q.9: CARDIAC PAIN IS REFERRED IN SOME CASES TO THE MANDIBLE AND THE REGION OF
THE TMJ. CUTANEOUS SENSATION OVER THE ANGLE OF THE MANDIBLE IS NORMALLY
SUPPLIED BY WHICH OF THE FOLLOWING0 NERVES?
A. C ervical branch of facial
B. G reat auricular nerve
C. M andibular branch of trigeminal nerve
D. M andibular branch of facial nerve
E. T ransverse cervical nerve
ANSWER: B.
 The great auricular nerve is derived from the ventral rami of the second and third cervical nerves
and supplies the skin over the angle of the mandible up to the level of the TMJ.
Q.10: A 65-YEARS-OLD LADY COMPLAINTS OF DRY EYES WHICH GANGLION IS MOST
LIKELY INVOLVED:

A. Otic
B. Pterygopalatine
C. Ciliary
D. Superior cervical
E. Submandibular
ANSWER: B.
Q.11: ON EXAMINATION OF A PATIENT WITH DAMAGE TO THE LEFT CERVICAL
SYMPATHETIC CHAIN GANGLIA AS A RESULT OF A NECK TUMOR, WHICH OF THE
FOLLOWING PHYSICAL SIGNS WOULD BE EXPECTED:

A. Ptosis on the left


B. Pupil dilation of the left eye
C. Lateral deviation of the left eye
D. Increased sweat secretion on the left side of face
E. Pale skin on the left side of the face.
ANSWER: A.
Q.12: AN ARMY MAN DURING BATTLE HAD A FIREARM INJIRY ON THE LEFT SIDE OF THE
NECK, PROBABLY, HE WILL HAVE ON THAT SIDE:

A. Complete ptosis
B. Pupil will not react to light
C. Loss of accommodation
D. Miosis
E. Increased sweating

Head & Neck (4TH semester—Unit-I PRACTICAL MANUAL) Page 175


ANSWER: D.

Head & Neck (4TH semester—Unit-I PRACTICAL MANUAL) Page 176

You might also like